Kaplan Cumulative: GI/GU/Endo

Ace your homework & exams now with Quizwiz!

65. Patients with inflammatory bowel disease may develop signs separate from involvement of the gastrointestinal tract. Which of the following extraintestinal manifestations of inflammatory bowel disease is more commonly seen in Crohn disease than ulcerative colitis? A. Erythema nodosum B. Pyoderma gangrenosum C. Rheumatoid nodules D. Sclerosing cholangitis E. All of the above

A. Erythema nodosum

306. A 43-year-old man reports a 9-pound weight loss over the past 9 months, accompanied by difficulty swallowing solids and liquids. He has woken on several occasions at 4 AM and regurgitated partially digested dinner contents. An upper gastrointestinal series reveals a widely dilated esophagus with a smoothly tapering distal esophagus. There appears to be partially digested food present in the esophagus. Which of the following is the most likely diagnosis? A. Achalasia B. Diffuse esophageal spasm C. Esophageal squamous cancer D. Peptic stricture E. Scleroderma

A. Achalasia

62. A 34-year-old bus driver comes to the health care provider with complaints of difficulty swallowing. The symptoms began gradually, approximately 9 months ago, and have prevented him from both swallowing solids or drinking liquids comfortably. He awakens at night with cough and occasional morning regurgitation of recognizable food from the night before. He has reduced his oral intake and has lost 6 pounds over the past 2 months. He does not smoke or drink,and has no family history of esophageal or other gastrointestinal illnesses. Physical examination is unremarkable. Which of the following is the most likely diagnosis? A. Achalasia B. Esophageal adenocarcinoma C. Lower esophageal web D. Peptic stricture E. Progressive systemic sclerosis

A. Achalasia

287. A 31-year-old man describes a burning sensation in the substernal area after eating chocolate, caffeine, or alcohol. The symptoms are exacerbated at night, and he has awoken on several occasions from sleep because of coughing. Over-the-counter antacids and H2 receptor antagonists provide little relief. He often takes antacids before dinner. Which of the following is the most likely underlying cause for his coughing? A. Acid-induced bronchoconstriction B. Aspiration of antacids taken before bedtime C. Aspiration of solid food ingested with dinner D. H2-receptor antagonist-induced bronchoconstriction E. Tracheoesophageal fistula

A. Acid-induced bronchoconstriction

248. A 23-year-old professional basketball player presents to his health care provider 3 hours before game time complaining of abdominal pain. The symptoms began approximately 8 hours ago in a diffuse fashion. Two hours later, he began feeling nauseated and vomited twice. Over the past 4 hours the abdominal pain has become more severe and well-localized in the right lower quadrant. His examination now reveals well-localized pain in the right lower quadrant inferolateral to the umbilicus. Which of the following is the most likely diagnosis? A. Acute obstruction of the appendiceal lumen by a fecalith B. Acute onset of ileocolitis C. Acute onset of ischemic colitis D. Acute Yersinia infection E. Obstruction of the ileocecal valve by a mass

A. Acute obstruction of the appendiceal lumen by a fecalith

A 53-year-old man comes to the health care provider because of progressive weakness and weight loss over the past 2 months. He says that he also began noticing areas of his skin getting darker even though it is winter and he is never in the sun. He takes no medications and has no other medical conditions. Physical examination shows no abnormalities except for orthostatic hypotension and hyperpigmentation of the skin, which is primarily seen in the skin folds. Laboratory studies show: Sodium 130 mEq/L (normal 135-146 mEq/L) Chloride 95 mEq/L (normal 96-106 mEq/L) Potassium 6.5 mEq/L (normal 3.5-5.2 mEq/L) Bicarbonate 20 mEq/L (normal 20-29 mEq/L) Leukocyte count 5,000/mm3 (normal 5,000-10,000/mm3) Segmented neutrophils 40% (normal 40-60%) Band forms 4% (normal 0-3%) Lymphocytes 40% (normal 20-40%) Monocytes 6% (normal 2-8%) Eosinophils 9.5% (normal 1-4%) Basophils 0.5% (normal 0.5-1%) Which of the following is the most likely diagnosis? A. Addison disease B. Conn syndrome C. Cushing disease D. Cushing syndrome E. Syndrome of inappropriate antidiuretic hormone secretion

A. Addison disease This patient most likely has Addison disease, which is primary adrenocortical deficiency. It is a rare disease caused by a progressive destruction of the adrenal glands, usually because of idiopathic atrophy, autoimmune disease, surgery, infection, or hemorrhage. Clinical symptoms include weakness, weight loss, hyperpigmentation, nausea and vomiting, and hypotension. Laboratory findings include hyponatremia (caused by aldosterone deficiency and inability to retain salt), hyperkalemia, and normocytic anemia with eosinophilia and lymphocytosis. Diagnosis is made with the ACTH stimulation test. Cortisol and aldosterone levels do not increase when the ACTH is given in primary adrenal insufficiency because the adrenal gland is unable to make these hormones. Treatment is lifetime glucocorticoid and mineralocorticoid replacement.

A 29-year-old man comes to the clinic with a 2-day history of severe left-sided scrotal pain and swelling. He is sexually active and has multiple sexual partners. He has no history of sexually transmitted diseases. His temperature is 38.2° C (100.8° F), blood pressure 120/70 mm Hg, and pulse 80/min. Examination shows unilateral intra-scrotal tenderness and swelling. Testicular support makes the pain less intense. Which of the following is the most likely diagnosis? A. Epididymitis B. Prostatitis C. Testicular torsion D. Urethritis E. Varicocele

A. Epididymitis most likely caused by chlamydia - supported by hx and positive phren's sign [Tx w/ Ceftriaxone & Doxy]

99. A 63-year-old woman comes to the emergency department with severe abdominal pain. She reports the pain is epigastric and worse with eating. There is some radiation to the back. There is associated diarrhea that is described as "fatty." Symptoms have been present for 3 to 4 months. Her medical history is significant for 2 episodes of acute pancreatitis, hypertension, and hyperlipidemia, but she takes no medications for these disorders. Her temperature is 38.3ºC (100.9ºF), blood pressure 160/90 mm Hg, pulse 143/min, and respirations 16/min. Physical examination shows epigastric tenderness, mild distention, and decreased bowel sounds. She appears uncomfortable and in moderate distress. An obstruction series reveals only scattered pancreatic calcifications. Laboratory studies show: Amylase 45 U/L (normal 23-140 U/L) Lipase 10 U/L (normal <160 U/L) Albumin 2.8 g/dL (normal 3.4-5.4 g/dL) AST 98 IU/L (normal <40 IU/L) ALT 45 IU/L (normal <40 IU/L) Total bilirubin 1.1 mg/dL (normal <1.9 mg/dL) Which of the following is the most likely underlying etiology for this patient's symptoms? A. Alcohol B. Elevated triglycerides C. Gallstones D. Hypercalcemia E. Idiopathic

A. Alcohol

193. A patient is seen in the office with anorexia, nausea, vomiting, fever, jaundice, and tender hepatomegaly. The patient has laboratory testing revealing aspartate aminotransferase (AST) elevated more than twice the alanine aminotransferase (ALT) level. Which of the following is the most likely diagnosis? A. Alcoholic hepatitis B. Non-alcoholic fatty liver disease (NAFLD) C. Autoimmune hepatitis D. Primary sclerosing cholangitis E. Alcohol-related fatty liver disease

A. Alcoholic hepatitis

82. A 66-year-old man comes to the urgent care clinic with progressive jaundice, which he first noticed 6 weeks ago. He has never had any chronic medical conditions and takes no medications. He has smoked 1 pack of cigarettes a day for 30 years and drinks a glass of wine each evening. His total bilirubin is 22 mg/dL (normal <1.9 mg/dL), with a direct (conjugated) bilirubin 16 mg/dL (normal <0.3 mg/dL). Transaminases are minimally elevated, whereas his alkaline phosphatase is about 6 times the upper limit of normal. Sonogram shows dilated intrahepatic ducts, dilated extrahepatic ducts, and a very distended, thin-walled gallbladder without stones. Which of the following is the most appropriate next step in diagnosis? A. CT scan of the upper abdomen B. Endoscopic retrograde cholangiopancreatography (ERCP) C. Exploratory laparotomy D. Percutaneous transhepatic cholangiogram (PTC) E. Serologies to define the type of hepatitis

A. CT scan of the upper abdomen

17. A 54-year-old man has cirrhosis, with obstruction of the portal circulation within the liver. Portal blood could still be conveyed to the caval system via which of the following? A. Azygos and hemiazygos veins B. Gonadal veins C. Internal iliac veins D. Splenic vein E. Vesical venous plexus

A. Azygos and hemiazygos veins

123. A 5-month-old boy is brought to the emergency department by his parents because he has been having crying spells alternating with lethargy for the past several hours. He was apparently doing fine in the morning and feeding well, when he suddenly started crying painfully, drawing his legs to his chest and kicking them in the air. This would last for 10-15 minutes, and then he would act very tired and drowsy. First they thought it was colic, but after several episodes of similar attacks he started vomiting and passed a stool that contained red blood and mucus, and they immediately brought him to the hospital. He has no medical problems and has been developing well. He is breast-fed and the mother had started adding solids over the previous 3-4 weeks. On physical examination, the patient is a chubby infant in moderate distress, pale, and diaphoretic. His temperature is 36.7ºC (98.0ºF), pulse is 100/min, and respirations are 22/min. A complete blood count and leukocyte differential are within normal limits. His abdomen is mildly tender to palpation but not distended, and there is a sausage-shaped mass in the right hypochondrium. The right lower quadrant feels empty on palpation. His stool is reddish in color and with hints of gelatinous material. A plain abdominal radiograph shows absence of air in the right lower quadrant. Which of the following is the most appropriate next step in the treatment? A. Barium enema B. Immediate surgical reduction C. Laparoscopy D. Manual reduction E. Observation in expectation of spontaneous resolution

A. Barium enema

A 65-year-old woman is admitted to the hospital for constant, severe abdominal pain that has worsened over the previous week. She has no other associated symptoms, such as nausea or vomiting, but has noticed that her daily urine output has sharply decreased. She has had a constant desire to urinate, but when she tries, only a small amount of bloody urine is discharged. The patient is a long-time smoker, having smoked 3 packs per day for 45+ years, though she claims to have quit 2 days ago. Bladder ultrasound in the emergency department reveals a large mass consistent with bladder cancer, as well as significant urinary retention. Which of the following is most likely to be detected upon imaging the patient's genitourinary system? A. Bilateral hydronephrosis B. Bladder dilation C. Bladder dyskinesis D. Unilateral hydronephrosis E. Ureteral dilation

A. Bilateral hydronephrosis Urinary Retention MC chronic manifestation --> bilateral hydronephrosis

26. A patient is admitted to the hospital with severe left lower quadrant pain. The patient is suspected of having acute diverticulitis. Which of the following diagnostic evaluations should be performed for this patient? A. CT scan with oral and IV contrast B. CT scan without contrast C. MRI D. Barium enema E. Colonoscopy

A. CT scan with oral and IV contrast

97. A patient has long-standing Crohn disease. As the disease progresses, several complications can occur. Which of the following complications is most likely to occur with this condition? A. Cholesterol gallstones B. Toxic megacolon C. Eosinophilic colitis D. Sarcoidosis E. Polymyalgia rheumatica

A. Cholesterol gallstones

374. A 52-year-old man with peptic ulcer disease has been on drug therapy for 3 months and has noticed changes in his bowel habits, increasing headaches, dizziness, skin rashes, loss of libido, and gynecomastia. Which of the following drugs is most likely responsible for these side effects? A. Cimetidine (Tagamet) B. Famotidine (Pepcid) C. Metronidazole (Flagyl) D. Omeprazole (Prilosec) E. Sucralfate (Carafate)

A. Cimetidine (Tagamet)

33. You see a patient who is being evaluated for gastrointestinal-related complaints thought to be due to Zollinger-Ellison (Z-E) syndrome. Patients with this condition will have complaints related to this syndrome that include all of the following signs or symptoms EXCEPT: A. Complaints of early satiety B. Symptoms refractory to therapy C. Ulcers being found in atypical locations D. Elevation of serum gastrin level E. Complaints of diarrhea

A. Complaints of early satiety

251. An 18-year-old college student without a significant past medical history comes to the health care provider complaining of a 2-week history of bloody diarrhea, vague intermittent abdominal pain, and a 15-pound weight loss. He denies any sick contacts, travel, or eating any foods that were "new or unusual." His temperature is 38.3ºC (100.9ºF), blood pressure 100/70 mm Hg, pulse 93/min, and respirations 13/min. Physical examination reveals a thin young man in no distress. His abdomen is mildly diffusely tender without rebound or guarding. Rectal examination is hemoccult positive. Laboratory studies show a mild iron-deficiency anemia and a mildly elevated sedimentation rate. Colonoscopy reveals continuous and extensive inflammation of the colon without skip lesions. Which of the following is associated with this patient's condition? A. Curative colectomy B. Fistula formation C. Noncaseating granulomas on biopsy D. Primary biliary cirrhosis E. Rectal sparing

A. Curative colectomy

227. A 44-year-old woman delivers a 3,120 g (6 lb 14 oz) newborn boy. Her pregnancy was normal except that she noted decreased fetal movement compared to her previous pregnancies. She declined an amniocentesis offered by her obstetrician. Physical examination of the newborn reveals an infant who has facial features suggestive of Down syndrome. The infant then has bilious vomiting. A radiograph showing the kidneys, ureters, and bladder (KUB) is performed, which shows a distended and gas-filled stomach and proximal duodenum and the absence of gas in the distal bowel. Which of the following is the most likely cause of the abdominal signs and symptoms? A. Duodenal atresia B. Hirschsprung's disease C. Malrotation D. Meconium ileus E. Pyloric stenosis

A. Duodenal atresia

A 32-year-old woman with increased frequency of urination, suprapubic pain, and dysuria for the past 3 days comes to the emergency department. She has no fever, nausea, or vomiting. A Gram stain reveals gram-negative rods. Which of the following is the most likely pathogen? A. Escherichia coli B. Neisseria gonorrhoeae C. Shigella dysenteriae D. Streptococcus pneumoniae E. Treponema pallidum

A. Escherichia coli UTI - MCC E. coli

A 25-year-old woman comes to the health care provider because of pain and burning with urination. She states that the symptoms started 2 days ago and have worsened since. She has no fever or chills and has never had these symptoms before. She has hypothyroidism for which she takes thyroid hormone replacement; otherwise she has no medical problems. Her temperature is 37° C (98.6° F). Examination is unremarkable, including a normal pelvic examination. KOH and normal saline "wet prep" is performed on her vaginal discharge and is negative. Urinalysis reveals numerous white blood cells. Which of the following is the most likely pathogen? A. Escherichia coli B. Neisseria gonorrhoeae C. Pseudomonas species D. Staphylococcus saprophyticus E. Trichomonas vaginalis

A. Escherichia coli MC UTI cause

261. Which of the following treatments is recommended for patients with peptic stricture? A. Esophageal dilation B. Fluticasone inhaler used without a spacer C. Botulism toxin injection D. Long-term proton-pump inhibitor (PPI) therapy E. Long-term H2 blockers

A. Esophageal dilation

253. A 44-year-old woman has a 10-year history of progressive dysphagia without pain. The dysphagia affects all kinds of food, without preference for solids. The food sticks in the xiphoid area and can be helped along by drinking large amounts of water and sitting up straight until the whole bolus passes into the stomach. She also describes many episodes of regurgitation of foul-smelling but undigested food, either when she leans forward or when she is asleep at night. She used to be heavier and lost weight as the disease progressed. A barium swallow shows a massively dilated proximal esophagus with a narrow, beak-like appearance in the lower sphincteric area. Which of the following is the most likely original pathophysiology that explains the development of this problem? A. Failure of the lower esophageal sphincter to relax B. Reflux of acid gastric juice into the lower esophagus C. Replacement of smooth muscle by fibrous tissue D. Spasm of the lower esophageal sphincter E. Weak or nonexistent peristaltic activity in the upper esophagus

A. Failure of the lower esophageal sphincter to relax

A 64-year-old man presents to his health care provider complaining of difficulty urinating and dribbling at the end of urination. Which of the following drugs would be most appropriate for treating this man's condition? A. Finasteride (Proscar) B. Leuprolide (Lupron) C. Mifepristone (Mifeprex, RU-486) D. Cabergoline (Dostinex) E. Tamoxifen (Novaldex)

A. Finasteride (Proscar) Pt has BPH, Proscar inhibitors conversion of testosterone --> DHT which is responsible for normal prostrate growth. (Prosper = 5-alpha reductive inhibitor)

A 7-year-old girl is brought to the emergency department by her parents with a complaint of severe polyuria and polydipsia. Laboratory examination reveals ketones in her urine. Which of the following is the most likely source of the ketones? A. Free fatty acid breakdown B. Gluconeogenesis C. Glycogenolysis D. Protein breakdown E. Triglyceride breakdown

A. Free fatty acid breakdown The patient is presenting with signs and symptoms highly suggestive of type 1 diabetes. The primary source of ketones in the urine is free fatty acid breakdown. Ketone body formation occurs as follows: insulin deficiency → activated lipolysis → increased plasma free fatty acids→ increased hepatic fatty acids → accelerated ketogenesis. In summary, as fatty acids break down, acetyl-CoA is generated. As acetyl-CoA levels increase, ketone bodies begin to form. When excessive amounts of ketone bodies are formed, the pathologic state known as ketosis can occur.

A 25-year-old woman presents with a 12-month history of palpitations, intermittent diarrhea, anxiety, and a 1-month history of bulging of both eyes. What is the most likely cause of her symptoms? A. Graves disease B. Hashimoto thyroiditis C. Multinodular toxic goiter D. Papillary carcinoma E. Subacute thyroiditis

A. Graves disease is the most common cause of hyperthyroidism in a young female and is the only condition that causes exophthalmos (bulging of both eyes) or pretibial myxedema. Graves disease is an autoimmune disorder in which a thyroid-stimulating IgG immunoglobulin (TSI) binds to the thyroid-stimulating hormone (TSH) receptors and causes an increased release of thyroid hormone. The exophthalmos is caused by lymphocytic infiltration of the extraocular muscles.

127. A patient is seen in the office complaining of pain in the epigastrium, radiating to the back. The pain is described as steady and dull but also severe. Lying down and eating meals make the pain worse. The patient is anorexic at present but did have a great deal of nausea and vomiting prior to this. Examination reveals abdominal distention along with epigastric tenderness. Physical examination reveals bilateral flank ecchymosis. What is this physical finding known as? A. Grey Turner sign B. Cullen sign C. Fox sign D. Murphy sign E. Obturator sign

A. Grey Turner sign

201. A patient is seen in the office complaining of pain in the epigastrium, radiating to the back. The pain is described as steady and dull but also severe. Lying down and eating meals make the pain worse. The patient is anorexic at present but did have a great deal of nausea and vomiting prior to this visit. Examination reveals abdominal distention along with epigastric tenderness. Physical examination reveals bilateral flank ecchymosis. What is this physical finding known as? A. Grey Turner sign B. Cullen sign C. Fox sign D. Murphy sign E. Obturator sign

A. Grey Turner sign

212. A 68-year-old woman reports that she had an operation 40 years earlier for intractable ulcer symptoms. She recalls that a large portion of her stomach was removed. Review of her prior medical records indicates that she underwent an antrectomy and a gastrojejunostomy. Which of the following complications is most likely in this scenario? A. Malabsorption B. Bleeding from the lower gastrointestinal tract C. Chronic pancreatitis D. Constipation E. Pancreatic insufficiency

A. Malabsorption

177. A 36-year-old schoolteacher comes to the clinic for a new-patient evaluation. She teaches second grade at a local school and just recently moved to the area. She has no current health complaints but reports a history of hepatitis. She apparently contracted "chronic hepatitis" from a blood transfusion received shortly after the birth of her first child 20 years ago, although she is unsure of the details of the disease. She was told she was infected, but she has always felt well and she doubts the diagnosis. Since then she has not suffered any symptomatic liver disease, and her liver function tests have always been normal. Review of symptoms fails to reveal any evidence of liver disease, and physical examination is unremarkable. A review of medical records that she has brought with her reveals the following: Hep A Ab: negative Hep B surface Ag: neg Hep B surface Ab: pos. Hep C Ab: pos. In addition to disease education and counseling, which of the following is an appropriate intervention for this patient at this time? A. Hepatitis A vaccination B. Hepatitis B vaccination C. Liver biopsy for risk stratification D. Referral for interferon/ribavirin treatment E. Right upper abdominal ultrasound

A. Hepatitis A vaccination

179. A patient has long-standing hemochromatosis with resulting cirrhosis. Which of the following is the most likely complication of the cirrhosis? A. Hepatocellular cancer B. Hepatitis A C. Hepatitis B D. Hepatitis D E. Portal hypotension

A. Hepatocellular cancer

338. Which of the following electrolyte abnormalities is most commonly seen with squamous cell carcinoma of the esophagus? A. Hypercalcemia B. Hypocalcemia C. Hyperkalemia D. Hypokalemia

A. Hypercalcemia

A 45-year-old man comes to a health care provider with complaints of weakness, fatigue, and near syncope when he stands up quickly. Screening chemistry studies demonstrate sodium 128 mEq/L (normal 135-145 mEq/L); potassium 5.2 mEq/L (normal 3.5-5.3 mEq/L); bicarbonate 17 mEq/L (normal 20-29 mEq/L); and urea nitrogen 45 mg/dL (7-20 mg/dL). The health care provider is considering Addison disease in the differential diagnosis. Which of the following features on physical examination would be most suggestive of this diagnosis? A. Hyperpigmentation in the palmar creases B. Large, furrowed tongue C. Many spider angiomas D. Protruding eyeballs E. Small glistening bumps on the lips

A. Hyperpigmentation in the palmar creases Although there is some variation in the usage of the term, Addison disease is usually taken to mean adrenocortical insufficiency related to autoimmune disease that destroys the adrenal gland. Most authors separate out secondary adrenocortical insufficiency caused by pituitary failure and recent or current exogenous steroid therapy. True Addison disease, which is not related to inadequate pituitary secretion of ACTH, frequently has stigmata of hyperpigmentation relating to a melanocyte-stimulating hormone (MSH) effect seen with high ACTH levels. The biochemical basis of this is a homology between part of the ACTH molecule and the MSH molecule.

203. Certain markers of increased mortality are seen in the setting of acute pancreatitis. Which of the following laboratory results is most closely associated with an increase in the mortality rate when they findings occur with acute pancreatitis? A. Hypoalbuminemia B. Elevated triglyceride level C. Hypocalcemia D. Hyperglycemia E. Elevated serum lipase level

A. Hypoalbuminemia

363. Which of the following electrolyte disturbances is most likely to occur in the setting of acute, prolonged vomiting? A. Hypokalemia B. Hyponatremia C. Hypocalcemia D. Hypomagnesemia E. Metabolic acidosis

A. Hypokalemia

22. A patient with gastric cancer has a palpable ovary that is thought to be secondary to metastatic disease to the ovary. What is this metastatic ovarian tumor known as? A. Krukenberg tumor B. Blumer shelf tumor C. Sister Mary Joseph nodule D. Virchow node E. Irish node

A. Krukenberg tumor

56. A 33-year-old man complains that his chest hurts when he eats, especially when he eats meat. A radiograph shows a dilated esophagus, and achalasia is suspected. Esophageal manometry is used to confirm the diagnosis. Swallowing induced relaxation is reduced at which anatomic location in this man? A. Lower esophageal sphincter (LES) B. Proximal esophagus C. Middle esophagus D. Pharynx E. Upper esophageal sphincter (UES)

A. Lower esophageal sphincter (LES)

350. A patient has had several episodes of pancreatitis due to chronic alcohol ingestion and is suspected of having chronic pancreatitis. If chronic pancreatitis exists, which of the following findings would most likely be seen? A. Normal amylase and lipase levels B. Elevated trypsinogen C. Low serum glucose D. No glucose in urine E. Elevated serum gastrin

A. Normal amylase and lipase levels

A 5-year-old boy is brought to an inner city pediatric clinic because of daytime urinary incontinence. His mother is very frustrated because he has never been dry during the day and the kids at school constantly tease him. Further history taking reveals that he has had inconsistent medical care, but fails to reveal any psychologic problems. On physical examination his blood pressure is 100/70 mm Hg. Both his weight and height are below the 5th percentile for his age. His bladder is enlarged and palpable above the symphysis pubis. The remainder of the examination is unremarkable. Which of the following is the most likely cause of his problem? A. Obstructive uropathy B. Primary polydipsia C. Reflux nephropathy D. Sickle cell trait E. Unstable bladder

A. Obstructive uropathy suggests bladder outlet obstruction, MCC of urethral obstruction is posterior urethral valves.

290. In addition to gastric acid, which of the following substances is most commonly linked with the development of peptic ulcer disease? A. Pepsin B. Acetylcholine C. Bicarbonate D. Prostaglandin E2 E. Serotonin

A. Pepsin

52. Clinicians should understand the potential adverse actions when prescribing therapy for patients with inflammatory bowel disease. Which of the following adverse effects occurs when metronidazole (Flagyl) is used in order to manage Crohn disease? A. Peripheral neuropathy B. Exfoliative dermatitis C. Peripheral edema D. Hyponatremia E. Metabolic acidosis

A. Peripheral neuropathy

6. A patient is seen in the office with a significant amount of ascites. Paracentesis is performed, and the patient is found to have an elevated serum ascites albumin gradient. Which of the following underlying choices is most likely to lead to this abnormal serum ascites albumin gradient? A. Portal hypertension B. Hypoalbuminemia C. High-salt diet D. Spontaneous bacterial peritonitis E. Acute hepatitis C infection

A. Portal hypertension

296. A postoperative patient is seen in the hospital during medicine rounds. The patient is complaining of abdominal pain. Abdominal plain film reveals a uniform distribution of gas in the small bowel, colon, and rectum. Which of the following electrolytes will need to be replaced to the greatest extent to clear this obstruction? A. Potassium B. Sodium C. Calcium D. Magnesium E. Chloride

A. Potassium

11. A patient is seen in the office complaining of pain in the epigastrium, radiating to the back. The pain is described as steady and dull but also severe. Lying down and eating meals make the pain worse. The patient is anorexic at present but did have a great deal of nausea and vomiting prior to this. This is his third episode this year. Which of the following abdominal film findings would be consistent with this diagnosis? A. Presence of sentinel loop B. Gastric air bubble displaced laterally C. Free air under the diaphragm D. Air-fluid levels of the large bowel E. Calcifications at the stomach pylorus

A. Presence of sentinel loop

167. Hepatitis A can be acquired in epidemics, such as if a patient with active hepatitis A is a food handler who does not practice good hygiene. If a patient acquires hepatitis A (HAV) following fecal-oral transmission of this virus, which of the following clinical entities will be the first to occur? A. Shedding of fecal HAV B. Jaundice C. Increased liver transaminase D. Production of IgM anti-HAV E. Production of IgG anti-HAV

A. Shedding of fecal HAV

A 20-year-old woman presents with a 2-day history of dysuria and increased urinary frequency. She states that she was recently sexually active. Physical examination reveals a temperature of 38.2° C (100.7° F) with normal vital signs. Gynecologic examination reveals no evidence of discharge, vaginitis, or cervicitis. Urinalysis reveals 14 WBC/hpf with many gram-negative rods. Which of the following is the most appropriate pharmacotherapy? A. Sulfamethoxazole/trimethoprim (Bactrim) B. Ceftriaxone (Rocephin) C. Fluconazole (Diflucon) D. Gentamicin E. Metronidazole (Flagyl)

A. Sulfamethoxazole/trimethoprim (Bactrim)

223. A patient is seen with complaints of abdominal pain radiating straight to the back. She is taking a lot of medications, which may be the precipitating cause for these symptoms. Which of the following medications is known to cause this acute condition? A. Sulfonamides B. Isoniazid (INH) C. Meloxicam (Mobic) D. Omeprazole (Prilosec) E. Ranitidine (Zantac)

A. Sulfonamides

28. Which of the following is a known risk factor for the development of hepatic adenoma? A. Use of oral contraceptives B. Use of thiazide diuretics C. Smoking D. Use of vitamins containing iron E. Male sex

A. Use of oral contraceptives

A 72-year-old diabetic woman is seen for hypertension. On physical examination, she has clear lungs and a soft abdomen. There is +1 pedal edema. Her blood pressure is 188/100 mm Hg. Her creatinine is 1.6 mg/dL (normal 0.6-1.2 mg/dL in females) and her blood urea nitrogen is 22 mg/dL (normal 7-20 mg/dL). Her urine reveals +3 protein (normal result shows no protein). Which of the following would be the most appropriate pharmacotherapy?

ACE inhibitors are the preferred agents for diabetics, as they lower both systemic blood and intraglomerular pressures. This will lessen the amount of proteinuria and sclerosis. All of the other agents will lower blood pressure but will not specifically lower intraglomerular pressure. ACE inhibitors are first-line agents used in the management of hypertension in those with diabetes and with chronic heart failure. These agents slow the progression of proteinuria as they reduce the glomerular filtration pressure, which helps to preserve nephrons in the kidney. If ACE inhibitors are not well tolerated, angiotensin-receptor blockers (ARBs) are used in patients who have diabetic nephropathy and hypertension. Tight blood glucose control in diabetic patients is also helpful in managing this microvascular complication of diabetes.

49. Hepatitis exists in various forms, each with certain associated pathophysiologic characteristics. Which of the following types of hepatitis is associated with immune-complex phenomena leading to arthritis, glomerulonephritis, and vasculitis? A. HAV B. HBV C. HCV D. HDV E. HEV

B. HBV

268. A patient is seen in the office to discuss the results of liver evaluation studies. Which of the following conditions is most likely to cause an elevated aspartate transaminase (AST) to alanine transaminase (ALT) ratio of 2:1? A. Cholestasis B. Alcoholic hepatitis C. Hepatitis C D. Hemochromatosis E. Wilson disease

B. Alcoholic hepatitis

A 45-year-old woman comes to the clinic with a 4-month history of headaches and changes in her vision. She has been previously healthy and has not been on any medications. On examination she is found to have a small field defect in both eyes. The diagnosis of a pituitary adenoma is entertained. Which of the following is the most sensitive diagnostic study for this condition?

An MRI with gadolinium is considered the most sensitive test for detecting microadenoma. The test can reveal small tumors such as microadenomas in 20% of normal women. MRI of the pituitary gland is preferred over CT scan because it provides more anatomic information, including information regarding invasion of the cavernous and sphenoidal sinuses. It is also able to better identify optic chiasm compression. CT scans of the pituitary gland should be performed in patients who have contraindications for MRI scans.

A 53-year-old woman comes to the health care provider because of a "lump" in her neck. She says that her masseuse noticed it 1 month ago. There is no associated pain, pressure, or hoarseness. She feels fine and has no other complaints. She has no history of radiation exposure. Examination reveals a palpable thyroid nodule that is approximately 3 cm. Which of the following is the most appropriate next step in diagnosis?

An ultrasound is the first step in the evaluation of a palpable thyroid nodule. An ultrasound is a noninvasive technique that can determine if the nodule is cystic or solid, the exact size of the lesion, and whether there are any additional masses.

39. Which of the following tumor markers for hepatocellular carcinoma is used for both screening and monitoring response to treatment? A. Beta human chorionic gonadotropin (HCG) B. Alpha fetoprotein (AFP) C. CA-125 D. Carcinoembryonic antigen (CEA) E. CA 19-9

B. Alpha fetoprotein (AFP)

165. Which of the following diagnostic studies is considered to be the gold standard for identifying gastroesophageal reflux disease (GERD)? A. Esophagogastroduodenoscopy (EGD) B. 24-hour pH monitoring C. Manometry D. Upper gastrointestinal (GI) series E. Capsule endoscopy

B. 24-hour pH monitoring

66. A male patient has a sudden onset of fretfulness and pain. He curls up with his legs located in a fetal position. Over the next few hours he alternates between episodes of pain/crying with tears and acting normally. The patient's mother fears something is terribly wrong and brings him to the emergency department. His past medical history is unremarkable. The previous week he had had a cold with a runny nose. Stools had been normal that day but at the hospital he has a semi-soft stool with some blood and mucus. On examination the patient is quiet. The abdomen is surprisingly soft and normal. This classic presentation of intussusception is most likely to occur in which of the following age groups? A. Birth to 4 weeks of age B. 3 to 12 months of age C. 3 to 5 years of age D. Early adolescence E. Late adolescence

B. 3 to 12 months of age

137. Patients with irritable bowel syndrome can have diarrhea-predominant, constipation-predominant, or alternating constipation and diarrhea as their symptom complex. Treatments recommended for patients who have constipation-predominant irritable bowel syndrome include all of the following EXCEPT: A. Metamucil B. Bile-acid sequestrant agents such as cholestyramine (Questran) C. Non-absorbable osmotic sugars such as lactulose D. Lubiprostone (Amitiza) E. Polyethylene glycol (MiraLax)

B. Bile-acid sequestrant agents such as cholestyramine (Questran)

299. A 24-year-old student complains of midepigastric pain that she describes as a "dull ache" that is relieved by eating. She has awakened from sleep on several occasions at 2 AM because of severe exacerbation of these symptoms, which are relieved with magnesium hydroxide. She takes frequent acetaminophen for menstrual cramping. Which of the following is the most likely cause of her symptoms? A. Autonomous gastrin secretion B. Gram-negative organism C. Gram-positive organism D. Prostaglandin inhibition E. Vagal inhibition

B. Gram-negative organism

241. A patient is seen in the office with gastrointestinal complaints. Which of the following is the main differentiating symptom between achalasia and esophageal cancer? A. Esophageal cancer alone causes weight loss. B. Achalasia causes dysphagia for solids and liquids from the onset. C. Only achalasia is associated with regurgitation. D. Only achalasia is associated with Schatzki ring. E. Only achalasia occurs with weight loss.

B. Achalasia causes dysphagia for solids and liquids from the onset.

60. After passing his physical examination, a 19-year-old army recruit gives urine and blood samples for further testing. Serum analysis yields elevated ALT, HBsAg, anti-HBc, HBeAg, and bilirubin. All other values are normal. What is the hepatitis B status of this recruit? A. Asymptomatic carrier B. Active carrier C. Fulminant hepatitis B D. Recovered from acute self-limited HBV E. Vaccinated against HBV

B. Active carrier

118. Which of the following is the most likely cell type for stomach carcinoma? A. Transitional cell B. Adenocarcinoma C. Squamous cell D. Epithelial cell

B. Adenocarcinoma

45. For patients who are diagnosed with gastric carcinoma, which of the following cell types is most likely? A. Transitional cell B. Adenocarcinoma C. Squamous cell D. Large cell E. Lymphoma

B. Adenocarcinoma

101. A 56-year-old alcoholic man gives a history of several years of constant epigastric pain, radiating straight through to the back. The pain is severe, present at all times, and exacerbated by eating. He also has steatorrhea and diabetes. He relates that he began to have episodes of acute alcoholic pancreatitis in his mid-thirties. At first these occurred every few years, but eventually they grew in number to several attacks per year. Eventually the pain became constant. Although he made numerous attempts to quit drinking, he was not successful until about 1 year ago; still, his current abstinence has not alleviated the pain. Which of the following is the most appropriate initial step in evaluation? A. Arteriogram B. CT scan of the upper abdomen C. Endoscopic retrograde cholangiopancreatogram (ERCP) D. Sonogram of the upper abdomen E. Upper gastrointestinal series with barium

B. CT scan of the upper abdomen

13. Which of the following types of cancer is most commonly associated with a palpable Blumer shelf, left supraclavicular or scalene adenopathy, and periumbilical mass? A. Esophageal cancer B. Gastric cancer C. Colorectal cancer D. Liver cancer E. Cholangiocarcinoma

B. Gastric cancer

316. A symptomatic patient has just been diagnosed with autoimmune hepatitis. Which of the following is the initial treatment of choice? A. Azathioprine (Imuran) B. Glucocorticoids C. Pegylated interferon D. Ribavirin E. Cyclosporine

B. Glucocorticoids

25. Which of the subtypes of hepatitis is a DNA rather than an RNA virus? A. HAV B. HBV C. HCV D. HDV E. HEV

B. HBV

315. Which of the following types of hepatitis is most likely to be spread via perinatal methods if no clinical intervention occurs? A. HAV B. HBV C. HCV D. HDV E. HEV

B. HBV

239. A patient with long-standing Crohn disease has recurrent flares that do not respond to medical management. The patient ultimately has an ileal resection, which results in bile salt malabsorption. Which of the following clinical manifestations is consistent with this bile salt deficiency state? A. Flushing due to serotonin loss B. Diarrhea C. Heartburn D. Melena E. Fecal incontinence

B. Diarrhea

209. A 70-year-old man comes to the emergency department complaining of abdominal pain. He describes the pain as crampy and primarily located in his left lower quadrant. He has had minimal nausea, but complains of constipation. His past medical history is significant for hypertension, hyperlipidemia, and gout. His medications include atenolol and simvastatin. He is allergic to penicillin. His temperature is 38.0º C (100.4º F), blood pressure is 140/60 mm Hg, pulse is 100/min, and respirations are 20/min. His physical examination is significant for tenderness to palpation at the left lower quadrant without rebound or guarding. His stool is negative for occult blood. His heart and lung examinations are unremarkable. Which of the following is the most likely diagnosis? A. Appendicitis B. Diverticulitis C. Diverticulosis D. Ischemic colitis E. Sigmoid volvulus

B. Diverticulitis

88. A patient with a history of chronic intake of alcohol and smoking cigarettes is seen in the office. The patient would like to have an esophagogastroduodenoscopy done because his uncle was just diagnosed with esophageal cancer. Which of the following signs or symptoms is considered to be the earliest complaint/finding in a patient with esophageal cancer? A. Dysphagia for liquids B. Dysphagia for solids C. Jaundice D. Early satiety E. Odynophagia

B. Dysphagia for solids

23. Infection with Helicobacter pylori is associated with all of the following clinical entities EXCEPT: A. Chronic gastritis B. Esophageal cancer C. Atrophic gastritis D. Gastric cancer E. Peptic ulcer disease

B. Esophageal cancer

318. A 45-year-old chronic alcoholic presented to the emergency department 5 years ago with 24 hours of epigastric pain radiating to his back, nausea, and vomiting. He gradually recovered from that acute episode. Over the next 5 years, he is repeatedly admitted for similar symptoms. He then presents with gradual onset of weight loss, midabdominal pain radiating to his back, and steatorrhea. Which of the following conditions has most likely occurred? A. Cholangiocarcinoma B. Exocrine insufficiency of the pancreas C. Gastric outlet obstruction D. Pancreatic adenocarcinoma E. Scarring of the entire length of the common bile duct

B. Exocrine insufficiency of the pancreas

A 54-year-old woman comes to the emergency department with severe left-sided flank pain that is referred to the left labia majora. She is very uncomfortable lying still on the stretcher and is continuously shifting, trying to find a comfortable position. A nonenhanced helical CT scan shows a 10-mm ureteral stone at the ureteropelvic junction. She has a normal coagulation profile. Which of the following would most likely be the best therapy in this case? A. Plenty of fluids and analgesics and await spontaneous passage B. Extracorporeal shock wave lithotripsy (ESWL) C. Endoscopic retrograde basket extraction D. Endoscopic retrograde laser vaporization of the stone E. Open surgical removal

B. Extracorporeal shock wave lithotripsy (ESWL) this is the MC method used to fragment urinary stones

240. A 9-month-old boy is brought to the health care provider's office because his weight is persistently below the tenth percentile. His mother states that the infant seems to be hungry all the time and usually consumes 8-12 oz of formula every 2-3 hours in addition to some table food. He also has frequent, bulky, and malodorous stools. He does not take any medications. A malabsorption syndrome is suspected. The results of which of the following tests will most likely be abnormal? A. Abdominal radiography B. Fecal fat quantification C. Serum albumin D. Stool culture for Clostridium difficile toxins E. Stool smear for leukocytes and eosinophils

B. Fecal fat quantification

110. A 74-year-old man comes to the emergency department after two episodes of passing large amounts of bright red blood per rectum that occurred without pain. He has a long history of chronic constipation without a recent change in his bowel habits. A flexible sigmoidoscopy reveals multiple large diverticula in the sigmoid colon with copious amounts of fresh blood in this region. No other abnormalities are seen on examination of the splenic flexure, where formed brown stool is encountered. Which of the following is the most likely underlying cause for his bleeding? A. Diffuse descending colon ischemia B. Hemorrhage from a single diverticular arteriole C. Oozing from an inferior mesenteric vein D. Thrombosis of a branch of the inferior mesenteric artery E. Thrombosis of internal hemorrhoids

B. Hemorrhage from a single diverticular arteriole

343. Which of the following viruses produces disease or sequelae that is/are more severe if the infection occurs at a very young age? A. Epstein-Barr virus B. Hepatitis B virus C. Measles virus D. Poliovirus E. Varicella zoster virus

B. Hepatitis B virus

A 5-month-old boy is brought to the office for a mass in his left groin area. The infant is on the examination table, quietly sucking on his pacifier. On examination there is a sacculated cavity that does not reach the inguinal ring. Light easily passes through the sac. Which of the following is the most likely diagnosis? A. Hematoma B. Hydrocele C. Inguinal hernia D. Testicular torsion E. Testicular tumor

B. Hydrocele fluid filled sac, not painful, transilluminates

A 31-year-old woman comes into the office because she has not had a menstrual period for 7 months. She previously had normal cycles. She states that over the past year she has felt increasingly weak and tired. She always feels cold and her hair has been thinning over the course of the year. She also complains of constipation, weight gain, and depression. Her temperature is 36.7º C (98º F), blood pressure 100/60 mm Hg, pulse 56/min, and respirations 10/min. Physical examination is significant for brittle hair and delayed deep tendon reflexes. Urine human chorionic gonadotropin (hCG) is negative. Thyroid-stimulating hormone (TSH) is 20 µU/mL (normal 0.4-4.0). Prolactin is normal. Which of the following is the most likely cause of this patient's amenorrhea? A. Hyperprolactinemia B. Hypothyroidism C. Kallmann syndrome D. Polycystic ovarian syndrome E. Pregnancy

B. Hypothyroidism Secondary amenorrhea is defined as the absence of menses for 6 cycle intervals or 12 months in a woman who previously had regular cycles. This patient, therefore, has secondary amenorrhea. She also has a constellation of signs and symptoms that are highly suggestive of hypothyroidism. Patients who have hypothyroidism often complain of some combination of weakness, fatigue, cold intolerance, constipation, weight gain, depression, or thinning of the hair. Physical examination can reveal bradycardia and low blood pressure. Laboratory evaluation often shows an elevated TSH as the pituitary attempts to stimulate the underfunctioning thyroid. Many patients who have hypothyroidism will be asymptomatic, however, and the thyroid abnormality is found by thyroid function tests. Hypothyroidism likely leads to amenorrhea through changes in GnRH production. Treatment with thyroid replacement will often return these patients to regular menses.

83. Patients with acute pancreatitis need appropriate management for improved outcomes. Which of the following clinical interventions is most important in the management of a patient with acute pancreatitis? A. IV antibiotics B. IV fluid resuscitation C. Bowel rest via nasogastric (NG) tube suctioning D. Narcotic pain medications E. IV somatostatin therapy

B. IV fluid resuscitation

369. A patient with cirrhosis has acute esophageal variceal bleeding. In addition to endoscopic therapy for these varices, which of the following medications is first-line therapy in this setting? A. IV vasopressin B. IV octreotide C. IV metoprolol D. IV diltiazem E. IV furosemide (Lasix)

B. IV octreotide

278. A patient is admitted to the hospital with tachypnea, tachycardia, and orthostasis. In a patient with previously normal renal function, which of the following laboratory results will be consistent with an upper gastrointestinal (GI) bleed? A. Increased serum creatinine B. Increased blood urea nitrogen (BUN) to creatinine ratio C. Hyponatremia D. Hypercalcemia E. Hyperamylasemia

B. Increased blood urea nitrogen (BUN) to creatinine ratio

247. A 43-year-old man with recurrent history of peptic ulcer disease associated with diarrhea and strong family history of duodenal ulcer disease is suspected of having Zollinger-Ellison syndrome (gastrinoma). Secretin (1 U/kg) is given as a rapid IV injection to test for gastrinoma. Which of the following results would support the existence of gastrinoma following secretin administration? A. Gastrin release from antrum B. Increased serum gastrin C. Inhibition of gastric emptying D. Inhibition of gastric secretion E. Stimulation of pancreatic HCO3- secretion

B. Increased serum gastrin

246. A 62-year-old man has had gastroesophageal reflux disease diagnosed by pH monitoring. This condition has been present for several years. He has been less than totally compliant with medical management, which he follows only when the pain is bad but he is not adherent with medication when he feels better. Endoscopy and biopsies show severe peptic esophagitis, with Barrett's esophagus and early dysplastic changes, but no overt carcinoma. Additional tests show good esophageal motility, with low pressure in the lower esophageal sphincter and normal gastric emptying. Which of the following is the most appropriate treatment at this time? A. Heller myotomy of the lower esophageal sphincter B. Laparoscopic Nissen fundoplication C. Transhiatal total esophagectomy D. Transthoracic resection of the lower esophagus E. Vagotomy, pyloroplasty, and fundic gastric wrap

B. Laparoscopic Nissen fundoplication

129. A 26-year-old woman comes to the health care provider complaining of epigastric pain and chest pain for 3 weeks. Symptoms are occurring one to three times per week. She describes the pain as burning, worse with eating, and nonradiating. She denies shortness of breath, nausea, vomiting, or palpitations. Review of systems is significant for a dry nocturnal cough. She smokes approximately 10 cigarettes each day and has two to three beers each night. She has not taken anything for these complaints and she denies being on any long-term medication. Physical examination reveals a moderately obese woman in no acute distress. Her heart is regular without murmurs, rubs, or gallops. Her abdomen is soft, nontender, and nondistended with normal bowel sounds. Which of the following is the most appropriate management at this time? A. H2 blockers B. Lifestyle modification C. Metoclopramide (Reglan) D. Nissen fundoplication E. Proton pump inhibitor

B. Lifestyle modification

A 54-year-old man with chronic renal failure is seen in the renal clinic. He has renal failure caused by chronic glomerular disease. His blood pressure is well controlled, and he has no complaints. He has no edema. Physical examination is generally normal. Of note, laboratory analysis reveals a calcium of 7.9 mg/dL (normal 8.5-10.2 mg/dL), albumin 3.7 g/dL (normal 3.4-5.4 g/dL), phosphate 8.2 mg/dL (normal 3-4.5 mg/dL), parathyroid hormone 512 pg/mL, (normal 10-55 pg/mL), urea nitrogen 80 mg/dL (normal 7-20 mg/dL), and creatinine 7.2 mg/dL (normal 0.8-1.4 mg/dL in males). Which of the following is the most appropriate initial step in management? A. Start oral vitamin D immediately B. Lower his phosphate with oral calcium given with meals C. Lower his phosphate with oral aluminum hydroxide binders D. Start urgent dialysis E. Urge him to have parathyroid surgery with resection of parathyroid adenomas

B. Lower his phosphate with oral calcium given with meals This patient has secondary hyperparathyroidism with a high parathyroid hormone (PTH) and a calcium x phosphate of about 64. The phosphate needs to be urgently lowered to avoid metastatic calcification. Oral calcium given with meals will bind the phosphates in the food, and lower the phosphate.

78. A football player is tackled and swallows a great deal of dirt that was kicked up into his face. The player gets up and starts retching and vomiting. He is found to have some flecks of blood in the vomitus. There is no chest pain and no subcutaneous emphysema is noted. Which of the following is the most likely diagnosis? A. Esophageal spasm B. Mallory-Weiss syndrome C. Boerhaave syndrome D. Esophageal varices E. Peptic ulcer bleeding

B. Mallory-Weiss syndrome

295. A 72-year-old woman comes to the emergency department complaining of severe left lower quadrant pain for 24 hours. She has a long history of constipation and frequently has left lower quadrant cramping after meals. Over the past 24 hours, she has had increasing discomfort in the left lower quadrant and has had a temperature to 39.4ºC (102.8ºF). On examination there is tenderness to palpation and guarding in the left lower quadrant. CT scan reveals an abscess involving and contiguous to the sigmoid colon. Which of the following is most likely causing this patient's condition? A. Bleeding at the site of the abscess B. Micro- or macroperforation of a diverticulum C. Mucosal inflammation D. Mucosal ischemia E. Pancreatitis

B. Micro- or macroperforation of a diverticulum

297. A 57-year-old alcoholic man is being treated for acute hemorrhagic pancreatitis. He was admitted to intensive care for 1 week, where he required chest tubes for pleural effusions and a ventilator for several days. Eventually he improved sufficiently to be transferred to the general medicine floor but 3 days later, and about 2 weeks after the onset of the disease, he spiked a fever and developed leukocytosis. Which of the following developments do these recent findings most likely suggest? A. Chronic pancreatitis B. Pancreatic abscess C. Pancreatic pseudocyst D. Pelvic abscess E. Subphrenic abscess

B. Pancreatic abscess

53. Advanced liver disease will result in prolongation of which of the following? A. Activated partial thromboplastin time (aPTT) B. Prothrombin time C. Thrombin time D. Bleeding time E. Platelet count

B. Prothrombin time

373. Which of the following treatments is first-line therapy for a patient with gastric complaints due to Zollinger-Ellison (Z-E) syndrome? A. Histamine-2 blockers B. Proton-pump inhibitors (PPIs) C. Sucralfate D. Misoprostol E. Sodium bicarbonate

B. Proton-pump inhibitors (PPIs)

204. Which of the following extraintestinal manifestations of inflammatory bowel disease is more commonly associated with ulcerative colitis than Crohn disease? A. Perianal fistulas B. Pyoderma gangrenosum C. Skip lesions D. Transmural intestinal involvement E. Rectal sparing

B. Pyoderma gangrenosum

A 7-day-old boy who is the product of an uncomplicated gestation is brought to the health care provider because of hypospadias. The infant is otherwise healthy and is urinating without any difficulty. On physical examination, vital signs are stable, lungs are clear, and the heart is beating at a regular rate. The only abnormal physical finding is the hypospadias. Urinalysis is negative for infection. Which of the following is the most appropriate next step? A. Measuring serum creatinine level B. Schedule a renal ultrasound C. Obtain an IV pyelogram D. Cystography E. Performing a circumcision

B. Schedule a renal ultrasound Renal U/S is a safe way of dx neonatal urinary tract pathology. Infants w/ hypospadias are prone to UTI's and other urinary tract anomalies.

182. Which of the following laboratory studies is considered the best test for identifying acute pancreatitis? A. Serum amylase B. Serum lipase C. Gamma-glutamyl transferase (GGT) D. Aspartate transaminase (AST) E. Alanine aminotransferase (ALT)

B. Serum lipase

70. Which of the following laboratory studies is the most specific for identifying acute pancreatitis? A. Serum amylase B. Serum lipase C. Gamma-glutamyl transferase D. Aspartate transaminase E. Alanine transaminase

B. Serum lipase

95. Diverticular disease can affect different portions of the colon, depending on what part of the world a person lives in. For patients living in the United States, which of the following sites is the most common location for diverticular disease to be present? A. Rectum B. Sigmoid colon C. Hepatic flexure D. Terminal ileum E. Cecum

B. Sigmoid colon

353. Which of the following is a known complication of long-standing achalasia? A. Adenocarcinoma of the esophagus B. Squamous cell carcinoma of the esophagus C. Plummer-Vinson syndrome D. Esophageal rings E. Schatzki ring

B. Squamous cell carcinoma of the esophagus

134. A patient is seen in the office complaining of pain in the epigastrium, radiating to the back. The pain is described as steady and dull but also severe. Lying down and eating meals make the pain worse. The patient is anorexic at present but did have a great deal of nausea and vomiting prior to this. This is his third episode this year. Abdominal film reveals pancreatic calcifications. Which of the following conditions would the patient also be expected to have in this scenario? A. Diabetes insipidus B. Steatorrhea C. Pyloric obstruction D. Peptic ulceration E. Biliary dyskinesia

B. Steatorrhea

202. A patient is seen in the office complaining of pain in the epigastrium, radiating to the back. The pain is described as steady and dull but also very severe in nature. Lying down and eating meals make the pain worse. The patient is anorexic at present but did have a great deal of nausea and vomiting prior to this visit. This is his third episode this year. Abdominal film reveals pancreatic calcifications. Which of the following conditions would the patient also be expected to have in this scenario? A. Diabetes insipidus B. Steatorrhea C. Pyloric obstruction D. Peptic ulceration E. Biliary dyskinesia

B. Steatorrhea

Patients with ulcerative colitis ad chrohn disease may receive asathrioprin or 6-mercaptopurine for which of the following reasons? A. To heal rectosigmoid disease B. To allow the dosage of steroids to be lowered C. To allow better penetration of metronidazole D. To prevent extension of the disease E. To lower the risk for subsequent colorectal cancer

B. To allow the dosage of steroids to be lowered

109. For patients with gastrointestinal disease, which of the following is the primary use for the Child-Pugh classification system? A. To determine prognosis in a patient with pancreatitis B. To assess the severity of liver disease C. To assess the activity of the liver to make clotting factors D. To assess the severity of sepsis E. To determine the risk for esophageal variceal bleeding

B. To assess the severity of liver disease

345. For patients with gastrointestinal disease, which of the following is the primary use for the Child-Pugh classification system? A. To determine prognosis in a patient with pancreatitis B. To assess the severity of liver disease C. To assess the activity of the liver to make clotting factors D. To assess the severity of sepsis E. To determine the risk for esophageal variceal bleeding

B. To assess the severity of liver disease

266. Cholangiocarcinoma typically occurs as a complication of which of the following conditions? A. Crohn disease B. Ulcerative colitis C. Diverticulitis D. Hepatitis E. Cirrhosis

B. Ulcerative colitis

A 74-year-old man complains of lower abdominal pain. He has become aware of a periumbilical mass during that time. His past medical history is significant for hypertension and type 2 diabetes. He had a history of diverticulitis 1 year ago. On physical examination, he is afebrile and appears mildly uncomfortable. On abdominal examination, there is a palpable mass extending from the pubic ramus to the umbilicus with a smooth contour. Laboratory results reveal a BUN of 14 mg/dL and a creatinine of 1.8 mg/dL. Which of the following is the most appropriate next step in the management of this patient? A. Digital rectal examination B. Urethral catheterization C. Abdominal sonogram D. Abdominal and pelvic CT scan E. Percutaneous nephrostomy

B. Urethral catheterization Onset and counter suggest "mass" is distended bladder. Modest increase in BUN and creating may be secondary to obstruction

19. A patient has bleeding due to acute diverticulosis. Medical management may be tried to stop this bleeding prior to the patient needing to have surgical resection. Which of the following medications can be utilized in order to curtail this bleeding prior to surgery? A. Dicyclomine (Bentyl) B. Vasopressin C. Tetracycline D. Growth hormone E. Papaverine

B. Vasopressin

140. Patients with chronic pancreatitis are at increased risk for the development of deficiency for which of the following? A. Glucose B. Vitamin B12 C. Vitamin B1 (thiamine) D. Vitamin B6 E. Vitamin C

B. Vitamin B12

A 20-year-old college student comes to the emergency department because of a sudden onset of excruciating left-sided flank pain radiating to the labia. She states that the flank pain is associated with tenesmus, rectal pain, nausea, and vomiting. She had 2 episodes of gross hematuria. She has no chronic medical conditions and has never had any similar episodes in the past. She has recently started taking "megadoses" of vitamin A, vitamin C, vitamin E, pyridoxine, and methylphenidate (her roommate's) to "help with final examinations." Her temperature is 37.8° C (100° F), blood pressure 130/80 mm Hg, pulse 80/min, and respirations 26/min. Examination shows severe left costovertebral angle tenderness. Radiograph of the kidneys, ureter, and bladder (KUB) shows a radio-opacity in the left ureter. Which of the following is the most likely explanation for these findings? A. Vitamin A excess B. Vitamin C excess C. Vitamin E excess D. Methylphenidate E. Pyridoxine excess

B. Vitamin C excess

178. Which of the following groups of patients is at highest risk for the development of autoimmune hepatitis? A. Young adult males who are using IV heroin B. Young adult females C. Elderly men D. Elderly females E. Anyone of the Jewish population

B. Young adult females

A 45-year-old man comes to a health care provider with complaints of weakness, fatigue, and feeling near fainting when he stands up quickly. Screening chemistry studies demonstrate sodium 128 mEq/L (normal 135-145 mEq/L); potassium 5.2 mEq/L (normal 3.5-5.3 mEq/L); bicarbonate 17 mEq/L (normal 20-29 mEq/L); and urea nitrogen 45 mg/dL (7-20 mg/dL). The health care provider is considering Addison disease in the differential diagnosis. Which of the following features on physical examination would be most suggestive of this diagnosis?

Black freckles on the shoulders. Although there is some variation in the usage of the term, Addison disease is usually taken to mean adrenocortical insufficiency related to autoimmune disease that destroys the adrenal gland. Most authors separate out secondary adrenocortical insufficiency caused by pituitary failure and recent or current exogenous steroid therapy. True Addison disease, which is not related to inadequate pituitary secretion of ACTH, frequently has stigmata of hyperpigmentation relating to a melanocyte-stimulating hormone (MSH) effect seen with high ACTH levels. The biochemical basis of this is a homology between part of the ACTH molecule and the MSH molecule. Typical hyperpigmentation features include black freckles of the shoulders, head, and neck or other areas of sun exposure, bluish-black discoloration of areolas and mucous membranes (both oral and anogenital), and diffuse tanning, specifically including non-sun-exposed skin. The pattern of laboratory screening studies illustrated in the question stem is also very suggestive, with very low serum sodium, high potassium, low bicarbonate, and high serum urea nitrogen.

A 14-year-old boy is evaluated for short stature. He has no significant past medical history and is considered otherwise healthy by his parents. He eats a normal diet and has regular meals. His height and weight have been consistently at the fifth percentile since early childhood. His physical examination is normal, with genitalia at Tanner stage 3. Which of the following is the most likely laboratory finding for this boy?

Bone age that is equivalent to chronologic age. This boy most likely has familial short stature (FSS). Children with FSS usually have a normal birth weight and length. At the age of 2-3 years, however, their growth begins to decelerate and drops to around the fifth percentile. The onset and progression of puberty in children with FSS are normal. Bone age is typically consistent with the chronologic age. Short stature is defined as a standing height more than two standard deviations (SDs) below the mean. Skeletal maturation is typically determined by the bone age, which is assessed using anteroposterior radiography of the left hand and wrist. Measurements that fall below the third percentile for height are termed short stature.

254. Which of the following contributes to pancytopenia that is seen in a patient with long-standing cirrhosis of the liver? A. Blood loss anemia B. Folate deficiency C. Vitamin B12 deficiency D. Bone marrow failure E. Hypersplenism

E. Hypersplenism

111. A young male teenager is seen in the office with recurrent bouts of abdominal pain. The patient appears ill. Barium enema reveals a cobblestone pattern due to submucosal thickening of the bowels, along with transmural inflammation, granuloma formation, fissures, and fistulas. Which of the following conditions is most likely affecting this patient? A. Chronic pancreatitis B. Eosinophilic colitis C. Crohn disease D. Ulcerative colitis (UC) E. Arteriovenous malformations

C. Crohn disease

15. Patients with signs and symptoms of cholecystitis can have obstruction at multiple points in the gastrointestinal (GI) tract. Which of the following signs or symptoms in a patient with known cholelithiasis would make a health care provider suspect common duct stones? A. Inability to tolerate oral intake B. Weight loss C. Jaundice D. Persistent nausea and vomiting E. Radiation of abdominal pain into the scapula

C. Jaundice

40. A patient with known hepatitis B (HBV) infection who did not receive any prenatal care delivers an infant in the hospital. The mother had not gotten any kind of treatment for HBV to date. Which of the following interventions is recommended in this setting? A. Administer the hepatitis B vaccine to the baby within 12 hours of birth. B. Administer hepatitis B immunoglobulin within 12 hours of birth. C. Administer hepatitis B immunoglobulin immediately after birth and the hepatitis B vaccination within 12 hours of birth. D. Administer the combined hepatitis A and hepatitis B vaccination within 12 hours of birth. E. No vaccination is indicated because hepatitis B is infrequently past perinatally.

C. Administer hepatitis B immunoglobulin immediately after birth and the hepatitis B vaccination within 12 hours of birth

265. A patient is hospitalized because of acute diverticulitis. The patient does not clinically respond to ciprofloxacin and metronidazole (Flagyl) antibiotic therapy and is suspected of having enterococcal infection as the cause of the acute diverticulitis. Which of the following antibiotics should be added to this treatment? A. Gentamycin B. Trimethoprim-sulfamethoxazole (Bactrim) C. Ampicillin D. Penicillin G E. Azithromycin (Zithromax)

C. Ampicillin

42. Outpatient treatment of acute diverticulitis may consist of which of the following single antibiotic regimens? A. Ciprofloxacin B. Trimethoprim-sulfamethoxazole (Bactrim) C. Ampicillin/clavulanate (Augmentin) D. Metronidazole (Flagyl) E. Tetracycline

C. Ampicillin/clavulanate (Augmentin)

A 45-year-old woman comes to the clinic with a 4-month history of headaches and changes in her vision. She has been previously healthy and has not been on any medications. On examination she is found to have a small field defect in both eyes. The diagnosis of a pituitary adenoma is entertained. Which of the following is the most sensitive diagnostic study for this condition? A. Computerized tomography (CT) scan B. Insulin-tolerance test C. Magnetic resonance imaging (MRI) D. Serum prolactin measurement E. Visual field examination

C. An MRI with gadolinium is considered the most sensitive test for detecting microadenoma. The test can reveal small tumors such as microadenomas in 20% of normal women.

A 45-year-old man comes to the emergency department with a 3-day history of hemoptysis. He has had a cough and has also been complaining of sinus congestion and drainage. Furthermore, he has noted blood in his urine. He has no other medical problems and takes no medications. A lung biopsy specimen shows necrotizing inflammation with alveolitis. Urine sediment shows red blood cell casts and dysmorphic red blood cells. Wegener granulomatosis is suspected, and steroids are started. The presence of which of the following entities will confirm the diagnosis? A. Elevated angiotensin-converting enzyme (ACE) B. Antiglomerular basement membrane antibodies (anti-GBM) C. Antineutrophilic cytoplasmic antibodies (ANCA) D. Eosinophilia E. X-bodies

C. Antineutrophilic cytoplasmic antibodies (ANCA) Wegner's classic triad *upper & lower respiratory involvement, & renal Failure* (GN); supported by + P-ANCA. Note that Sarcoidosis may have UR involvement & incr. ACE

282. A patient is seen in the office with progressive jaundice, anorexia, abdominal pain, anorexia, fever, and fatigue. Anti-liver/kidney microsomal (anti-LKM) antibodies are found in the serum. Which of the following is the most likely diagnosis? A. Wilson disease B. Alpha-1 anti-trypsin deficiency C. Autoimmune hepatitis D. Primary biliary cholangitis E. Primary sclerosing cholangitis

C. Autoimmune hepatitis

309. A patient is seen in the office with progressive jaundice, anorexia, abdominal pain, anorexia, fever, and fatigue. Anti-smooth muscle antibodies are found in the serum. Which of the following is the most likely diagnosis? A. Wilson disease B. Alpha-1 anti-trypsin deficiency C. Autoimmune hepatitis D. Primary biliary cholangitis E. Primary sclerosing cholangitis

C. Autoimmune hepatitis

317. In addition to Mallory-Weiss tears of the gastroesophageal mucosa, which of the other complications can occur in the setting of severe vomiting? A. Pyloric stenosis B. Achalasia C. Boerhaave syndrome D. Carcinoid syndrome E. Pernicious anemia

C. Boerhaave syndrome

334. A patient with long-standing Crohn disease presents to the office with right-sided flank pain and hematuria. Which of the following is most likely responsible for this presentation? A. Cysteine stones B. Struvite stones C. Calcium oxalate stones D. Uric acid stones E. Acute pyelonephritis

C. Calcium oxalate stones

325. Spider telangiectasia, palmar erythema, jaundice, scleral icterus, parotid and lacrimal gland enlargement, clubbing, Dupuytren contracture, gynecomastia, testicular atrophy, hepatosplenomegaly, and ascites are clinical manifestations for which of the following conditions? A. Hepatocellular cancer B. Autoimmune hepatitis C. Cirrhosis D. Wilson disease E. Hepatorenal syndrome

C. Cirrhosis

273. In the setting of acute pancreatitis, which of the following evaluation strategies has the benefit of both confirming the diagnosis and identifying potential complications of this condition? A. Plain film of the abdomen B. Ultrasound of the abdomen C. Computed tomography (CT) of the abdomen D. Magnetic resonance angiography (MRA) E. Positron emission (PET) scan of the abdomen

C. Computed tomography (CT) of the abdomen

210. A patient is admitted to the hospital with nausea and vomiting. The patient's abdominal film reveals dilated loops of small bowel along with air-fluid levels. The patient is unable to hold anything down. All of the following will be clinical features with this case EXCEPT: A. Dehydration B. Metabolic alkalosis C. Hypokalemia D. Hypochloremia E. Hyponatremia

E. Hyponatremia

A 70-year-old man comes to his health care provider because of an episode of apparently bloody urine. He denies prior episodes and has been previously healthy. He is not on any medications. In the office, a urinalysis confirms gross hematuria without proteinuria or casts. The patient denies any pain. The physical examination is normal. A repeat urinalysis shows similar findings. Which of the following is the most appropriate next step? A. Pelvic CT scan B. Trimethoprim-sulfamethoxazole C. Cystoscopy D. Renal angiogram E. Transrectal prostate biopsy

C. Cystoscopy Best way to narrow ddx of hematuria is direct visualization of the bladder and urethra via cystoscopy. noninvasive and fast.

37. Which of the following is the treatment of choice for a patient with liver disease caused by Wilson disease? A. Cisplatin B. High-copper diet C. D-penicillamine D. Dialysis E. Plasma exchange

C. D-penicillamine

308. A patient in the hospital undergoes surgery. The patient has an x-ray performed because of abdominal pain, and the radiologist suspects Ogilvie syndrome. Which of the following findings is present on this patient's abdominal film? A. Air-fluid levels in the small bowel B. Thumbprinting on the intestinal wall C. Dilation of the large bowel without other findings of mechanical obstruction D. Sentinel loop sign E. Air within the small bowel and no air in the large bowel

C. Dilation of the large bowel without other findings of mechanical obstruction

51. Which of the following attributes is most compatible with gastric ulceration rather than duodenal ulceration? A. Association with Helicobacter pylori infection B. Associated with type O blood C. Eating worsens pain D. Not associated with smoking E. Occurs in younger rather than older population

C. Eating worsens pain

108. Which of the following laboratory study results is consistent with nonalcoholic steatohepatitis? A. Elevated prothrombin time B. Elevated activated partial thromboplastin time (aPTT) C. Elevated aspartate transaminase (AST) and alanine transaminase (ALT) D. Low level of bilirubin E. Low level of ceruloplasmin

C. Elevated aspartate transaminase (AST) and alanine transaminase (ALT)

153. A 48-year-old woman with a long history of hepatitis C is admitted for hematemesis. She is stabilized in the emergency department, and an upper endoscopy reveals bleeding esophageal varices. On examination, she is also noted to have a moderate amount of ascites. Which of the following is the most appropriate next step in the management of her bleeding? A. Alpha interferon B. Vasopressin C. Endoscopic variceal banding D. TIPS (transjugular intrahepatic portosystemic shunt) E. Liver transplant

C. Endoscopic variceal banding

327. Which of the following treatments is the initial intervention performed in a patient who develops upper gastrointestinal bleeding as a result of esophageal variceal bleeding? A. Infusion of somatostatin B. Infusion of octreotide C. Endoscopic variceal ligation D. Insertion of Sengstaken-Blakemore tube E. Transjugular intrahepatic portosystemic shunt (TIPS)

C. Endoscopic variceal ligation

341. A 32-year-old woman has had a 15-year history of heartburn. Over the past 4 months, she has had difficulty swallowing large bites of solid food. She has no difficulty with soft foods or liquids, and she has not lost weight. Which of the following is the most likely explanation for her symptoms? A. Adenocarcinoma in the lower third of the esophagus B. Barrett's esophagus in the distal esophagus C. Fibrosis and narrowing at the distal esophagus D. Schatzki ring in the distal esophagus E. Squamous carcinoma in the mid-third of the esophagus

C. Fibrosis and narrowing at the distal esophagus

362. A patient is seen in the office complaining of pain in the epigastrium, radiating to the back. The pain is described as steady and dull but also severe. Lying down and eating meals make the pain worse. The patient is anorexic at present but did have a great deal of nausea and vomiting prior to this. Examination reveals abdominal distention along with epigastric tenderness. Ecchymosis of the inguinal ligament is also noted. What is this physical finding known as? A. Grey Turner sign B. Cullen sign C. Fox sign D. Murphy sign E. Obturator sign

C. Fox sign

191. Patients who are diagnosed with hepatitis have many questions about whether they will recover or become chronically infected. Which of the following subtypes of hepatitis is most likely to progress to chronic disease if not treated? A. HAV B. HBV C. HCV D. HDV E. HEV

C. HCV

237. Which of the following types of hepatitis virus is most likely to be spread during transfusions? A. HAV B. HBV C. HCV D. HDV E. HEV

C. HCV

48. Which of the following types of hepatitis is least likely to cause fulminant hepatitis with massive hepatic necrosis and impairment of consciousness? A. HAV B. HBV C. HCV D. HDV E. HEV

C. HCV

27. Which type of hepatitis infection of the liver is most likely to cause cirrhosis? A. Hepatitis A B. Hepatitis B C. Hepatitis C D. Hepatitis D E. Hepatitis E

C. Hepatitis C

256. Patients with cirrhosis may go on to develop ascites. In addition to increased portal pressures, which of the following conditions provides the other explanation as to why ascites develops in this situation? A. Lack of clotting factors produced by the liver B. Overhydrating the patient C. Hypoalbuminemia D. High-salt diet E. Continued intake of alcohol

C. Hypoalbuminemia

A 67-year-old woman comes to her health care provider because of pain with urination and frequent urination. She has hypertension for which she takes a beta-blocker, but no other medical problems. She states that she is not sexually active. She does not smoke and drinks cranberry juice daily. Examination shows mild suprapubic tenderness and genital atrophy but is otherwise unremarkable. Urinalysis shows 50-100 leukocytes/hpf and 5-10 erythrocytes/hpf. Which of the following is the most likely cause of infection? A. Cardiac disease B. Cranberry juice ingestion C. Hypoestrogenism D. Nephrolithiasis E. Sexual intercourse

C. Hypoestrogenism Pt has UTI. post-menopausal women t increased risk due to decreases estrogen.

255. Which portion of the gastrointestinal tract is most commonly involved with Crohn disease? A. Duodenum B. Jejunum C. Ileum D. Ascending colon E. Sigmoid colon

C. Ileum

40-year-old man complains of loss of sensation over the right scrotum and on the medial right thigh. Damage to which of the following nerves would result in such symptoms? A. Genitofemoral B. Iliohypogastric C. Ilioinguinal D. Lateral cutaneous E. Pudendal

C. Ilioinguinal The ilioinguinal nerve supplies the skin of the scrotum and the medial thigh with sensory fibers

136. A patient is seen in the office for follow-up of Plummer-Vinson syndrome. In addition to esophageal dilation, which of the following is recommended for additional therapy? A. Antacids B. Calcium and vitamin D supplementation C. Iron replacement D. Heller myotomy E. Calcium-channel blockers

C. Iron replacement

275. A 36-year-old woman without significant past medical history is complaining of 8 weeks of intermittent diarrhea. She reports the diarrhea is nonbloody and without mucus. There is mild abdominal cramping but no severe pain. There is no associated constipation, and symptoms are not worse during stressful periods. The diarrhea occurs shortly after meals and improves with fasting. She denies any weight loss. She denies sick contacts, travel history, camping history, or eating at any unusual locations. She is unsure if there is an association with milk products. Her temperature is 36.7ºC (98ºF), blood pressure is 100/70 mm Hg, pulse is 73/min, and respirations are 13/min. Her abdomen is soft, nontender, and nondistended with normal bowel sounds. Which of the following is the most likely etiology of her diarrhea? A. Enteroinvasive E. coli B. Irritable bowel syndrome C. Lactose intolerance D. Pseudomembranous colitis E. Zollinger-Ellison syndrome

C. Lactose intolerance

358. Liver damage and liver disease are common entities seen in primary care practices. All of the following are known complications for liver failure EXCEPT: A. Portal hypertension B. Low albumin C. Low estrogen D. Low blood glucose E. Elevated bilirubin levels

C. Low estrogen

31. Which of the following clinical interventions should be performed if a patient is suspected of having an esophageal motility disorder? A. Esophagogastroduodenoscopy (EGD) B. 24-hour pH monitoring C. Manometry D. Upper gastrointestinal (GI) series E. Capsule endoscopy

C. Manometry

14. A 28-year-old woman has significantly symptomatic peptic ulcer disease. Extensive medical management, including eradication of Helicobacter pylori, fails to heal her ulcers. Endoscopy shows several duodenal ulcers in the first and second portions of the duodenum. She also complains of watery diarrhea. Which of the following is the most appropriate next step in management? A. Biopsy of the duodenal ulcers B. Culture of the watery stools C. Measurement of serum gastrin D. Repetition of the H. pylori eradication every 2 months E. Replenishment of the normal gut flora

C. Measurement of serum gastrin

184. A male homosexual is seen in the office with complaints of fever, right upper quadrant pain, nausea/vomiting, hepatomegaly, and diarrhea. Stool antigen test for Entamoeba histolytica is positive. What is the treatment of choice? A. Liver transplantation B. Clindamycin C. Metronidazole D. Ciprofloxacin E. Albendazole (Albenza)

C. Metronidazole

367. A patient with long-standing osteoarthritis is seen in the office with complaints related to his stomach. Only nonsteroidal anti-inflammatory agents (NSAIDs) provide him pain control, and he is not willing to stop these agents. Which of the following therapeutic regimens is specifically aimed at protecting patients who are faced with this scenario? A. Proton-pump inhibitors (PPIs) B. Antacids C. Misoprostol (Cytotec) D. Sucralfate (Carafate) E. Sodium bicarbonate

C. Misoprostol (Cytotec)

A 65-year-old woman with an 8-year history of type 2 diabetes dies in the hospital. She had no other significant medical history. Which of the following was the most likely cause of death? A. Diabetic ketoacidosis B. Infection C. Myocardial infarction D. Renal failure E. Stroke

C. Myocardial infarction is the leading cause of death in diabetics. The advanced glycosylated products associated with long-standing diabetes mellitus accelerate the atherosclerotic process. Other risk factors for coronary artery disease (CAD) include hypertension, smoking, hypercholesterolemia, family history of CAD at a young age, male sex, and being a postmenopausal female.

In clinical trials, an experimental drug is found to cause impotence in a large percentage of male patients. Inhibition of which of the following could be responsible for this side effect? A. Conversion of DHT to testosterone B. Forward motility factor C. Nitric oxide synthase (NOS) D. Oxytocin E. Prostaglandins

C. Nitric oxide synthase (NOS) The neurons involved in mediating an erection release NO; NOS is the enzyme required for the formation of NO from. Inhibition of this enzyme --> impotence as a drug SE.

92. Which of the following medications is indicated for lowering portal venous pressures in a patient who has portal hypertension due to liver failure? A. Calcium-channel blockers B. Cardioselective beta-blockers C. Nonselective beta-blockers D. Spironolactone (Aldactone) E. Loop diuretics

C. Nonselective beta-blockers

313. Which of the following is the biggest risk for the development of hepatocellular adenomas? A. Chronic ingestion of significantly hot tea B. Chronic hepatitis C infection C. Oral contraceptives D. Fulminate hepatitis A infection E. Non-alcoholic steatohepatitis (NASH)

C. Oral contraceptives

277. Patients with chronic pancreatitis are given H2 blockers along with which other medication as part of their treatment regimen? A. Proton-pump inhibitors (PPIs) B. Antibiotics C. Pancreatic enzymes D. Glucose tablets E. Magnesium

C. Pancreatic enzymes

10. The Child-Pugh classification system used for classifying the severity of cirrhosis includes calculation for all of the following factors EXCEPT: A. Serum bilirubin B. Serum albumin level C. Partial thromboplastin time D. Presence or absence of ascites E. Presence of absence of hepatic encephalopathy

C. Partial thromboplastin time

158. A patient with long-standing alcoholic hepatitis has progressive decline in his health. The patient's MELD (Model for End-State Liver Disease) score is elevated at greater than 20, revealing severe alcoholic hepatitis. Which of the following medications improves survival by decreasing the occurrence of hepatorenal syndrome? A. Clopidogrel (Plavix) B. Lactulose C. Pentoxifylline (Trental) D. Trimethoprim-sulfamethoxazole (Bactrim) E. Pancreatic enzyme replacement

C. Pentoxifylline (Trental)

77. A patient with long-standing alcoholic hepatitis has progressive decline in his health. The patient's MELD (Model for End-State Liver Disease) score is elevated at greater than 20, revealing severe alcoholic hepatitis. Which of the following medications should be given in this scenario? A. Clopidogrel (Plavix) B. Lactulose C. Prednisone D. Trimethoprim-sulfamethoxazole (Bactrim) E. Pancreatic enzyme replacement

C. Prednisone

173. Which of the following diagnostic studies is most likely to be positive in the setting of ulcerative colitis? A. Antibodies to Saccharomyces cerevisiae B. Presence of P-antineutrophil cytoplasmic antibodies (P-ANCA) C. Presence of C-antineutrophil cytoplasmic antibodies (C-ANCA) D. Presence of plasma cell antibodies E. Elevation of serum gastrin levels

C. Presence of C-antineutrophil cytoplasmic antibodies (C-ANCA)

106. A patient with acute hepatitis B comes to the office complaining of severe fatigue, low-grade fevers, and weight loss. He was diagnosed with hepatitis B two weeks earlier, when hepatitis B surface antigen was positive. Physical examination reveals jaundice with diffuse skin excoriations. Liver and spleen are both markedly enlarged and tender. Peripheral edema is present. Which of the following would be the worst prognostic sign? A. Albumin of 3.1 g/dL (normal 3.5-5.5 g/dL) B. Bilirubin of 9.4 mg/dL (normal <1.9 mg/dL) C. Prothrombin time of 19 (normal 11-15) seconds with an INR of 2.1 D. SGOT (AST) of 2 U/L (normal 7-40 U/L) E. SGOT (AST) of 1,200 U/L (normal 7-40 U/L)

C. Prothrombin time of 19 (normal 11-15) seconds with an INR of 2.1

222. A 7-year-old boy passes a large, bloody bowel movement. He is hemodynamically stable, with hemoglobin 14 g/dL (normal 13.8-17.0 g/dL). Nasogastric aspiration yields clear, greenish fluid. Physical examination, including anoscopy, is unremarkable. Which of the following is the most appropriate next diagnostic test? A. Celiac arteriogram B. Colonoscopy C. Radioactively labeled technetium scan D. Radioactively tagged red blood cell study E. Upper gastrointestinal endoscopy

C. Radioactively labeled technetium scan

293. Which of the following tests for Helicobacter pylori infection is useful to perform for patients who have had exposure to H. pylori but is not useful as a test for cure demonstrating eradication of this organism? A. Rapid urease test B. Histology with staining C. Serology testing D. Urea breath testing E. Stool antigen testing

C. Serology testing

235. An 8-year-old girl is brought to the office because she has been complaining of frequent abdominal pain. The episodes occur every several days, show no particular pattern, and resolve without treatment within 10-15 minutes. During the episodes, the child has to lie down and bring her legs up to her chest for relief. The parents state that she started having this problem approximately a year ago, and they believed it was related to her diet. After trying to restrict various kinds of food with no effect, they decided to have her examined. Her bowel movements are regular. She has no significant past medical history and takes no medication on a regular basis. She has an older brother who is in excellent health. The mother tells you that she loves school and is a very good student. She is very popular among her friends and has no problems in social relationships. Of interest, they immigrated to the United States 3 years earlier and have moved 3 times since then for work. She did not seem to have any trouble adjusting to that situation. They will be moving to another city again in 6 months. On physical examination the girl is in no acute distress. She is in the 90th percentile for height and weight. Vital signs are within normal limits, as is a complete blood count with white blood cell differential. Her abdomen is not distended and is soft and nontender to palpation. Plain abdominal films show no abnormalities. Which of the following descriptions/interventions should be pursued? A. A computerized tomography scan of the abdomen is indicated B. A trial of oral antibiotics is warranted C. She is likely to continue having episodes of abdominal pain even as an adult D. She may have Hirschsprung's disease and a biopsy of the colon is indicated E. A barium enema is indicated at this time

C. She is likely to continue having episodes of abdominal pain even as an adult

9. A patient with gastric cancer has metastatic disease signified by a palpable lymph node in the periumbilical area. What is this palpable lymph node known as? A. Krukenberg tumor B. Blumer shelf tumor C. Sister Mary Joseph nodule D. Virchow node E. Irish node

C. Sister Mary Joseph nodule

302. Which of the following features is known to be protective against the development of ulcerative colitis? A. Oral contraceptive use B. Jewish population C. Smoking cigarettes D. Two alcoholic drinks per day E. Female population

C. Smoking cigarettes

324. A 65-year-old patient with long-standing alcoholism presents to the office with increasing amounts of ascetic fluid. In addition to furosemide (Lasix), which of the following medications is recommended for treatment of ascites? A. Chlorthalidone B. Hydrochlorothiazide C. Spironolactone D. Indapamide (Lozol) E. Chlorothiazide (Diuril)

C. Spironolactone

163. A patient with long-standing alcohol-related cirrhosis and ascites is admitted to the hospital with a high fever and mental status change. Examination reveals abdominal pain, vomiting, and fever. Which of the following conditions is most likely? A. Hepatic encephalopathy B. Hepatorenal syndrome C. Spontaneous bacterial peritonitis D. Decompensated cirrhosis E. Portal hypertension

C. Spontaneous bacterial peritonitis

43. A patient presents to the emergency department with acute abdominal pain. Which of the following descriptions of the patient's pain would be consistent with a diagnosis of diverticulitis? A. Burning substernal pain after meals B. Severe, diffuse ache in the periumbilical region C. Steady ache in the left lower quadrant with referral to the back D. Steady, boring epigastric pain with referral to the back E. Sudden, severe pain in the lower quadrant with referral to the flank

C. Steady ache in the left lower quadrant with referral to the back

291. A patient was diagnosed with esophageal cancer, and staging is going to be done prior to the esophagectomy. Which of the following clinical interventions is most commonly used in order to judge the depth of the tumor for performing staging of this localized cancer? A. CT of the chest B. MRI of the chest C. Transesophageal ultrasound D. Plain chest film E. Positron-emission topography (PET) scan

C. Transesophageal ultrasound

A 59-year-old African American man has a hard, discrete, 1.5-cm nodule that is felt in his prostate during a routine physical examination. He is completely asymptomatic, and his prostate specific antigen (PSA) 3 months ago was normal for his age. His last rectal examination was performed a year earlier and was unremarkable. He denies any family history of prostate cancer. Which of the following will best establish the diagnosis? A. Clinical follow-up during the ensuing year B. Repeat determination of PSA C. Transrectal needle biopsy of the mass D. Transrectal ultrasound of the prostate E. Transurethral resection of the prostate (TURP)

C. Transrectal needle biopsy of the mass Recent normal PSA does not exclude need to bx a hard nodule felt on prostate.

170. Which of the following is a known risk factor for the development of primary sclerosing cholangitis? A. Infection with Escherichia coli B. Porcelain gallbladder C. Ulcerative colitis D. Crohn disease E. Diverticulitis

C. Ulcerative colitis

243. A 50-year-old patient is seen in the office with complaints of 4 weeks of heartburn, increased gas, eructation, and bloating. The patient denies any hematochezia, weight loss, dysphagia, or melena. Which of the following is the test of choice to evaluate this patient? A. Urease breath test B. CLO test C. Upper endoscopy D. Colonoscopy E. Push procedure

C. Upper endoscopy

A 27-year-old woman comes to the clinic because of a 3-month history of amenorrhea, decreased libido, and fatigue. She is sexually active but does not believe she is pregnant because she always uses birth control. She denies visual changes or headache. Her temperature is 37.0ºC (98.6ºF), blood pressure 100/70 mm Hg, pulse 73/min, and respirations 13/min. Physical examination reveals a thin woman in no acute distress. The pupils are reactive and visual fields are full. She has galactorrhea. The remainder of the physical examination is normal. A urine pregnancy test is negative. Additional laboratory studies confirm the diagnosis. Which of the following is the most appropriate next step in treatment?

Carbegoline (Dostinex). This patient has a prolactin-secreting pituitary adenoma. The combination of amenorrhea and galactorrhea, along with a negative pregnancy test, strongly suggest the diagnosis. Additional laboratory results would have revealed a mild to moderately high prolactin level. Women typically have microadenomas, whereas men have macroadenomas. Dopamine agonists such as carbegoline (Dostinex) or bromocriptine (Parlodel) will reduce prolactin levels in almost all patients. Carbegoline is preferred as initial therapy over bromocriptine because it has fewer side effects as long as the patient does not intend to become pregnant. Radiation therapy and surgical resection are reserved for those patients with tumors unresponsive to appropriate medical therapy or with significant neurologic compromise.

A patient undergoes esophagogastroduodenoscopy, and squamous cell carcinoma of the esophagus is found. All of the following are known risk factors for the development of this condition EXCEPT: A. Tobacco use B. Alcohol use C. Achalasia D. Lye ingestion E. Chronic indigestion/reflux

Clinical Pearls In the past, esophageal cancer was primarily squamous cell cancer with the primary risk factors being use of alcohol and tobacco. With the increase in the rate of abdominal obesity, more patients are developing esophageal reflux with an increase in the number of adenocarcinoma cases. Barrett's esophagus, with the change in the lower esophagus from squamous to columnar cells, puts a patient at risk for the development of adenocarcinoma when these columnar cells undergo dysplasia and then overt cancer. Barrett's esophagus occurs as a result of these cells in the distal esophagus undergoing changes when they are exposed to gastric acid from reflux. Patients who are colonized with H. pylori infection who develop GERD are less likely to have Barrett's esophagus as the materials refluxing not being as acid due to the H. pylori colonization.

A 53-year-old woman comes to the health care provider complaining of fatigue over the past 6 months. During this time, she has also developed pruritus and lost 4 pounds. She is not sexually active. On physical examination she is afebrile and has mildly icteric sclera. There are excoriations noted on all 4 extremities and trunk and back. The liver edge is smooth and non-tender and measures 9 cm at the midclavicular line. There is no ascites, splenomegaly, or peripheral edema. Laboratory results reveal a normal complete blood count, normal electrolytes, liver function tests revealing an alkaline phosphatase of 260 U/L (normal <110 U/L), total bilirubin of 3.1 mg/dL (normal <1.9 mg/dL), and normal transaminase levels. Which of the following is the most likely diagnosis? A. Acute cholecystitis B. Acute hepatitis A infection C. Bacterial cholangitis D. Primary biliary cirrhosis E. Primary sclerosing cholangitis

Clinical Pearls Primary biliary cirrhosis most commonly occurs in women over age 45. It is a progressive disease that classically presents with pruritus and fatigue. The pruritus occurs as a result of a toxic accumulation of bile salts retained in the liver, which also leads to progressive liver damage and cirrhosis. Autoantibodies in the form of antimitochondrial antibodies are seen in this autoimmune disease. Treatment is bile salt therapy (ursodiol or ursodeoxycholic acid), which can slow or stop the progression to advanced liver disease, and cholestyramine, which controls the itching. End-stage liver disease can be treated only with liver transplantation.

Another term for congenital hypothyroidism?

Cretinism

A 65-year-old man develops truncal obesity and a buffalo hump. He takes no medications. Early morning cortisol is 35 mcg/dL (normal .025-0.6 mcg/dL). Evening salivary cortisol is 4 mcg/dL (normal 0.01-0.09 mcg/dL). Cortisol production does not suppress with either low or high dexamethasone. CT scan of the head demonstrates a normal sella turcica and pituitary gland. An ectopic source of ACTH production is suspected. A malignancy of which of the following organs is most likely to be the source of the ACTH?

Cushing syndrome is a cluster of clinical abnormalities seen in the setting of chronic exposure to high cortisol levels. Truncal obesity, buffalo hump, increased susceptibility to infection, hypertension, and glucose intolerance are common manifestations. Cushing syndrome can be caused by administration of exogenous corticosteroids, adrenal hyperplasia or tumor, pituitary adenoma, or ectopic ACTH production by another malignancy. The latter is most commonly a small cell carcinoma of the lung. Lung cancers are also notorious for a wide variety of other paraneoplastic syndromes, including hypercalcemia (especially with squamous cell carcinoma), hypophosphatemia (because of the influence of parathyroid-related hormone production), somatostatinoma syndrome (with prominent GI complaints), syndrome of inappropriate antidiuretic hormone secretion (SIADH), polymyositis, cerebellar degeneration, peripheral neuropathy, osteoarthropathy, migratory thrombophlebitis, and disseminated intravascular coagulation.

A 14-year-old boy slides down a banister and crashes into a large ornamental knob at its base, thereby injuring his scrotal contents. He presents in the emergency department with acute testicular pain and a scrotal hematoma the size of a grapefruit. He is able to void normally, and his urine does not contain blood. Rectal examination is unremarkable. Findings from which of the following tests will most likely determine further therapy? A. Aspiration of scrotal contents B. Retrograde cystogram C. Retrograde urethrogram D. Scrotal ultrasound E. Scrotal surgical exploration

D. Scrotal ultrasound clinical findings do not suggest injury but testicular fx would require surgical tx so do u/s to r/o

194. Primary sclerosing cholangitis and cholangiocarcinoma are known complications that occur as a long-term consequence for which of the following gastrointestinal conditions? A. Chronic pancreatitis B. Eosinophilic colitis C. Crohn disease D. Ulcerative colitis (UC) E. Arteriovenous malformations

D. Ulcerative colitis (UC)

226. A patient presents with recurrent bouts of bloody diarrhea and abdominal pain. The patient is 20 years old and has been losing weight unintentionally. She also complains of tenesmus. Barium enema reveals loss of haustrations in the colon, mucosal irregularities, and ulcerations. Which of the following is the most likely diagnosis? A. Chronic pancreatitis B. Eosinophilic colitis C. Crohn disease D. Ulcerative colitis (UC) E. Arteriovenous malformations

D. Ulcerative colitis (UC)

A 57-year-old man comes to the healthcare provider because of "erectile problems." He says that he has not been able to have an erection in months. He has seen so many television commercials lately that he expects a cure in the form of a "little blue pill." He is married, has 3 children, works as a trader on the stock exchange, and is an avid cyclist. He takes no medications, drinks no alcohol, and has no serious medical conditions. Physical examination is unremarkable. Which of the following questions is most likely to help determine the cause of this patient's condition? A. "How often do you ride your bicycle?" B. "Are you sexually attracted to your wife?" C. "Are you feeling unusually anxious lately?" D. "Do you have nocturnal or early morning erections?" E. "Do you love your wife?"

D. "Do you have nocturnal or early morning erections?" Need to determine if ED is functional vs. organic.

372. Toxic megacolon is a known complication for which of the following gastroenterology conditions? A. Chronic pancreatitis B. Eosinophilic colitis C. Acute appendicitis D. Ulcerative colitis (UC) E. Arteriovenous malformations

D. Ulcerative colitis (UC)

252. A patient with a history of hypertension calls his health care provider's office for advice. He has had long-standing heartburn and recently consulted with a gastroenterologist. He underwent an endoscopy and was told that "Barrett mucosa" was found by biopsy. The patient has read in the newspaper that people with this condition will probably develop esophageal cancer. Which of the following is the most appropriate response to this concern? A. "Your concerns are ungrounded." B. "It is foolish to worry because this type of cancer is unlikely to develop and would occur many years later." C. "You should chew food very carefully to prevent the possibility of a mechanical obstruction." D. "Only a small minority of patients with Barrett's esophagus will develop cancer, but you should undergo endoscopic surveillance." E. "You should consult with an oncologist regarding esophageal cancer prevention strategies."

D. "Only a small minority of patients with Barrett's esophagus will develop cancer, but you should undergo endoscopic surveillance

272. A patient is seen in the office 4 weeks following admission to the hospital. If a pancreatic pseudocyst is suspected as a complication of acute pancreatitis, which of the following is the most common presenting sign or symptom? A. Fever B. Jaundice C. Tender, enlarged liver D. Abdominal pain E. Persistent nausea and vomiting

D. Abdominal pain

225. A frail 70-year-old female patient presents to the office with complaints of what she perceives as chronic blood loss through her gastrointestinal (GI) tract. A complete blood count reveals a low hemoglobin and hematocrit with normal mean corpuscular volume (MCV) and red cell distribution width (RDW). She has never been anemic before. Which of the following clinical conditions is most likely? A. None B. Iron deficiency anemia C. Thalassemia D. Anemia of chronic disease/chronic inflammation E. Sideroblastic anemia

D. Anemia of chronic disease/chronic inflammation

176. A patient is seen in the office with recurrent bouts of painless rectal bleeding. The patient's bleeding has spontaneously stopped without other intervention. She is currently bleeding when seen in the office. Which of the following diagnostic evaluations should be performed for this patient? A. Plain abdominal films B. CT scan without contrast C. MRI D. Barium enema E. Colonoscopy

D. Barium enema

332. Which of the following is a known risk factor for the development of adenocarcinoma of the esophagus? A. Chronic ingestion of extremely hot tea B. Smoking opiates C. Smoking cigarettes D. Barrett esophagus E. Comorbid melanoma diagnosis

D. Barrett esophagus

A 62-year-old man reports an episode of gross, painless hematuria. There is no history of trauma, and further questioning determines that he had total hematuria, rather than initial or terminal hematuria. The man does not smoke and has had no other symptoms referable to the urinary tract. He has no known allergies. Physical examination, including rectal examination, is unremarkable. His serum creatinine level is 0.8 mg/dL, and except for the presence of many red blood cells, urinalysis is normal and shows no red blood cell casts. Hematocrit is 46%. Which of the following is the most appropriate initial step in the workup? A. Coagulation studies and urinary cultures B. Place a Foley catheter C. PSA determination and prostatic biopsies D. CT scan with IV contrast of both kidneys, ureters, and bladder, and cystoscopy E. Retrograde cystogram and pyelograms

D. CT scan with IV contrast of both kidneys, ureters, and bladder, and cystoscopy silent hamturia can be caused by renal, ureteral, or bladder cx so need CT.

220. A patient is seen in the office with complaints of nocturnal epigastric pain and gastric pain that is relieved by eating. Stool antigen testing is positive for Helicobacter pylori. Quadruple therapy for this patient includes all of the following medications EXCEPT: A. Omeprazole B. Bismuth subsalicylate C. Metronidazole D. Clarithromycin E. Tetracycline

D. Clarithromycin

79. A 37-year-old accountant comes to the health care provider to ask for advice regarding the future management of his ulcerative colitis. He has had pancolitis for the past 19 years and has been told that he is at an increased risk for developing colorectal cancer. He asks for the health care provider's recommendation regarding appropriate surveillance. Which of the following is the most appropriate response? A. Annual stool guaiac testing B. Barium enema C. Colonoscopy D. Colonoscopy and multiple biopsies E. Flexible sigmoidoscopy with multiple biopsies

D. Colonoscopy and multiple biopsies

A 45-year-old woman reports that she has been having increased involuntary loss of urine after experiencing the sudden need to urinate. This is especially pronounced when she is feeling nervous or while sitting at her desk. Jogging does not worsen the incontinence. She has not had these symptoms in the past and is otherwise healthy. She is not taking any medications and has never been pregnant. On physical examination she is afebrile with stable vital signs. Her abdomen is benign, and vaginal examination reveals no prolapse. Sensation in all extremities is intact, with good motor strength. Her gait is normal, and reflexes are intact. Her workup reveals a negative urinalysis, and blood cultures and urine cultures are all negative. Serum glucose level and glycosylated hemoglobin level is normal. Which of the following tests would be the most appropriate next step in diagnosis? A. IV pyelogram (IVP) B. Stress testing C. Q-tip test D. Cystometry E. Cystoscopy

D. Cystometry Pt may have detrusor instability - this test can asses and evaluate bladder capacity and tone

A 29-year-old woman comes to the health care provider because of fevers and back pain. She is otherwise healthy with no significant past medical history. Examination is significant for a temperature of 38.3º C (101º F), moderate costovertebral angle tenderness, leukocytosis, and white blood cells and red blood cells in the urine. The patient is diagnosed with pyelonephritis and admitted to the hospital for therapy with intravenous antibiotics. Over the next two days, she rapidly improves, and by hospital day 3, she is tolerating oral intake, voiding without difficulty, feeling no pain, and she has not had a fever for 48 hours. Which of the following is the most appropriate next step in management? A. Continue intravenous antibiotics for 2 weeks B. Discharge home and recommend post-coital prophylaxis C. Discharge home off all antibiotics D. Discharge home to complete a 2-week course of oral antibiotics E. Obtain surgical evaluation

D. Discharge home to complete a 2-week course of oral antibiotics ALL PYELONEPHRITIS require tx for AT LEAST *2 weeks*

74. A patient is seen in the office complaining of pain in the epigastrium, radiating to the back. The pain is described as steady and dull but also severe. Lying down and eating meals make the pain worse. The patient is anorexic at present but did have a great deal of nausea and vomiting prior to this. Examination reveals abdominal distention along with epigastric tenderness. Which of the following underlying conditions could lead to this presentation? A. Elevated chylomicrons B. Elevated low-density lipoprotein (LDL) cholesterol C. Low high-density lipoprotein (HDL) cholesterol D. Elevated triglycerides E. Elevated lipoprotein (Lp)(a)

D. Elevated triglycerides

250. A patient with long-standing alcohol-related cirrhosis and ascites is admitted to the hospital with a high fever and mental status change. Examination reveals abdominal pain, vomiting, and fever. Which of the following pathogens is most likely the cause of this presentation? A. Pseudomonas B. Streptococcus pneumoniae C. Staphylococcus aureus D. Escherichia coli E. Klebsiella

D. Escherichia coli

233. A 62-year-old social worker with a past history of recurrent GERD, complains of increasing difficulty with swallowing. She has had trouble with solid foods and senses a discomfort in the mid-lower chest after eating meats or dry bread. She has not lost any weight and denies any other medical problems. There is no family history of gastrointestinal malignancy. Physical examination is unremarkable. Which of the following would be the most appropriate next step in the evaluation of this patient's symptoms? A. Barium esophagram B. Chest radiograph C. CT scan of the chest D. Esophagoscopy E. 24-hour pH monitor

D. Esophagoscopy

58. A 22-year-old man is diagnosed with Crohn's disease limited to the terminal ileum. His symptoms of mild right lower quadrant pain and postprandial diarrhea resolve after the initiation of treatment with mesalamine. Two years later, he develops recurrent episodes of abdominal distention, nausea, and vomiting after large meals. On two occasions, these symptoms are accompanied by inability to pass flatus or bowel movements. Which of the following has this patient most likely developed? A. A fistula from the ileum to the sigmoid B. An obstructing cecal carcinoma C. An obstructing ileal carcinoid D. Fibrosis and a stricture in the terminal ileum E. Gastric outlet obstruction

D. Fibrosis and a stricture in the terminal ileum

149. A patient has forceful vomiting and then develops severe chest pain. Physical examination reveals Hamman's sign. Which of the following is the next urgent step in the evaluation of this patient? A. CT of the chest B. MRI C. MRA D. Gastrografin swallow E. Endoscopy

D. Gastrografin swallow

286. In addition to preventing hepatitis B in a patient, hepatitis B vaccination will also prevent which of the following other types of hepatitis? A. HAV B. None C. HCV D. HDV E. HEV

D. HDV

24. A patient with esophageal varices had failed clinical intervention to prevent further bleeding. The patient undergoes transjugular portosystemic shunt (TIPS) surgery in order to lower portal pressures. Which of the following is the most concerning complication for the TIPS procedure? A. Dumping syndrome B. Diabetes insipidus C. Hepatorenal syndrome D. Hepatic encephalopathy E. Spontaneous bacterial peritonitis

D. Hepatic encephalopathy

A 65-year-old woman with a long-standing disease has bone marrow fibrosis and increased bone remodeling, with bone resorption exceeding bone formation. She has a history of passing calcium-oxalate kidney stones. Which of the following laboratory result profiles would be expected in this patient's serum? A. Increased serum potassium B. Low serum parathyroid hormone, low serum calcium C. High serum parathyroid hormone, low serum calcium D. High parathyroid hormone, increased serum calcium, low serum phosphate E. Increased serum sodium, low serum potassium

D. High parathyroid hormone, increased serum calcium, low serum phosphate This patient has long-standing hyperparathyroidism (elevated PTH), which predisposes to the development of osteitis fibrosa, her bone disease. PTH acts initially on osteocytes of bone tissue (osteocytic osteolysis) and subsequently on osteoclasts (osteoclastic resorption) to resorb calcium from bone matrix and make it available to the circulation. This increases plasma calcium levels. PTH also causes decreased phosphate reabsorption in the proximal renal tubule, yielding hypophosphatemia. Hypercalciuria is another sequela of excess PTH production, which predisposes the patient to the formation of calcium oxalate stones.

A 27-year-old woman comes to the health care provider complaining of fever and back pain. She states that a few days ago she had burning with urination. Over the next few days she developed fever with chills and had pain on the right side of her back. She has no medical problems and takes no medications. Her temperature is 38.9° C (102° F), blood pressure 110/70 mm Hg, pulse 102/min, and respirations 16/min. Examination shows a patient in mild distress with shaking chills and right costovertebral angle tenderness. Leukocyte count is 18,000/mm3. Urinalysis shows 100 leukocytes/high-power field. Which of the following is the most appropriate next step in management? A. Observation only B. Spinal MRI scan C. Outpatient management with oral trimethoprim-sulfamethoxazole D. Hospital admission and initiation of IV ciprofloxacin (Cipro) E. Hospital admission and administration of a 2-week course of IV tetracycline

D. Hospital admission and initiation of IV ciprofloxacin (Cipro) Pyelonephritis - combo of back pain, fever, chills, dyrusia, N/V. Can be managed out pt if the patient is healthy and has NO complicating factors. This patient w/ fever and shaking chills and elevated leukocytes COULD have sepsis, which requires admission.

54. Which of the following metabolic conditions is most likely to cause constipation? A. Addison disease B. Diabetes C. Short bowel syndrome D. Hypothyroidism E. Gastrinoma

D. Hypothyroidism

67. Which of the following treatments is recommended for patients with erosive esophagitis? A. Esophageal dilation B. Fluticasone used without a spacer C. Botulism toxin injection D. Long-term proton-pump inhibitor (PPI) therapy E. Long-term H2 blockers

D. Long-term proton-pump inhibitor (PPI) therapy

349. A 5-year-old boy is brought to the emergency department with 2-day history of fever, anorexia, loose stools, and yellow skin color. He attends a large daycare center. On physical examination his temperature is 38.1ºC (100.7ºF), blood pressure 88/56 mm Hg, pulse 74/min, and respirations 15/min. Initial laboratory evaluation reveals a total bilirubin of 1.8 mg/dL (normal <1.9 mg/dL) and alanine aminotransferase 764 U/L (normal <25 U/L in this age group). Nobody else in his family is sick. Which of the following is the most appropriate diagnostic test? A. Hepatitis B surface antigen in serum B. IgG for hepatitis A in serum C. IgG for hepatitis B surface antigen in serum D. IgM for hepatitis A in serum E. Stool culture for hepatitis A

D. IgM for hepatitis A in serum

301. A 57-year-old woman with a 30-year history of alcoholism and liver disease visits her health care provider complaining of abdominal swelling and shortness of breath. The health care provider determines that she has severe ascites. Which of the following factors contributes to the accumulation of fluid in the abdominal cavity? A. Decreased plasma epinephrine and norepinephrine B. Decreased plasma volume C. Increased hepatic lymph flow D. Increased hydrostatic pressure in splanchnic capillary beds E. Increased natriuresis

D. Increased hydrostatic pressure in splanchnic capillary beds

34. Pancreatitis can occur as a result of certain metabolic disorders. This is the reason as to why people are treated for these conditions so that acute pancreatitis does not occur. Which of the following metabolic conditions is most commonly associated with acute pancreatitis? A. Increased low-density lipoprotein (LDL) cholesterol B. Decreased high-density lipoprotein (HDL) cholesterol C. Increased chylomicrons D. Increased triglycerides E. Increased lipoprotein (Lp)(a)

D. Increased triglycerides

Which of the following is a known cause for a false negative Hemoccult study of the stool? A. Ingestion of red meat B. Ingestion of red Kool Aid C. Ingestion of iron D. Ingestion of vitamin C E. Ingestion of bismuth salicylate

D. Ingestion of vitamin C

331. A 24-year-old dress designer complains of a crampy periumbilical pain over the past 9 months since she began her first job. During that time she has experienced several episodes of constipation lasting 4-5 days, typically followed by 3-4 days of frequent loose bowel movements. She denies any bloody stools, fever, weight loss, or change in appetite. Her symptoms are generally milder on weekends. Physical examination is normal. White blood cell count is 6,700/mm3 (normal 5,000-10,000/mm3), hematocrit 38%, (normal 36-44%), and erythrocyte sedimentation rate 4 mm/h (normal <25 mm/h in women). Serum albumin and liver function tests are normal. Which of the following is the most likely diagnosis? A. Crohn's disease B. Diverticulosis C. Giardia infection D. Irritable bowel syndrome E. Ulcerative colitis

D. Irritable bowel syndrome

A 27-year-old-man with no significant past medical history comes to the health care provider for an infertility evaluation. He and his wife have been trying to conceive for 2 years without success. A complete workup of his wife found no abnormalities. History reveals that the patient had normal childhood development but late puberty. He had problems throughout his life in school secondary to what counselors told him was a moderate learning disability. Physical examination reveals a tall man with long legs and arms that seem out of proportion to the rest of the body. His testes are small, and moderate gynecomastia is present. Early cataracts are noted. Which of the following findings is consistent with this patient's condition? A. Decreased luteinizing hormone, follicle-stimulating hormone, and testosterone B. Elevated prolactin level C. Increased luteinizing hormone to follicle-stimulating hormone level D. Karyotype 47,XXY E. 21-Hydoxylase deficiency

D. Karyotype 47,XXY

63. You are following a patient in the hospital with hepatorenal syndrome. The patient's medical course continues to deteriorate. Which of the following treatments will provide the best outcome for this patient and will also improve the status of the kidney? A. Fluid expansion with IV normal saline B. Fluid expansion with potassium supplementation C. Vasopressin D. Liver transplant E. Kidney transplant

D. Liver transplant

133. A 58-year-old man with known hepatitis C and cirrhosis complains of worsening fatigue and confusion over the past 5 days. He has been admitted 3 times in the past 4 months for variceal bleeding and has had ascites that has been refractory to high-dose oral diuretic use. He also reports that over the past 48 hours he has had a declining urinary output. On physical examination, he is gaunt and jaundiced. He has tense ascites and a liver span of 7 cm in the midclavicular line. Laboratory results reveal a white blood cell count of 4,600/mm3 (normal 5,000-10,000/mm3), hemoglobin 9.4 g/dL (normal 13.8-17.2 g/dL), and hematocrit 29% (normal 41-50%). His electrolytes reveal BUN 34 mg/dL (normal 7-20 mg/dL) and creatinine 3.1 mg/dL (normal in men 0.8-1.4 mg/dL). Urinary sodium is less than 10 mEq/L (normal spot sodium >20 mEq/L). Which of the following is the most appropriate treatment for his elevated BUN and creatinine? A. Hemodialysis B. Kidney transplantation C. Large volume paracentesis D. Liver transplantation E. Mesocaval shunt

D. Liver transplantation

244. A patient is admitted to the hospital with suspected acute diverticulitis. Which of the following signs or symptoms would be unusual for patients who have acute diverticulitis? A. Altered bowel habits (constipation or diarrhea) B. Left lower quadrant pain C. Fever D. Lower gastrointestinal bleeding E. Leukocytosis

D. Lower gastrointestinal bleeding

333. A previously healthy 11-year-old boy is brought to the health care provider with a fever and persistent vomiting for 4-5 days. Initially the emesis was clear, but now it contains streaks of bright red blood. Physical examination, complete blood count, and serum electrolytes are all normal. Which of the following is the most likely diagnosis? A. Esophageal varices B. Esophagitis C. Gastritis D. Mallory-Weiss tear E. Peptic ulcer disease

D. Mallory-Weiss tear

196. A 70-year-old man complains of chronic heartburn. It is painful for him to bend over, and he sleeps on a wedge-shaped pillow to try to reduce the burning sensation. Which of the following agents would be most efficacious in reducing his symptoms? A. Bisacodyl (Dulcolax) B. Cimetidine (Tagamet) C. Magnesium hydroxide (Milk of Magnesia) D. Omeprazole (Prilosec) E. Promethazine (Phenergan)

D. Omeprazole (Prilosec)

279. A 24-year-old graduate student is found to have acute myelogenous leukemia (AML). Before undergoing a planned bone marrow transplant, he begins aggressive multi-drug chemotherapy. His tolerance of this regimen is severely limited, however, by intractable vomiting with each chemotherapy cycle. Which of the following will most likely reduce vomiting in this patient? A. Dopamine B. Erythromycin C. Omeprazole (Prilosec) D. Ondansetron (Zofran) E. Opiates

D. Ondansetron (Zofran)

A 24-year-old woman comes to the health care provider because of burning with urination. She states that every time she urinates there is pain and that she has a feeling that she constantly needs to urinate even though only a little comes out. She has never had any similar symptoms before. She has no medical problems and no known drug allergies. Examination is unremarkable. Urinalysis demonstrates that the urine is positive for leukocyte esterase and nitrites. Which of the following is the most appropriate pharmacotherapy? A. Intramuscular ceftriaxone B. IV levofloxacin C. Oral levofloxacin for 7 days D. Oral trimethoprim-sulfamethoxazole for 3 days E. Wait for the culture results to institute therapy

D. Oral trimethoprim-sulfamethoxazole for 3 days

215. A patient undergoes a total gastrectomy because of a proximally located gastric cancer. After the surgery, which of the following digestive enzymes will be produced in inadequate amounts? A. Amylase B. Chymotrypsin C. Lipase D. Pepsin E. Trypsin

D. Pepsin

336. A 34-year-old tax lawyer comes to the clinic complaining of difficulty swallowing. On several occasions over the past few months, he has been aware of meat becoming stuck in the mid-chest immediately after eating. After each episode, he has several hours of chest pain that resolves gradually. On 2 occasions he induced vomiting to obtain relief. Over the past 10 days the difficulty swallowing has become worse, and he now has trouble with even soft foods. He has been taking ranitidine (Zantac), magnesium hydroxide, and omeprazole (Prilosec) for 4 years but has remained symptomatic despite these measures. He has smoked 1 pack of cigarettes daily for 15 years and denies any alcohol use. Physical examination is normal. Which of the following is the most likely cause for these symptoms? A. Diffuse esophageal spasm B. Esophageal squamous carcinoma C. Lower esophageal web D. Peptic esophageal stricture E. Scleroderma

D. Peptic esophageal stricture

269. A 42-year-old alcoholic man comes to the emergency department with severe abdominal pain that began 3 hours ago. He is well known to the medical staff because of 3 prior hospitalizations for acute pancreatitis. He says that this time his pain began suddenly and in short order became constant, generalized, and extremely severe. He lies motionless on the stretcher, and on physical examination he has tenderness, rigidity, and rebound in all four abdominal quadrants. Bowel sounds are absent. Initial studies are significant for a serum amylase of 1,200 U/L and free air under both diaphragms on an upright plain film of the abdomen. Which of the following is the most likely diagnosis? A. Acute edematous pancreatitis B. Acute hemorrhagic pancreatitis C. Gangrenous acute cholecystitis D. Perforated viscus E. Ruptured pancreatic pseudocyst

D. Perforated viscus

A 28-year-old man is 7 hours status-post an appendectomy and complains of severe lower abdominal-suprapubic pain. On examination his incision is clean, dry, and intact and without pain to the touch. Palpation of the suprapubic area produces intense discomfort. He has not urinated since the procedure. The nurse notes that he is tachycardic with a regular pulse of 110/min. Which of the following is the most appropriate next step in management? A. Administer a beta-blocker such as metoprolol B. Administer an anti-adrenergic agent such as doxazosin C. Begin IV antibiotics D. Perform a straight catheterization of the bladder E. Place a nasogastric tube (NGT)

D. Perform a straight catheterization of the bladder

81. In addition to lack of relaxation of the lower esophageal sphincter, which of the following other criteria is needed in order to make the diagnosis of achalasia? A. Esophageal ulceration B. Esophageal web C. Esophageal ring D. Poor peristalsis of the esophagus E. Alkaline pH of the esophageal aspirate

D. Poor peristalsis of the esophagus

126. A 53-year-old woman comes to the health care provider complaining of fatigue over the past 6 months. During this time, she has also developed pruritus and lost 4 pounds. She is not sexually active. On physical examination she is afebrile and has mildly icteric sclera. There are excoriations noted on all 4 extremities and trunk and back. The liver edge is smooth and non-tender and measures 9 cm at the midclavicular line. There is no ascites, splenomegaly, or peripheral edema. Laboratory results reveal a normal complete blood count, normal electrolytes, liver function tests revealing an alkaline phosphatase of 260 U/L (normal <110 U/L), total bilirubin of 3.1 mg/dL (normal <1.9 mg/dL), and normal transaminase levels. Which of the following is the most likely diagnosis? A. Acute cholecystitis B. Acute hepatitis A infection C. Bacterial cholangitis D. Primary biliary cirrhosis E. Primary sclerosing cholangitis

D. Primary biliary cirrhosis

50. Various treatments are given for the management of gastrointestinal (GI)-related complaints. What is the mechanism of action for metoclopramide (Reglan) in the management of GI complaints? A. Blocks histamine release B. Blocks histamine receptors C. Stimulates the proton pump to stop making acid D. Promotility agent E. Antacid

D. Promotility agent

A 45-year-old man consults his health care provider because of dysuria. The patient is treated with antibiotics, but symptoms recur 1 week after antibiotic therapy is stopped. A different antibiotic is tried, but symptoms again recur after cessation of the antibiotic. Rectal examination demonstrates an enlarged prostate with areas of tenderness and a fluctuant mass. Which of the following is the most likely diagnosis? A. Benign prostatic hyperplasia B. Chronic nonbacterial prostatitis C. Prostadynia D. Prostatic abscess E. Prostatic carcinoma

D. Prostatic abscess This should be suspected in males w/ recurrent UTI's that initially improve w/ abx but recur.

370. Patients with chronic pancreatitis need to have pain control along with an intervention that will result in the resolution of malabsorption. Which of the following is the most effective intervention for managing malabsorption in this setting? A. Insulin administration B. Use of a GLP-1 agonist such as exenatide (Byetta) C. Cholestyramine (Questran) D. Replacement of pancreatic enzymes E. IV lipid emulsion therapy

D. Replacement of pancreatic enzymes

A 24-year-old woman sustains multiple injuries in a car accident, including a pelvic fracture. She is hemodynamically stable. Initial assessment shows no vaginal or rectal injuries; however, when a Foley catheter is inserted, bloody urine is recovered. Which of the following would be the best way to evaluate her urologic injury? A. Ultrasound of the bladder B. IV pyelogram C. Cystoscopy D. Retrograde cystogram including post-void films E. Retrograde cystogram including views of the ureters

D. Retrograde cystogram including post-void films Bloody urine + pelvic fx = bladder injury! Injecting dye will help visualize, and post-void gives better visualization of some areas.

112. Three months after a needle-stick exposure to blood from a patient with hepatitis B, a nurse is evaluated for infection with the virus. Laboratory results reveal: HBsAg: absent anti-HBs antibody: absent IgM anti-HBc: present IgG anti-HBc: absent HBeAg: absent On the basis of these results, which of the following most accurately describes the nurse's hepatitis B status? A. She had been effectively vaccinated against hepatitis B before the needle-stick exposure occurred. B. She has mounted an inappropriate antibody response to hepatitis B as a result of an immunocompromised state. C. She is a carrier of hepatitis B. D. She is actively infected with hepatitis B. E. She was not infected with hepatitis B.

D. She is actively infected with hepatitis B.

229. Which of the following anatomic regions of the colon is most commonly involved with diverticulosis for patients living in the United States? A. Cecum B. Transverse colon C. Splenic flexure D. Sigmoid colon E. Rectum

D. Sigmoid colon

172. Which of the following colonic conditions can be treated with sigmoidoscopy? A. Diverticulitis B. Angiodysplasia C. Diverticulosis D. Sigmoid volvulus E. Cecal volvulus

D. Sigmoid volvulus

A 53-year-old woman with no past medical history complains of incontinence. She thinks that she must have "a small bladder capacity" because she has to go to the restroom so frequently. She also complains that she tends to lose her urine when she coughs or laughs. Particularly embarrassing is the fact that when she goes to the gym, she has to wear an undergarment while she does light weightlifting so she does not lose her urine in public. A recent urinalysis and urine culture she had at her employee health center were unremarkable. She denies recent back or head injury, neurologic dysfunction, and renal, bladder/urethral/ureteral disorder. She denies any psychological disorder and appears well-adjusted. She only takes oral contraceptive pills regularly; she does not drink or smoke but admits to drinking 3 cups of coffee a day. Which of the following is the most likely diagnosis? A. Drug-induced incontinence B. Functional incontinence C. Overflow incontinence D. Stress incontinence E. Urge incontinence

D. Stress incontinence

A 57-year-old painter comes to his health care provider for evaluation of difficulty voiding for the past few years. He describes a delay when he "attempts to go" and that he often wets his underwear with additional urine after he has finished. He is married and has a teenage child. His past medical history is remarkable for essential hypertension that he claims is diet controlled. Blood pressure is 160/92 mm Hg and pulse 82/min. Physical examination is remarkable for a prominent apical impulse, laterally displaced, and faint abdominal bruit. The prostate is nontender and appears large with no palpable nodules or irregularities. Which of the following is the most appropriate antihypertensive medication for this patient? A. Lisinopril (Zestril, Prinivil) B. Metoprolol (Lopressor, Toprol) C. Nifedipine (Procardia) D. Terazosin (Hytrin) E. Verapamil (Calan, Isoptin)

D. Terazosin (Hytrin) alpha-1-receptor blocker tx HTN and can help w/ BPH by relaxing smooth muscle tissue in bladder

Foot examination is performed on a 45-year-old man who was diagnosed with type 1 diabetes mellitus at age 15. Although the man has no complaints of foot pain, both feet appear clearly deformed, with loss of the normal arches and abnormal orientation of the toes. No inflammation is noted. Radiographs show deformed bones with new bone formation adjacent to the bone cortex. Several large, bizarrely shaped osteophytes are seen at the joint margins. Which of the following is the most likely diagnosis? A. Avascular necrosis B. Gout C. Infectious arthritis D. Neurogenic arthropathy E. Rheumatoid arthritis

D. This patient most likely has neurogenic arthropathy. The disorder develops in a setting of impaired pain perception and position sense, and can cause a rapidly destructive osteoarthritis-like arthropathy. The problems appear to be caused by unfelt minor injury, without the normal response of pain, causing resting of, and natural splinting (by muscle contraction or shifting position) of the affected joints. The most commonly encountered setting is diabetic foot disease. The joints affected are those that have lost pain innervation, possibly in association with palsies of nearby muscles.

156. A patient is seen with abdominal pain and bloody diarrhea. The patient has a family history of many abdominal complaints. Which of the following clinical conditions has histologic features that consist of epithelial damage, inflammation, crypt abscesses, and loss of goblet cells in colonic biopsies? A. Chronic pancreatitis B. Eosinophilic colitis C. Crohn disease D. Ulcerative colitis E. Arteriovenous malformations

D. Ulcerative colitis

181. Some systemic diseases have physical manifestations outside of the original site that the condition affects. For example, extraintestinal signs of disease can help to diagnose an underlying gastrointestinal (GI) problem. Pyoderma gangrenosum is a skin manifestation of which of the following GI conditions? A. Chronic pancreatitis B. Eosinophilic colitis C. Crohn disease D. Ulcerative colitis E. Arteriovenous malformations

D. Ulcerative colitis

A 7-year-old boy is brought to the clinic for a lifetime history of bed-wetting. He has otherwise been completely healthy and has met all developmental milestones. His parents deny a history of trauma, and the history is not consistent with abuse. The patient has been wetting every night but not during the daytime. He has no incontinence. Which of the following is the most appropriate next step in his evaluation? A. IV pyelogram B. Renal ultrasound C. 24-hour urine collection D. Urinalysis E. CT of pelvis

D. Urinalysis Initial analysis - as in the NEXT STEP and not the "best dx step" is the UA to r/o infection or blood. noninvasive, easy, fast.

A 67-year-old man comes to the emergency department because he has not been able to void for the past 12 hours. He feels the need to, but he cannot do it. He gives a history that, for several years now, he has been getting up 4 or 5 times a night to urinate. It would take him a considerable time to get the urinary stream going, and the stream typically lacked force and often ended in a dribble. Because of a cold 2 days ago he began taking an antihistamine, a decongestant, and drinking plenty of fluids. Physical examination shows a palpable, smooth, round mass arising from the pubis and reaching about halfway toward the umbilicus. The mass is dull to percussion, and pushing on it accentuates the feeling of needing to void. Rectal examination reveals a large, boggy, nontender prostate gland without nodules. This is a classic presentation for which of the following acute conditions? A. Bacterial prostatitis B. Cystitis in a patient with bladder cancer C. Renal failure D. Urinary retention in a patient with benign prostatic hyperplasia E. Urinary retention in a patient with prostate cancer

D. Urinary retention in a patient with benign prostatic hyperplasia Hx & DRE = BPH --> large palpable bladder

A 62-year-old woman had an abdominal hysterectomy and salpingo-oophorectomy 3 days ago. She had an indwelling bladder catheter during the procedure, which was removed in the recovery room. She has been voiding at will since then. She also had compression pneumatic stockings on both lower extremities during the operation. She began ambulation on the first postoperative day, and has been as active as possible under the circumstances, including faithful adherence to a prescribed program of incentive spirometry. On the evening of the third postoperative day she spikes a fever, with a temperature to 39.4º C (103º F). Which of the following is the most likely source of the fever? A. Atelectasis B. Deep thrombophlebitis C. Intra-abdominal abscess D. Urinary tract infection E. Wound infection

D. Urinary tract infection

A 4-year-old boy falls from the jungle gym at preschool. He sustains minor abrasions and contusions, and is taken care of by the school nurse. His parents take him that same afternoon to his regular health care provider and demand "a thorough checkup" for possible internal injuries. The provider complies, and a complete physical examination is normal. His hemoglobin is 14 g/dL, and a urinalysis shows the presence of microscopic hematuria. There are no additional signs of bleeding. Which of the following is the most appropriate next step in management? A. CT scan of the abdomen and pelvis B. Reassure the parents that microscopic hematuria from minor trauma will resolve spontaneously C. Serial hemoglobin and hematocrit determinations D. Urologic workup, starting with an ultrasound E. Retrograde ureterogram and cystogram

D. Urologic workup, starting with an ultrasound Hx and presence of hematuria warrants evaluation. ANY hematuria does! so do the least invasive and easiest as first step.

130. Clinicians should understand the mechanism of action when prescribing therapy for patients with inflammatory bowel disease. Medication reconciliation for a patient with mild to moderate Crohn disease reveals that this patient is taking budesonide as part of the treatment. What activity does budesonide have in this setting? A. Heals anal fistulas B. Heals enterovesical fistulas C. Used for patients who cannot take antibiotics D. Used for patients with predominately ileal and right colon disease E. Used for patients who have left-sided colon disease

D. Used for patients with predominately ileal and right colon disease

A 15-year-old boy presents to the physician assistant with a chief complaint of burning on urination. A purulent urethral discharge is noted on physical examination; a Gram stain of the smear is consistent with gonorrhea. The patient explains that he had sexual intercourse with a consenting female friend who is the same age as the patient approximately 7 days earlier. The physician assistant may treat the patient A. but must have both parents' consent B. but must inform the parents C. with one parent's consent D. without a parent's consent or informing the parents E. without informing parents, but must inform the court

D. without a parent's consent or informing the parents

What is the MC complication of diabetes?

Diabetic peripheral neuropathy

A 44-year-old homeless woman is brought to the emergency department because she is "bleeding from the breast." Physical examination shows a huge, fungating, ulcerated mass that occupies the entire right breast and is firmly attached to the chest wall. The right axilla is full of hard masses that are not movable either. Core biopsies of the breast are interpreted as undifferentiated infiltrating ductal carcinoma, and assays for estrogen and progesterone receptors are negative. Which of the following is the most appropriate next step in management?

Diathermy, radiation, and chemotherapy. Although this is an impressive, very advanced cancer with a poor prognosis, the tumor can be expected to shrink significantly with local heat application, radiation, plus systemic chemotherapy. It may do so to the point at which a palliative mastectomy becomes technically feasible, something that cannot be done at this time. The tumor is staged at the time of surgical intervention so pre-surgical neoadjuvant therapy can help to limit the size of the tumor which may have an effect on overall prognosis of the patient. Surgery does not typically play a role in patients who have metastatic breast cancer.

A 22-year-old woman develops secondary amenorrhea and galactorrhea, and an MRI scan of the head reveals a small intrasellar tumor. Which of the following is the most appropriate pharmacologic treatment?

Dopamine Agonist. Whenever secondary amenorrhea develops in association with galactorrhea, hyperprolactinemia should be suspected. A prolactin-secreting pituitary adenoma is the most frequent cause of this condition, which can be treated either pharmacologically or surgically. Prolactin secretion is regulated by factors released by the hypothalamus. In this case, prolactin secretion is inhibited by dopamine. Drugs such as bromocriptine (Parlodel) and cabergoline (Dostinex), which act as agonists on dopamine receptors, thus will inhibit prolactin release. In contrast, dopamine antagonists, such as most antipsychotic drugs, may increase prolactin secretion and cause amenorrhea-galactorrhea syndrome. Long-term use of the older, traditional anti-psychotics was known to lead to this complication.

A 35-year-old woman arrives on the medical-surgical floor after an uneventful hysteroscopy to evaluate her long history of uterine fibroids. About 30 minutes after her arrival, she begins to complain of nausea and has 2 episodes of vomiting. The health care provider administers 0.625 mg of droperidol (Inapsine) and 400 mg of acetaminophen by mouth. On follow-up evaluation the patient's neck is involuntarily flexed to one side. She is alert, oriented, and conversant and has an otherwise normal neurologic examination. Which of the following is the most likely diagnosis?

Dystonic reaction to droperidol (Inapsine). The most likely diagnosis is a dystonic reaction to the droperidol. Droperidol causes its antiemetic effect by antagonizing dopaminergic receptors in the vomiting center (central chemoreceptor zone) of the brain. This antidopaminergic action can produce torticollis or other dystonias.

219. A patient undergoes esophagogastroduodenoscopy, and squamous cell carcinoma of the esophagus is found. All of the following are known risk factors for the development of this condition EXCEPT: A. Tobacco use B. Alcohol use C. Achalasia D. Lye ingestion E. Chronic indigestion/reflux

E. Chronic indigestion/reflux

190. All of the following are risk factors for the development of sigmoid volvulus EXCEPT: A. Institutionalized elderly B. Chronic constipation C. Use of antimotility drugs D. Chronic laxative abuse E. Congenital lack of fixation of sigmoid colon

E. Congenital lack of fixation of sigmoid colon

262. Patients who are suspected of having common duct stones or who want to have common duct stone removal can undergo either endoscopic retrograde cholangiopancreatography (ERCP) or magnetic resonance cholangiopancreatography (MRCP). Which of the following complications is seen after ERCP procedure but not MRCP? A. Ileal injury with resultant malabsorption B. Upper gastrointestinal bleeding C. Jaundice D. Acute cholangitis E. Acute pancreatitis

E. Acute pancreatitis

A 7-year-old girl complains of increased urinary frequency, dysuria, and itching on urination. Her urinalysis is consistent with a urinary tract infection. This is her 20th infection in the past year, despite adequate antibiotic coverage. Further imaging of her bladder, kidneys, and ureters reveals findings that are consistent with vesicoureteral reflux. Which of the following is the next appropriate step? A. CT scan of the pelvis B. IV antibiotics for 2 weeks C. IV pyelogram D. Renal arteriogram E. Antireflux surgery

E. Antireflux surgery common anatomic cause of recurrent UTI's - retrograde flow of urine into uretes and/or kidney before voiding.

276. On review of a patient's laboratory results during a follow-up visit, the patient's initial laboratory profile revealed that he had an elevated alkaline phosphatase level. A gamma-glutamyl transferase (GGT) was then ordered, and it has returned as normal. Which of the following conditions is consistent with this finding? A. Ongoing intake of alcohol B. Blockage of the gallbladder C. Acute hepatitis D. Sclerosis of the gallbladder ducts E. Bone disease

E. Bone disease

55. Although both Crohn disease and ulcerative colitis are both types of inflammatory disease, their treatment may be significantly different. Which of the following therapeutic agents is much more useful in patients with Crohn disease rather than ulcerative colitis? A. Sulfasalazine B. Systemic corticosteroids C. Azathioprine D. 6-Mercaptopurine (6-MP) E. Cholestyramine (Questran)

E. Cholestyramine (Questran)

A 2-week-old infant is brought to the clinic because of vomiting and severe dehydration. The infant was born at home to a 23-year-old woman who did not receive any prenatal care. Physical examination shows dehydration and ambiguous genitalia. Laboratory studies show hyponatremia. Additional laboratory studies are most likely to reveal which of the following findings? A. Decreased levels of plasma renin B. Decreased levels of serum 17-hydroxyprogesterone C. Elevated levels of aldosterone D. Elevated levels of serum chloride E. Elevated levels of serum potassium

E. Elevated levels of serum potassium This infant most likely has congenital adrenal hyperplasia caused by 21-hydroxylase deficiency. 21-Hydroxylase deficiency results in virilization and ambiguous genitalia in females. Patients may present with vomiting and hyponatremic dehydration with hypotension and salt-wasting with low aldosterone levels. Salt-wasting leads to low serum sodium with resultant elevation of serum potassium.

93. A 4-year-old girl is found drinking liquid drain cleaner by her parents and is immediately brought to the emergency department. She appears very irritable and is unwilling to swallow any liquid medication. Examination of the oral cavity reveals no evidence of burns or ulceration. Which of the following is the most appropriate management of this patient? A. Barium swallow B. CT scan of the abdomen C. NPO for 12 hours, then clear liquid diet for 3 days D. Indirect laryngoscopy E. Esophagoscopy

E. Esophagoscopy

119. A 31-year-old woman smashes her car against a bridge abutment. She sustains multiple injuries, including upper and lower extremity fractures. She is fully awake and alert, and she reports that she was not wearing a seat belt and distinctly remembers hitting her abdomen against the steering wheel. Her blood pressure is 135/75 mm Hg and pulse88/min. Physical examination reveals a rigid and tender abdomen. There is severe tenderness when external pressure is applied to her abdomen and then suddenly released. She has no bowel sounds. Which of the following would be the most appropriate step to pursue for this patient? A. Continued clinical observation B. CT scan of the abdomen C. Sonogram of the abdomen D. Diagnostic peritoneal lavage E. Exploratory laparotomy

E. Exploratory laparotomy

285. Which of the following is the most likely cause of a patient with cirrhosis to develop macrocytic anemia? A. Blood loss anemia from poor development of clotting factors B. Concurrent hypothyroidism C. Vitamin B12 deficiency state D. Pernicious anemia state E. Folate deficiency

E. Folate deficiency

115. Treatment of acute viral hepatitis may include all of the following interventions EXCEPT: A. High-calorie diet B. IV hydration if vomiting is a persistent symptom C. Cholestyramine (Questran) for pruritus D. Liver transplantation for fulminant hepatic failure E. Glucocorticoids for persistent inflammation

E. Glucocorticoids for persistent inflammation

200. In addition to hepatitis A (HAV), which other type of hepatitis is obtained from fecal-oral transmission? A. None B. HBV C. HCV D. HDV E. HEV

E. HEV

271. Which type of hepatitis infection has a high fatality rate if acquired during pregnancy? A. HAV B. HBV C. HCV D. HDV E. HEV

E. HEV

174. Clinical intervention for fulminant hepatitis includes all of the following EXCEPT: A. Restriction of protein intake B. Lactulose C. Neomycin D. Prophylactic antibiotics E. Hemodialysis to prevent hepatorenal syndrome

E. Hemodialysis to prevent hepatorenal syndrome

298. Which of the following conditions causes the highest elevations in aspartate transaminase (AST) and alanine transaminase (ALT)? A. Hepatitis A B. Hepatitis B C. Hepatitis C D. Alcoholic hepatitis E. Hepatic necrosis

E. Hepatic necrosis

292. A patient with long-standing cirrhosis is examined in the office. The patient is noted to have spider angiomas, palmar erythema, gynecomastia, and testicular atrophy. Which of the following underlying conditions is responsible for these findings? A. Low albumin levels B. Chronically low glucose levels C. Hepatic encephalopathy D. Elevated bilirubin levels E. High estrogen levels

E. High estrogen levels

189. A patient is seen in the office complaining of pain in the epigastrium, radiating to the back. The pain is described as steady and dull but also severe. Lying down and eating meals make the pain worse. The patient is anorexic at present but did have a great deal of nausea and vomiting prior to this visit. Examination reveals abdominal distention along with epigastric tenderness. Which of the following electrolyte abnormalities can lead to this presentation? A. Hyponatremia B. Hypokalemia C. Hyperkalemia D. Hypocalcemia E. Hypercalcemia

E. Hypercalcemia

258. A patient is seen in the office complaining of pain in the epigastrium, radiating to the back. The pain is described as steady and dull but also severe. Lying down and eating meals make the pain worse. The patient is anorexic at present but did have a great deal of nausea and vomiting prior to this. Examination reveals abdominal distention along with epigastric tenderness. Which of the following electrolyte abnormalities can lead to this presentation? A. Hyponatremia B. Hypokalemia C. Hyperkalemia D. Hypocalcemia E. Hypercalcemia

E. Hypercalcemia

A 72-year-old man has a 3-mm ureteral stone impacted at the ureterovesical junction. He has been having mild ureteral colicky pain for about 12 hours, and he has been given fluids and analgesics in the expectation that he will spontaneously pass the stone. He then has shaking chills and spikes a temperature of 40° C (104° F). When seen shortly thereafter, he has flank pain and looks quite ill. Which of the following is the most appropriate next step in management? A. Addition of IV antibiotics to the current therapeutic regimen B. Crushing and extraction of the stone via cystoscopy C. Extracorporeal shock wave lithotripsy and parenteral antibiotics D. Immediate insertion of a suprapubic catheter into the bladder E. IV antibiotics and immediate decompression of the urinary tract above the stone

E. IV antibiotics and immediate decompression of the urinary tract above the stone obstruction + infection = bad news. restore flow and IV abx

352. Which of the following is a contributing cause of a patient having low-grade mucosal-associated lymphoid tissue (MALT) lymphoma? A. Smoking cigarettes B. Eating highly processed foods containing nitrates C. Eating raw foods D. Blood type A E. Infection with Helicobacter pylori

E. Infection with Helicobacter pylori

89. A 56-year-old alcoholic comes to the health care provider complaining of 6 months of worsening midepigastric pain radiating to his back. The pain is exacerbated by eating and continued alcohol use. One month ago he was told that he had developed diabetes, which requires insulin for control. He has also noted that, over the past 2 months, he has had greasy, foul-smelling, large-volume stools. Which of the following is the most likely cause of this man's insulin-dependent diarrhea? A. Acid inactivation of pancreatic enzymes B. Bacterial overgrowth C. Biliary insufficiency D. Excess secretion of glucagon E. Insufficient lipase available for normal fat digestion

E. Insufficient lipase available for normal fat digestion

A 45-year-old man with history of previously well-controlled hypertension presents to the office because colleagues at his office are telling him that his appearance is changing. The patient has recently noted that his old shoes felt too tight and he had to buy new, larger shoes. He also complains of shooting pains in his hands, which his company nurse told him were consistent with carpal tunnel syndrome. Review of systems reveals increased thirst and urination. His temperature is 37.0ºC (98.6ºF), blood pressure 190/90 mm Hg, pulse 73/min, and respirations 13/min. Physical examination shows frontal bossing, mandibular enlargement, and widening of the spaces between his teeth. His hands and feet seem larger and more out of proportion to his body size than usual. Which of the following is the most appropriate diagnostic study at this time? A. Cortisol level B. CT scan of brain C. Fasting glucose level D. Growth hormone levels E. Insulin-like growth factor

E. Insulin-like growth factor This patient has acromegaly, an excess of growth hormone, evidenced by increased hand and foot size and coarse facial features with a protruding jaw. Other features, such as tooth-space widening and macroglossia, can be present. Impaired glucose tolerance is often present, as is diabetes. Hypertension is present in 1/3 of these patients. Complaints of headache and visual field losses can occur. Patients may have carpal tunnel syndrome. The test of choice for diagnosis is insulin-like growth factor, which is elevated >5 times the normal limit. Another test which can be used for diagnosis is measurement of growth hormone after the administration of 75 g of glucose. In a normal patient, the growth hormone level would be suppressed, but in a patient with acromegaly, the levels remain high (greater than 5 ng/mL).

A patient with dysuria and suprapubic pain is treated with ciprofloxacin. What is the mechanism of action of this antibiotic? A. It inhibits dihydrofolate reductase B. It inhibits DNA-dependent RNA polymerase C. It inhibits protein synthesis by binding to the 30s ribosomal subunit D. It inhibits protein synthesis by binding to the 50s ribosomal subunit E. It inhibits topoisomerase II (DNA gyrase)

E. It inhibits topoisomerase II (DNA gyrase) Ciprofloxacin - fluoroquinolone

32. Patients with acute pancreatitis may go on to develop chronic pancreatitis. Clinicians should have a high index of suspicion for chronic pancreatitis when they are following patients who have been treated for acute pancreatitis. Signs and symptoms of chronic pancreatitis include all of the following EXCEPT: A. Abdominal pain B. Diarrhea after eating C. Weight loss D. Steatorrhea E. Jaundice

E. Jaundice

259. A patient with chronic liver disease is seen in the office. The patient is noted to have Kayser-Fleischer rings. Which of the following laboratory findings is most likely to be present? A. Alpha-1 antitrypsin deficiency B. Elevated hemoglobin A1c C. Positive antibodies to the glomerular basement membrane D. Howell-Jolly bodies on peripheral smear E. Low ceruloplasmin levels

E. Low ceruloplasmin levels

128. A patient with long-standing osteoarthritis is seen in the office with complaints related to his stomach. The patient only has pain control for his osteoarthritis by taking nonsteroidal anti-inflammatory drugs (NSAIDs), and he is not willing to stop these agents. Which of the following therapeutic regimens is specifically designed to lower his risk for the development of peptic ulcer disease in this scenario? A. Proton-pump inhibitors (PPIs) B. Antacids C. H2 blockers D. Sucralfate (Carafate) E. Misoprostol (Cytotec)

E. Misoprostol (Cytotec)

75. A newborn examination of an infant reveals a scaphoid abdomen and a palpable fullness of the epigastrium. An abdominal radiograph of the infant shows gaseous distention of the stomach and proximal duodenum. A nasogastric tube is placed and suction produces bilious fluid from the stomach. Which of the following prenatal studies might have revealed this abnormality? A. Alpha-fetoprotein level B. Antibody screens for common fetal infections C. Hemoglobin electrophoresis D. Human choriogonadotropin levels E. Ultrasonography

E. Ultrasonography

346. A 64-year-old man with a long history of "heart burn" comes to the emergency department with 6 episodes of hematemesis. He denies alcohol use, smoking, or drug use. He also denies significant nonsteroidal anti-inflammatory drug use. He has no known liver disease. His laboratory results are normal with the exception of hemoglobin 7.1 g/dL (normal male 13.8-17.2 g/dL). He is taken for an emergent esophageal gastroduodenoscopy, which reveals a gastric ulcer with a bleeding visible vessel. Compared with duodenal ulcers, gastric ulcers have which of the following? A. Better response to medication B. Less commonly associated with significant gastrointestinal bleeding C. Greater chance for development in those who drink coffee D. More common association with H. pylori E. More common association with malignancy

E. More common association with malignancy

242. A 4-month-old boy has gained only 10 ounces since birth. He has failed to gain weight with multiple different formula preparations. His stools have been loose and fatty. An older sister had similar symptoms and has been repeatedly hospitalized for failure to thrive and recurrent pulmonary infections. Which of the following is the most likely cause of this patient's gastrointestinal symptoms? A. Achlorhydria B. Bacterial overgrowth C. Colonic inertia D. Gastric hypersecretion E. Pancreatic exocrine insufficiency

E. Pancreatic exocrine insufficiency

211. A postoperative patient is seen in the hospital during medicine rounds. The patient is complaining of abdominal pain. Abdominal plain film reveals a uniform distribution of gas in the small bowel, colon, and rectum. Which of the following is the most likely diagnosis? A. Complete small bowel obstruction B. Incomplete small bowel obstruction C. Complete large bowel obstruction D. Incomplete large bowel obstruction E. Paralytic ileus

E. Paralytic ileus

294. Surgical therapy for duodenal ulcer can consist of a variety of different techniques. Which of the following surgical procedures has the lowest risk of complications while at the same time has the highest rate of recurrence for duodenal ulcer? A. Billroth I (gastroduodenostomy) B. Billroth II (gastrojejunostomy) C. Vagotomy and pylorroplasty D. Vagotomy and antrectomy E. Parietal cell vagotomy

E. Parietal cell vagotomy

A 63-year-old man who weighs 65 kg is in his second postoperative day after an abdominoperineal resection for cancer of the rectum. An indwelling Foley catheter was left in place after surgery. The nurses are concerned because, even though his vital signs have been stable, his urinary output in the past 2 hours has been zero. In the preceding 3 hours, they had collected 56 mL, 73 mL, and 61 mL, respectively. Which of the following is the most likely diagnosis? A. Acute renal failure B. Damage to the bladder during the operation C. Damage to the ureters during the operation D. Dehydration E. Plugged or kinked catheter

E. Plugged or kinked catheter In the resence of normal perfusion pressure, biological problems do not suddenly drive urinary output to zero - a change invariably caused by a mechanical problem. ARF is not usu characterized by 0 UO; B & C injuries would have been noticed sooner & Dehydration would show gradual decline in urine.

328. A 45-year-old man with alcoholic cirrhosis is bleeding from a duodenal ulcer. He has required 6 units of blood over the past 8 hours, and all conservative measures to stop the bleeding, including irrigation with cold saline, intravenous vasopressin, and endoscopic use of the laser have failed. He is being considered for surgical intervention. Laboratory studies done at the time of admission, when he had received only 1 unit of blood, showed a bilirubin 4.5 mg/dL (normal <1.9 mg/dL), prothrombin time 22 seconds (normal 11-15 seconds), and serum albumin 1.8 g/dL (normal 3.5-5.5 g/dL). He was mentally clear when he came in but has since developed encephalopathy and is comatose. Which of the following best describes his operative risk? A. Acceptable B. Amenable to improvement if he receives vitamin K C. Amenable to improvement if he is given albumin D. Prohibitive unless he is dialyzed to normalize his bilirubin E. Prohibitive regardless of attempts to improve his condition

E. Prohibitive regardless of attempts to improve his condition

. A patient is suspected of having cirrhosis. Which of the following laboratory findings would be an expected finding if this patient has long-standing cirrhosis? A. Hypernatremia B. Hyperalbuminemia C. Hyperglycemia D. Elevated testosterone level in male E. Prolonged prothrombin time (PT)

E. Prolonged prothrombin time (PT)

A 78-year-old woman with a history of renal calculi and recurrent urinary tract infections presents with fever, chills, leukocytosis, and cloudy urine that has a pH of 8.2. A urine culture grows a lactose-negative, urease-positive, gram-negative rod. Which of the following microorganisms is most likely responsible for her infection? A. Candida albicans B. Enterococcus faecalis C. Escherichia coli D. Klebsiella pneumoniae E. Proteus mirabilis

E. Proteus mirabilis Gram neg rod, family of Enterobacteriae; lactose-negative which increases pH. Its w/ stones @ increased risk for this b/c the org can "hide" in the stones

A 43-year-old man seeks help for erectile dysfunction. He gives a history of impotence of sudden onset, which was first manifested at a time of significant marital stress. There is no history of perineal trauma or pelvic surgery. Among the tests done he was found to have normal testosterone levels and presence of nocturnal penile tumescence (nocturnal erections during REM sleep). Which of the following is the most likely diagnosis? A. Hyperprolactinemia B. Impotence due to arterial insufficiency C. Impotence due to venous leak D. Neurogenic impotence E. Psychogenic impotence

E. Psychogenic impotence

A 28-year-old woman is diagnosed with lupus nephritis, World Health Organization (WHO) type IV. She has a malar rash, diffuse arthritis, and edema. Her blood pressure is 190/110 mm Hg. Her creatinine level is 2.1 mg/dL with blood urea nitrogen 28 mg/dL. Her urine reveals 25 red blood cells/hpf, and 3+ protein. One red blood cell cast is seen. A 24-hour urine collection reveals 11 grams of protein with a total creatinine amount of 1 gram. Which of the following would be the most appropriate management? A. Oral azathioprine (Imuran) B. Oral cyclophosphamide (Cytoxan) C. Oral gold D. Oral prednisone E. Pulse IV cyclophosphamide (Cytoxan)

E. Pulse IV cyclophosphamide (Cytoxan) Lupus Nephritis - best agent to tx this

359. A 1-month-old boy is brought to the emergency department by his mother, who states that he has been having forceful vomiting for the past several days. She states that he vomits every time she feeds him, and the situation seems to be getting worse, though he seems in no pain. She describes the vomitus as non-bilious and his stools as normal with no blood. On examination the infant appears to be mildly dehydrated, with soft abdomen and palpable 2-cm, firm, mobile mass in the right upper quadrant. The patient has been gaining weight appropriately since birth. Which of the following is the most likely diagnosis? A. Duodenal atresia B. Intussusception C. Hirschsprung's disease D. Midgut volvulus E. Pyloric stenosis

E. Pyloric stenosis

86. A 39-year-old woman comes to the health care provider with complaints of difficulty swallowing. She has a history of scleroderma for the past 15 years, during which time she has required a variety of medications to reduce the symptoms of heartburn. Over the past 6 months, she has also noted difficulty swallowing food, such as steak, and has felt as if food "sticks" in her lower chest. She is able to tolerate liquids without difficulty. She denies any weight loss. Which of the following is the underlying cause for these symptoms? A. Development of a squamous carcinoma in the upper third of the esophagus B. Recent return of peristaltic activity in the body of the esophagus C. Reverse peristalsis in the body of the esophagus D. Reverse peristalsis of the lower esophagus E. Scarring at the lower esophagus because of chronic acid reflux

E. Scarring at the lower esophagus because of chronic acid reflux

304. A 25-year-old man sustains multiple stab wounds to the abdomen when he is mugged while jogging. The assault takes place in the evening. He is dumped by the attackers behind thick vegetation and is not found until the next morning, at which time exploratory surgery reveals multiple small bowel and colonic lacerations. All of the lacerations are repaired. In the post-op period, the patient has persistent hypotension, even though he received adequate fluid infusion and his central venous pressure is 12 mm Hg. Further studies done with the help of a pulmonary artery catheter reveal high cardiac output and low peripheral resistance. Which of the following is the most likely diagnosis? A. Adrenal insufficiency B. Cardiogenic shock C. Hypovolemic shock D. Neurogenic shock E. Septic shock

E. Septic shock

A 50-year-old man with moderate familial hypertriglyceridemia is treated with gemfibrozil (Lopid). Which of the following is the primary mechanism of action of this drug? A. Binding of bile acids in the intestine B. Inhibition of hepatic VLDL secretion C. Inhibition of HMG-CoA reductase D. Stimulation of HDL production E. Stimulation of lipoprotein lipase

E. Stimulation of lipoprotein lipase Gemfibrozil (Lopid), as well as fenofibrate (Tricor), works by increasing the activity of lipoprotein lipase, leading to increased clearance of VLDLs, along with inhibiting the synthesis of VLDL carrier apoprotein B. This medication also inhibits peripheral lipolysis and decreases hepatic free fatty acid extraction. Increased triglycerides are elevated in familial hypertriglyceridemia.

A 47-year-old woman undergoes a minor elective operation and develops a cascade of complications. She has to be intubated after the procedure, and she is admitted to the intensive care unit. An older sister told the health care provider that the patient was normal when she last saw her before moving to a distant city, but that when she encountered her again several years later, the patient seemed to be "in slow motion" all the time. She spoke slowly and with a croaky voice, had gained weight, seemed to be mentally limited, and only talked about how cold she was and how ineffective the laxatives that she took were. Pursuing that information, stat T4 determination is made, and the report shows an extremely low value. At this time the patient most urgently needs which of the following therapies? A. ACTH B. Iodine C. Potassium iodide D. T3 E. T3 plus T4

E. T3 plus T4 The diagnosis is clear: the patient is profoundly hypothyroid, a condition that had not been suspected when she had the surgery. Hypothyroidism can develop insidiously. Patients who are severely hypothyroid indeed do very poorly when they have surgery. Replacement of T4 with IV levothyroxine will replace the missing thyroid hormone that this patient has as a result of her previously unrecognized and untreated hypothyroidism. This patient will also need corticosteroid coverage during this replacement in order to support the adrenal gland functioning when thyroid hormone replacement is given. The current recommendation is to provide both T4 and T3 intravenously as both hormones are needed in order to treat this life-threatening condition. IV T3 is more biologically active, which will produce a fast onset of action, but T4 is what is needed for long-term treatment.

207. A 4-month-old girl is brought to the office because of frequent crying, sleep disturbance, and decreased appetite. She has been irritable and crying a great deal since birth, especially after feeding when she might spit up and even vomit. She also seems to get hiccups frequently. She has not been gaining weight as expected and her parents are concerned that something may be wrong. Last night they thought she had stopped breathing for approximately 20 seconds. She has been on a diet of breast milk and some formula supplementation. On physical examination the infant seems in no acute distress. Her vital signs are within normal limits and her weight and length are in the tenth percentile. Otherwise the physical examination is within normal limits. Which of the following is the most appropriate next step in management? A. Inserting a nasogastric feeding tube B. Nissen fundoplication C. Prokinetics D. Proton pump inhibitors E. Thickening of feeds with cereal and elevating the head of the bed

E. Thickening of feeds with cereal and elevating the head of the bed

121. A 21-year-old college senior comes to the health care provider with a 2-month history of frequent episodes of loose stool, preceded by lower abdominal cramping. Over the past 4 weeks, the stools have become increasingly bloody. On a number of occasions he has had the sensation of rectal fullness but is unable to pass any fecal matter. A sigmoidoscopy reveals inflammation in a circumferential pattern from the anal verge to the mid-sigmoid colon, where a transition to normal mucosa is seen. Which of the following is the most appropriate treatment for this patient? A. IV hydrocortisone B. IV infliximab (Remicade) C. Oral azathioprine (Imuran) D. Oral prednisone E. Topical mesalamine (Asacol, Pentasa)

E. Topical mesalamine (Asacol, Pentasa)

A 34-year-old woman goes into labor in week 42 of gestation and delivers a girl who weighs 2,600 g and is 43 cm long. At 1 minute, the neonate is assessed to have a pale pink body with mottled and cool extremities. Her pulse is 95/min; there is a weak cough, generalized hypotonia with some flexion of the extremities, and a slow respiratory effort. Her skin is dry, with scarce lanugo. There is no significant change at her 5-minute assessment. Detailed physical examination reveals macroglossia with coarse facial features, enlarged fontanelles, and an umbilical hernia. Which of the following is the most appropriate screening test for this neonate? A. Antithyroid antibodies B. Perchlorate flush test (KClO2) C. Radioactive iodine uptake (132I) D. Thyroid binding globulin E. Total T4 and thyrotropin

E. Total T4 and thyrotropin This neonate has the typical clinical presentation of congenital hypothyroidism. The most appropriate screening tests to diagnose this disorder would be to measure total T4 and thyrotropin levels. Congenital hypothyroidism results from a deficiency of thyroid hormone. Congenital hypothyroidism occurs in approximately 1/4,000 births. Most cases occur because of thyroid dysgenesis. This is caused by either complete absence of or ectopic thyroid tissue. Congenital hypothyroidism is twice as common in females. Symptoms appear gradually. Prolonged jaundice may be the earliest sign. Poor feeding, somnolence, a large tongue, constipation, and an umbilical hernia may be present. The skin may be cold and mottled. The full clinical picture develops by 3-6 months. Growth and development are retarded. The fontanels stay widely open. Hair is coarse and brittle. There is generalized muscular hypotonia. Serum levels of total T4 are low, whereas thyrotropin (thyroid-stimulating hormone) levels are elevated. Newborn screening tests detect most cases of congenital hypothyroidism. Early diagnosis and treatment has greatly reduced the incidence of complications.

41. A 24-year-old man comes to the clinic for evaluation of a 4-month history of postprandial diarrhea, weight loss of 9 pounds, and lower abdominal pain. He denies recent travel or antibiotic use. On physical examination his temperature is 38.0ºC (100.4ºF), and he has several oral aphthous ulcers. On abdominal examination there is tenderness and mild voluntary guarding in the right lower quadrant. Rectal examination reveals brown stool that is strongly positive for occult blood. Which of the following is most likely causing this patient's symptoms? A. Gram-negative organism B. Folate deficiency C. Mucosal ulceration with no transmural involvement in the ascending colon D. Toxin-producing organism E. Transmural inflammation in the region of the terminal ileum

E. Transmural inflammation in the region of the terminal ileum

117. Which of the following esophageal disorders is most likely to be treated surgically rather than medically? A. Achalasia B. Corkscrew esophagus C. Plummer Vinson syndrome D. Type 1 esophageal hernia (sliding) E. Type 2 esophageal hernia (paraesophageal)

E. Type 2 esophageal hernia (paraesophageal)

A 55-year-old man has severe lower abdominal pain and suprapubic pressure. Earlier in the day, while undergoing coronary catheterization for unstable angina, he had transient hypotension and bradycardia that was successfully treated with atropine and epinephrine. He has not urinated since the procedure. The nurse notes that he is tachycardic, with a regular pulse of 110/min. A review of systems taken on admission reveals nocturia and a weak urinary stream. From a historical context, which of the following is the most likely etiology of his abdominal pain? A. Cholesterol emboli syndrome B. Contrast-induced acute renal failure C. Nephrolithiasis D. Pyelonephritis E. Urinary retention

E. Urinary retention

169. A 50-year-old chronic alcoholic presents with dementia, paralysis of lateral gaze, and difficulty walking. Which of the following vitamin deficiencies is most likely responsible for this clinical picture? A. Vitamin B12 B. Vitamin B6 C. Folate D. Vitamin D E. Vitamin B1 (thiamine)

E. Vitamin B1 (thiamine)

120. A 45-year-old woman has been experiencing generalized weakness and a sensation of "pins and needles" for the past 3 weeks. She exercises daily, rarely drinks alcohol, and is a strict vegetarian. Her temperature is 37ºC (98.6ºF), blood pressure 110/70 mm Hg, pulse 60/min, and respirations 18/min. Examination shows weakness of the proximal and distal muscles of the lower extremities. Deep tendon reflexes are increased. The gait is ataxic. Which of the following is the most likely cause of these symptoms? A. Guillain-Barré syndrome B. Lambert-Eaton syndrome C. Myasthenia gravis D. Polymyositis E. Vitamin B12 deficiency

E. Vitamin B12 deficiency

228. A 5-day-old full-term boy born at home is being evaluated in the urgent care clinic for bruising and gastrointestinal bleeding. Laboratory findings include: Partial thromboplastin time and prothrombin time >2 minutes (normal for newborns ≤60 seconds) Serum bilirubin 4.7 mg/dL (normal for newborns) Prothrombin time 15.2 seconds (normal 11-13 seconds) Alanine aminotransferase 18 mg/dL (normal <45 mg/dL) Platelet count 330,000/mm3 (normal 150,000-300,000/mm3) Hemoglobin 16.3 g/dL (normal 13.2-21.0 g/dL) The boy's mother has factor V Leiden deficiency. Which of the following is the most likely cause of the boy's bleeding? A. Factor VIII deficiency B. Factor IX deficiency C. Idiopathic thrombocytopenic purpura D. Liver disease E. Vitamin K deficiency

E. Vitamin K deficiency

12. A patient is seen in the office with concerns about her daughter just being diagnosed with acute hepatitis A (HAV). The mother and daughter live in the same household. When is hepatitis A immunoglobulin ideally given to the mother if this is indicated? A. It is not indicated B. Immediately, regardless of when she was exposed C. Within 48 hours of exposure D. Within 1 week of exposure E. Within 2 weeks of exposure

E. Within 2 weeks of exposure

Which of the following medications is the first-line treatment for primary biliary cholangitis? A. Ursodeoxycholic acid (Actigall) B. Clopidogrel (Plavix) C. Lactulose D. Pentoxifylline (Trental) E. Prednisone

The correct answer is A. Ursodeoxycholic acid improves biochemical and histology features of primary biliary cholangitis.

A 2-week-old infant is brought to the clinic because of vomiting and severe dehydration. The infant was born at home to a 23-year-old woman who did not receive any prenatal care. Physical examination shows dehydration and ambiguous genitalia. Laboratory studies show hyponatremia. Additional laboratory studies are most likely to reveal which of the following findings?

Elevated levels of serum potassium. This infant most likely has congenital adrenal hyperplasia caused by 21-hydroxylase deficiency. 21-Hydroxylase deficiency results in virilization and ambiguous genitalia in females. Patients may present with vomiting and hyponatremic dehydration with hypotension and salt-wasting with low aldosterone levels. Salt-wasting leads to low serum sodium with resultant elevation of serum potassium. Congenital adrenal hyperplasia may be suspected in female infants who are virilized at birth or shortly after birth. This condition may also be suspected in males who have virilization in childhood. This condition should be suspected in infants of either sex who have a salt-wasting crisis in the first 4 weeks of life. This diagnosis is confirmed by demonstrating an elevated serum concentration of 17-hydroxyprogesterone (precursor for corticosteroids). Serum androstenedione and progesterone levels are also increased on laboratory testing because stimulation of the gland which makes corticosteroids causes a large amount of sex hormones to be made when the adrenal gland is stimulated. Testosterone is increased in affected females and prepubertal males. This is an inherited condition in which there are inherited enzyme deficiencies that impair normal corticosteroid synthesis by the adrenal cortex. The most common presenting symptoms are related to insufficient aldosterone formation with subsequent vomiting and dehydration occurring early in infant life with the resultant risk for life-threatening adrenal crisis.

A 32-year-old woman with one child has tried unsuccessfully to conceive for the past 2 years. She tells her health care provider that she has had problems putting on weight and is often anxious and irritable. On physical examination her thyroid is enlarged with no palpable nodules. Which of the following laboratory findings would be most suggestive of a diagnosis of secondary hyperthyroidism?

Elevated thyroid-stimulating hormone (TSH). Secondary hyperthyroidism clinically presents in a similar fashion to primary hyperthyroidism. Secondary hyperthyroidism, however, is the result of a pituitary hyperfunction resulting in an excess production of thyroid-stimulating hormone (TSH) and a secondary elevation of thyroid hormones T4 and T3. Some of the symptoms of hyperthyroidism include nervousness, anxiety, heat intolerance, increased sweating, tachycardia, amenorrhea, and infertility.

A 45-year-old alcoholic man with cirrhosis is transferred to intensive care after developing esophageal varices complicated by shock. A screening battery of tests is ordered, revealing a total thyroxine (T4) of 3.8 mg/dL (normal 4.5-11.2 mg/dL). He appears seriously ill. Physical examination of the thyroid gland is unremarkable. Follow-up studies showed a total tri-iodothyronine (T3) of 30 ng/dL (normal 100-200 ng/dL) and TSH (third-generation test) of 0.7 mIU/mL (normal 0.4-4 mIU/mL). Which of the following is the most likely diagnosis?

Euthyroid Sick Syndrome. This patient is seriously ill, with low T4 and low T3 but normal TSH. This is typical for euthyroid sick syndrome, which occurs in many seriously ill patients who do not have clinical hypothyroidism. It can be enough of a diagnostic problem that some references suggest that thyroid hormones should not be measured while a person is in intensive care, unless clinical tests are suggestive of thyroid disease. TSH level is usually most helpful in distinguishing euthyroid sick syndrome from true hypothyroidism, as it is often above 30 mIU/mL in true hypothyroidism and may be below normal, normal, or minimally elevated in euthyroid sick syndrome. Disproportionately decreased T3 is also typical of euthyroid sick syndrome, and T4 may be normal or decreased. Euthyroid sick syndrome occurs in very ill patients and is characterized by low T4 and low T3 levels with a normal, mildly elevated, or mildly decreased TSH level. It can also manifest with a disproportionately decreased T3 and a T4 in the normal or decreased range. TSH level above 30 mIU/mL is consistent with true hypothyroidism, in contrast to the mild variations in TSH with euthyroid sick syndrome. Patients who have euthyroid sick syndrome actually have the correct amount of thyroid hormone in circulation (as evidenced by the normal TSH level) but because of a serious underlying illness, the conversion of T4 is to reverse T3 rather than free T3. Correction of the underlying condition will treat euthyroid sick syndrome, which is not truly a real thyroid disease.

A 35-year-old man has been complaining of headaches for the past 6 months and reporting changes in his vision. On examination there is loss of visual fields bilaterally. MRI of the brain shows a pituitary adenoma. Which of the following hormones is most likely to be elevated in this patient?

Fifty percent of pituitary adenomas have been found to secrete prolactin. Levels of this hormone must be measured in a patient suspected of having a pituitary tumor. Pituitary adenomas can cause the pituitary gland to either hyper- or hypofunction. Diagnosis of this misfiring of hormones is through the measurement of these hormonal levels. Prolactin hormone secretion abnormalities are the most common; luckily, of all the hormone abnormalities, prolactin has the least effect on life and mortality.

A 19-year-old woman with no significant past medical history presents for a pre-college physical examination required by her school. She has no complaints, and her review of systems is negative. Her temperature is 37.0ºC (98.6ºF), blood pressure 120/80 mm Hg, pulse 80/min, and respirations 10/min. Physical examination reveals a single nodule on the left lobe of the thyroid gland. Her thyroid-stimulating hormone is within normal limits. Which of the following is the most appropriate next step in management?

Fine-needle biopsy. The first test for a thyroid nodule is the thyroid-stimulating hormone test; if normal, the most appropriate next step is fine-needle aspiration, which can be done safely and without complication. An ultrasound can be used to differentiate between solid and cystic structure and can be used to guide the needle aspiration. The fine-needle biopsy provides a definitive diagnosis in most cases and is the most helpful test in guiding management. A patient who has a thyroid nodule should have a full evaluation performed. The first test performed should be the TSH; a suppressed level suggests hyperthyroidism and a hot nodule, which lessens the possibility for cancer. The next test should be a fine-needle biopsy (if the TSH is not suppressed). Histology will identify whether the patient has cancer and also will identify the type of cancer that the patient has. A thyroid scan can help to identify the nodule and whether it is hyper- or hypofunctioning.

A 4-week-old boy is brought to the emergency department with tachycardia, tachypnea, and poor weight gain. His arterial blood gas shows a pH of 7.34 (normal 7.35-7.45 in this age population), PaCO2 of 41 mm Hg (normal 27-44 mm Hg in neonatal period), and PaO2 of 74 mm Hg (normal 83-100 mm Hg in the neonatal period). Chest radiograph shows cardiomegaly. Echocardiography reveals a structurally normal heart, left ventricular dilatation, a left ventricular ejection fraction of 20%, and mild mitral and tricuspid regurgitation. Intravenous administration of which of the following medications is the best initial step in management?

Furosemide. This infant has acute congestive heart failure (CHF). CHF may be defined as inadequate contractile heart function for the specific hemodynamic needs. Clinical manifestations include respiratory distress, tachycardia and/or hyperdynamic precordium, and cardiac enlargement evidenced by echocardiogram. This patient is developing acute symptom manifestation as a result of the patient not being able to adequately meet the metabolic demands of his body. The differential diagnosis of CHF includes left-to-right shunt (ASD, VSD, PDA, atrioventricular canal, or AV fistula), left-sided obstruction leading to myocardial dysfunction (severe coarctation or AS), or intrinsic myocardial dysfunction (myocarditis, cardiomyopathy, or infarct caused by anomalous coronary artery). In acute CHF, treatment should begin immediately. IV furosemide is the drug of choice because its onset is very rapid. It can provide quick symptomatic relief and improve respiratory distress.

A 25-year-old woman presents with a 12-month history of palpitations, intermittent diarrhea, anxiety, and a 1-month history of bulging of both eyes. What is the most likely cause of her symptoms?

Graves disease is the most common cause of hyperthyroidism in a young female and is the only condition that causes exophthalmos (bulging of both eyes) or pretibial myxedema. Graves disease is an autoimmune disorder in which a thyroid-stimulating IgG immunoglobulin (TSI) binds to the thyroid-stimulating hormone (TSH) receptors and causes an increased release of thyroid hormone. The exophthalmos is caused by lymphocytic infiltration of the extraocular muscles. Graves disease, a common cause of hyperthyroidism, is caused by an autoimmune phenomenon wherein patients develop antibodies to the TSH receptor with resultant production and release of thyroid hormone. Of the various causes of hyperthyroidism, only Graves is manifested by exophthalmos and pretibial myxedema.

A 48-year-old type 2 diabetic woman on daily extended-release glipizide (Glucotrol) presents with complaints of polyuria and polydipsia. Laboratory evaluation reveals a blood glucose of 192 mg/dL (normal <100 mg/dL). She states that her diabetes had been well controlled and she had been symptom-free for the past 8 years. Recently, however, she began taking medication for hypertension. Which of the following antihypertensive drugs is she most likely taking?

HCTZ. The fact that the patient had well-controlled diabetes until the addition of an antihypertensive medication suggests that the new agent is responsible for increasing the blood glucose level. Hydrochlorothiazide is a thiazide diuretic known to increase fasting blood glucose in diabetics. Dosage adjustments of both oral hypoglycemic agents, like glipizide, and insulin may be required to maintain euglycemia. When these medications are used together, hyperglycemia may occur, because the HCTZ may decrease the activity of the oral sulfonylureas.

A 65-year-old woman with a long-standing disease has bone marrow fibrosis and increased bone remodeling, with bone resorption exceeding bone formation. She has a history of passing calcium-oxalate kidney stones. Which of the following laboratory result profiles would be expected in this patient's serum?

High parathyroid hormone, increased serum calcium, low serum phosphate. This patient has long-standing hyperparathyroidism (elevated PTH), which predisposes to the development of osteitis fibrosa, her bone disease. PTH acts initially on osteocytes of bone tissue (osteocytic osteolysis) and subsequently on osteoclasts (osteoclastic resorption) to resorb calcium from bone matrix and make it available to the circulation. This increases plasma calcium levels. PTH also causes decreased phosphate reabsorption in the proximal renal tubule, yielding hypophosphatemia. Hypercalciuria is another sequela of excess PTH production, which predisposes the patient to the formation of calcium oxalate stones.

An 8-year-old boy is brought to the emergency department with decreased mental status. His mother states that she has noticed he has been drinking and urinating more frequently over the past several weeks. He was hard to wake up this morning and complained of abdominal pain. Physical examination reveals an afebrile drowsy male with clear airways and mild tachycardia. Mucous membranes are dry and his lips are cracked. His abdomen is mildly tender to palpation diffusely, but there is no rebound or guarding. Laboratory evaluation reveals glucose of 560 mg/dL (normal 60-100 mg/dL in a child) and potassium 5.9 mEq/L (normal 3.5-5.3 mEq/L). Arterial blood gas analysis reveals pH 7.18 (normal 7.35-7.43 in a child). Urinalysis is positive for ketones and glucose. A CT scan of the abdomen is normal. A chest radiograph is clear. Two hours after initiation of treatment the health care provider adds potassium to the patient's intravenous fluids. Which of the following best explains the rationale used in making this therapeutic decision?

Hypokalemia will result as acidosis is corrected. Total body potassium is considerably depleted during diabetic ketoacidosis, even when the serum potassium is normal or elevated. This is caused by renal losses from osmotic diuresis caused by hyperglycemia. The intracellular potassium exchanges for extracellular hydrogen ions in an attempt to correct the acidosis. During correction of ketoacidosis, insulin decreases potassium levels as the potassium goes back into the cells. This results in a true potassium deficiency in the extracellular compartment, and potassium depletion will only be verified in the laboratory after the acidosis is corrected. Despite what the initial laboratory serum potassium level is, the patient will be potassium depleted and will need additional potassium supplementation.

A 33-year-old woman comes to the health care provider because of palpitations, restlessness, sweating, weight loss, and a tremor for the past 3 weeks. She does not drink coffee, tea, soda, or alcohol, and she does not smoke cigarettes. Her temperature is 37ºC (98.6ºF), blood pressure 130/80 mm Hg, and pulse 90/min. Examination shows a fine tremor, lid lag and stare, and pretibial myxedema. The thyroid gland is diffusely enlarged, asymmetric, and lobular. A bruit is present over the gland. Laboratory studies show an undetectable level of thyroid-stimulating hormone, an increased level of thyroid hormones, and an increased radioactive iodine uptake (RAIU). The diagnosis of Graves disease is made and treatment options are discussed. The patient selects radioactive iodine therapy. The patient is at greatest risk for which of the following long-term complications following treatment?

Hypothyroidism is the main complication of radioactive iodine therapy, affecting up to 70% of patients in 10 years. Radioactive iodine therapy is a safe and effective treatment for Graves disease, because it can provide the same ablative effects of surgery without the surgical complications. Radioactive iodine therapy is considered to be first-line therapy for the management of Graves disease. It is also used if antithyroid medications fail to treat Graves after a sufficient medication trial is performed. The goal of treatment is to ablate the thyroid tissue and to induce hypothyroidism, and then start lifetime replacement of thyroid hormone. . When radioactive thyroid treatment is given, radiation thyroiditis could occur, causing the release of thyroid hormone in 5 to 20 days; this would exacerbate symptoms temporarily. Because patients may have an exacerbation of symptoms (including ophthalmopathy), corticosteroids may be given to prevent exacerbation of this condition. Typically, 30 to 40 mg of prednisone is given to prevent this complication. Radioactive iodine is contraindicated for use in pregnancy and in lactating women. It is not associated with thyroid or other cancer.

A 39-year-old woman has unexplained fatigue, weight loss, nausea, and diffuse muscle pains over the last 2 months. She has a history of severe asthma for which she has been using an inhaled beta agonist, together with oral prednisone (10 mg/day) that was started earlier this year. Three months ago her breathing seemed to be getting a lot better, so she decided to stop the prednisone because she felt it was making her gain weight. Her temperature is 37.0 °C (98.6 °F), blood pressure is 95/40 mm Hg, pulse is 108/min, and respirations are 15/min. On physical examination she has normal skin. Her heart is tachycardic and regular without murmurs. Her lungs have a few scattered wheezes. Which of the following is the most likely underlying cause of this patient's condition

Iatrogenic. This patient has adrenal insufficiency. Clues to the diagnosis are nonspecific findings such as fatigue, weight loss, and muscle aches. Laboratory abnormalities such as hypoglycemia, hyponatremia, and hyperkalemia are seen. The most common reason for patients to have adrenal insufficiency is prolonged steroid use (iatrogenic). As little as a 2 weeks exposure to pharmacologic doses of glucocorticoids may lead to suppression of the CRH-ACTH-adrenal axis with effects lasting from weeks to months after withdrawal of treatment. The suppression can be so great that acute withdrawal or stress may prevent the axis from responding with sufficient cortisol production to prevent an acute adrenal crisis. This patient has a history of asthma that was controlled with prednisone. She gives a history of discontinuing the steroids before the onset of her symptoms which is a clue in this patient's diagnosis.

A 17-year-old girl is referred for evaluation because of persistent headaches for the past 3 months and diminished peripheral vision first noticed 2 weeks ago. Physical examination reveals that she is short for her age. She has fully developed breasts but no axillary or pubic hair. She started menstruating 2 years ago but has irregular periods. She has no libido. The absence of axillary and pubic hair in this girl is indicative of which of the following conditions?

Insufficient production of androgens. In both men and women, growth of axillary and pubic hair is stimulated by androgens. Androgens are also responsible for libido, which is absent in this girl. The rest of the presentation suggests the presence of a tumor in the pituitary area, from which some degree of hypopituitarism has ensued. This diagnosis can be confirmed by identifying low target hormones in the serum of the affected individuals due to the pituitary gland not making these tumors. The headaches are most likely caused by pituitary adenoma, which is the most common cause of hypopituitarism in the adult population.

A 45-year-old man with history of previously well-controlled hypertension presents to the office because colleagues at his office are telling him that his appearance is changing. The patient has recently noted that his old shoes felt too tight and he had to buy new, larger shoes. He also complains of shooting pains in his hands, which his company nurse told him were consistent with carpal tunnel syndrome. Review of systems reveals increased thirst and urination. His temperature is 37.0ºC (98.6ºF), blood pressure 190/90 mm Hg, pulse 73/min, and respirations 13/min. Physical examination shows frontal bossing, mandibular enlargement, and widening of the spaces between his teeth. His hands and feet seem larger and more out of proportion to his body size than usual. Which of the following is the most appropriate diagnostic study at this time?

Insulin-like growth factor. This patient has acromegaly, an excess of growth hormone, evidenced by increased hand and foot size and coarse facial features with a protruding jaw. Other features, such as tooth-space widening and macroglossia, can be present. Impaired glucose tolerance is often present, as is diabetes. Hypertension is present in 1/3 of these patients. Complaints of headache and visual field losses can occur. Patients may have carpal tunnel syndrome. The test of choice for diagnosis is insulin-like growth factor, which is elevated >5 times the normal limit. Another test which can be used for diagnosis is measurement of growth hormone after the administration of 75 g of glucose. In a normal patient, the growth hormone level would be suppressed, but in a patient with acromegaly, the levels remain high (greater than 5 ng/mL).

A 54-year-old man with chronic renal failure is seen in the renal clinic. He has renal failure caused by chronic glomerular disease. His blood pressure is well controlled, and he has no complaints. He has no edema. Physical examination is generally normal. Of note, laboratory analysis reveals a calcium of 7.9 mg/dL (normal 8.5-10.2 mg/dL), albumin 3.7 g/dL (normal 3.4-5.4 g/dL), phosphate 8.2 mg/dL (normal 3-4.5 mg/dL), parathyroid hormone 512 pg/mL, (normal 10-55 pg/mL), urea nitrogen 80 mg/dL (normal 7-20 mg/dL), and creatinine 7.2 mg/dL (normal 0.8-1.4 mg/dL in males). Which of the following is the most appropriate initial step in management?

Lower his phosphate with oral calcium given with meals. This patient has secondary hyperparathyroidism with a high parathyroid hormone (PTH) and a calcium x phosphate of about 64. The phosphate needs to be urgently lowered to avoid metastatic calcification. Oral calcium given with meals will bind the phosphates in the food, and lower the phosphate. As a result of the occurrence of renal osteodystrophy, patients with chronic kidney disease will develop an increase in serum phosphate and decrease in serum calcium as the kidney disease progresses. This loss of calcium leads to renal osteodystrophy with resultant secondary hyperparathyroidism as the body attempts to save calcium. Patients who have chronic kidney disease are treated with low phosphate diets, phosphate binders, and increased amounts of calcium and vitamin D supplementation. Phosphate binders are used if dietary supplementation fails.

Describe pathophysiology behind lung cancer and Cushings Syndrome

Lung cancer (small cell) can produce ACTH --> adrenal glands stimulated to make more cortisol --> elevated lvls of corisol = Cushing syndrome Underlying problem = increased cortisol from lung cancer tumor cells

A 15-year-old girl with type 1 diabetes mellitus is brought to her primary care provider for a routine check-up. Evaluation of her blood glucose chart indicates that her recorded blood glucose levels are routinely between 120 and 150 mg/dL before breakfast, dinner, and bedtime, with the normal being 116 mg/dL (normal levels 60-100 mg/dL in childhood). She is on NPH and regular insulin. She does well in school and plays on the school tennis team. Which of the following is the most appropriate step in management?

Make no changes and obtain a glycosylated Hb test. The patient has adequate glycemic control, though she may be experiencing higher levels with snacks. Compliance is often an issue in adolescents. Her adequate glucose levels are a bit suspicious and it is advisable to obtain a glycosylated hemoglobin level to ensure agreement between her reports and the hemoglobin A1C levels. The hemoglobin A1C level provides the average glucose level over the past 3 months and would give insight into the overall glycemic control in this patient. Patients with type 1 diabetes are typically diagnosed with the condition in childhood and adolescence. This translates into the patient having a long disease course, because it is hoped that diabetic patients achieve a normal lifespan with appropriate treatment. In today's treatment environment, control of diabetes means frequent blood glucose measurements (about five times per day) along with serial blood glucose administrations. Lispro insulin (Humalog) is a rapidly acting insulin that is given immediately before meals, with dosing adjusted according to the carbohydrate load of the meal. Glargine insulin is typically given once daily, because it is a peakless insulin with a 24-hour activity life. This long-acting insulin is used to control the basal production of insulin. NPH insulin is an intermediate-acting insulin with onset of activity within 1-2 hours and a peak at 4-14 hours. The total duration of NPH insulin is 10-24 hours.

A 59-year-old man complains of headache and decreasing visual ability. History reveals that the patient has also outgrown his pants and shirts within the past year. The man exhibits a marked overbite. Which of the following is the best method to diagnose this patient?

Measuring growth hormone (GH) and insulin-like growth factor-1 (IGF-1) levels before and after administrating glucose. Acromegaly is most commonly caused by a pituitary adenoma. Acromegalic persons secrete excessively high levels of growth hormone (GH) and insulin-like growth factor-1 (IGF-1) from pituitary gland tissue. These hormones are not suppressed by glucose, as they would be in a normal person. Acromegaly is the condition in which there is an increase in the release of growth hormone and insulin-like growth factor after closure of the growth plates. Insulin-like growth factor has a longer half-life in the circulation at 18-20 hours, so it is more predictable than just the measurement of growth hormone, which is released in a very cyclical pattern. A growth hormone measurement that is taken during an oral glucose tolerance test serves as the standard method to confirm the diagnosis of acromegaly. Glucose administration typically causes growth hormone to be suppressed in patients who do not have acromegaly, and the failure of this suppression is associated with the syndrome of acromegaly in a patient who has symptoms consistent with this condition.

Which of the following drugs used in the treatment of type 2 diabetes mellitus has no effect on the secretion of insulin?

Metformin is often used in conjunction with oral hypoglycemic agents for the treatment of type 2 diabetes. Its mechanism of action is twofold: (1) it decreases the production of glucose in the liver, and (2) it increases the uptake of glucose in the liver. Metformin also has a role in decreasing the intestinal glucose absorption. It does not cause the manufacturing or release of insulin, so there is no chance for hypoglycemia when it is used alone. It is one of the few diabetic medications that promote weight loss. It should not be used in patients who have impaired renal or liver function.

A 65-year-old woman with an 8-year history of type 2 diabetes dies in the hospital. She had no other significant medical history. Which of the following was the most likely cause of death?

Myocardial infarction is the leading cause of death in diabetics. The advanced glycosylated products associated with long-standing diabetes mellitus accelerate the atherosclerotic process. Other risk factors for coronary artery disease (CAD) include hypertension, smoking, hypercholesterolemia, family history of CAD at a young age, male sex, and being a postmenopausal female.

A 27-year-old woman complains that she is constantly thirsty and has to urinate every 2 hours. Her plasma osmolality is 295 mOsmol/kg (normal 280-300 mOsmol/kg), and her urine osmolality is 100 mOsmol/kg (normal 50-1,200 mOsmol/kg). Her urine is negative for glucose. As part of a diagnostic workup, the patient is deprived of fluids for 3 hours. Her urine osmolality remains 100 mOsmol/kg. One hour after injection of arginine vasopressin (AVP), her urine osmolality becomes 400 mOsmol/kg. Which of the following is the most likely diagnosis?

Neurogenic (central) diabetes insipidus. ADH (antidiuretic hormone, vasopressin) is normally released from the posterior pituitary in response to hypovolemia, high serum osmolality, and increased osmotic pressure. It acts on the collecting duct of the nephron, increasing its permeability to water. This concentrates the urine and conserves water. Diabetes insipidus (DI) is a condition in which ADH function is absent. Two forms of DI exist: neurogenic and nephrogenic. Neurogenic (central) DI is caused by the absence of ADH secretion from the posterior pituitary; thus, serum ADH levels are always low, even in states of serious volume depletion.

An infant comes to the office for his 1-year check-up. His father states that he is worried that his son is smaller than he should be. The child's weight is 8.6 kg (19 lb), and his length is 71 cm (28 in). He appears to be growing appropriately on his growth curve. Which of the following is the most appropriate explanation about growth to be given to the father?

Normal infant development based on birth weight (BW) and birth length (BL): Double BW = 6 months Triple BW = 12 months Quadruple BW = 24 months Double BL = 4 years Triple BL = early puberty

A 14-year-old girl is brought to the clinic by her parents because of a multitude of symptoms that have developed over the previous several months. They first noticed that she had become nervous and hyperactive, could not sit in one place for longer than a few minutes, and would burst into tears very easily. They thought she was going through a "phase" and initially ignored it, but then the teacher called to discuss her performance in school. She was always an excellent student but her grades have significantly dropped lately. She cannot seem to concentrate in class and her handwriting has deteriorated. In addition, she has trouble sleeping and keeps getting out of bed to eat all night. Despite that, she seems to have significantly lost weight and they wonder if she may be doing something they are not aware of. She dropped out of her gymnastics class because it had become too exhausting for her. Her medical history is unremarkable and she takes no medication that the parents are aware of. On physical examination, the patient seems to be uncomfortable and jittery. She keeps shifting her weight on the chair and complaining how hot it is in the office. Her temperature is 36.7ºC (98.0ºF), blood pressure 135/65 mm Hg, pulse 120/min, and respirations 18/min. Inspection of her scalp shows diffuse thinning and brittle hair. Her skin is smooth and clammy to the touch, whereas her palms are sweaty. Extension of both arms reveals a moderately severe tremor. Palpation of the neck confirms the presence of a diffuse, nontender, symmetric enlargement of the thyroid gland. Auscultation of the heart is remarkable for tachycardia. Results of serum hormone levels are shown below:

PTU. This patient has Graves disease resulting in hyperthyroidism. Hyperthyroidism results from overproduction of thyroid hormone. It almost always results from Graves disease, a diffuse toxic goiter. . Graves disease is associated with HLA-B8 and HLA-DR3. Patients may have emotional disturbances, motor hyperactivity, emotional lability, and deterioration in school performance. Tremors of the fingers may be noticed on extension of the arms. Voracious appetite without weight gain may be described. Exophthalmos results from binding of antibodies to extraocular muscles, producing a cytotoxic effect. Sweating and flushing are common. Tachycardia and palpitations are cardiac manifestations. Thyroid storm is a severe presentation that can lead to death. The diagnosis is confirmed by showing elevated T4, free T4, T3, and free T3, whereas thyroid-stimulating hormone (TSH) is low because of the negative feedback loop to the hypothalamus and pituitary gland. Thyroid receptor-stimulating antibodies are often present. Medical treatment consists of propylthiouracil (PTU) or methimazole (Tapazole). Propranolol (Inderal) is useful for reversing symptoms in severely toxic patients but does not treat the underlying disease. Surgery or radioablation with iodine are other options for treatment, with most experts recognizing radioactive iodine as first-line treatment.

A 58-year-old woman is found on a routine medical check-up to have a serum calcium level of 11.8 mg/dL, where the upper limit of normal is 9.5 mg/dL. Repeated determinations confirm values between 10.9 and 12.2 mg/dL, and she is found to have elevated concentrations of parathyroid hormone. She is asymptomatic, has no pertinent family history, and has no evidence of renal stones or bone disease. She is offered the option of elective parathyroidectomy, but she declines and elects to have close medical follow-up. In addition to monitoring serum calcium levels, which of the following therapeutics should be given?

Pamidronate (Bisphosphonate). The underlying pathology is most likely a single parathyroid adenoma, for which surgical removal is the only cure. In postmenopausal women, however, bisphosphonate therapy can be given to inhibit osteoclastic activity, which reduces bone resorption and turnover. Hydration is an essential treatment for this condition to help decrease serum calcium levels. Patients who have hypercalcemia caused by primary hyperparathyroidism need to have serum calcium levels monitored so that if they rise to 15 mg/dL or higher, intervention can take place. When patients have hyperparathyroidism and conservative therapy is elected, adequate hydration states are essential to maintain. Definitive therapy with surgery to remove the parathyroid hormone can be performed if medical therapy fails to control the hypercalcemia. In addition to hydration therapy, bisphosphonates may be used to inhibit osteoclastic activity and reduce bone resorption. Calcitonin may be used as adjunctive short-term therapy, but its effectiveness in hypercalcemia management in patients who have primary hyperparathyroidism is short-lived.

Which of the following patients with acute hepatitis A is at highest risk for the development of fulminant hepatic failure? A. Patients who already have chronic hepatitis C B. Patients who are noncompliant with therapy C. Patients who recently received the hepatitis A vaccine and then became exposed to hepatitis A D. Patients who have type 2 diabetes E. Patients who have chronic kidney disease before becoming infected

Patients who already have chronic hepatitis C

A 17-year-old girl comes to the health care provider because she has not yet had a menstrual period. She also complains of a lack of breast development. Past medical history is significant for loss of smell and color blindness. Past surgical history is significant for a cleft palate that was repaired in childhood. She takes no medications and has no allergies to medications. Examination is significant for absent breast development and a hypoestrogenic vulva and vagina. Urine hCG is negative. Which of the following is the most likely diagnosis?

Patients with Kallmann syndrome (i.e., isolated gonadotropin deficiency or familial hypogonadotropic hypogonadism) can present with primary amenorrhea. Primary amenorrhea is defined as the absence of menses in a female by the age of 16 years. Associated findings in Kallmann may include anosmia or hyposmia, color blindness, and cleft lip or cleft palate—all attributable to the fact that during embryogenesis, the GnRH neurons originally develop in the epithelium of the olfactory placode and normally migrate into the hypothalamus. Thus exists the link between the midline defects and the amenorrhea. Kallman syndrome is a cause of isolated hypogonadotropic hypogonadism. Its association with anosmia is what isolates this from the other causes of hypogonadism. Physical examination may reveal absent to minimal breast development. Treatment of the patient who has Kallmann syndrome is with exogenous estrogen and progestin replacement therapy. If pregnancy is desired, ovulation induction can be brought about with the pulsatile administration of exogenous GnRH.

A 55-year-old man with a 5-year history of type 2 diabetes mellitus is most likely to exhibit which of the following signs or symptoms?

Patients with type 2 diabetes typically have the disease for 5-7 years before the actual diagnosis is made. Thus a patient may have disease progression before the diagnosis is made. For this reason, patients diagnosed with type 2 diabetes should have immediate referral to ophthalmology for evaluation of retinopathy, which typically presents 5 years into the diagnosis. Because patients with type 1 diabetes typically present at the onset of the disease, ophthalmology referral is typically at 5 years into the disease process. Retinopathy is typically used as the marker for microvascular disease, because it occurs early in the disease course and is readily identifiable by an ophthalmologist. Tight blood glucose control has been shown to positively affect the development and progression of microvascular disease in diabetic patients. Diabetic retinopathy usually precedes the development of diabetic nephropathy and proteinuria.

A 66-year-old man with diabetes and generalized arteriosclerotic occlusive disease notices a gradual loss of erectile function over several years. Initially he could get an initial erection but it didn't last very long. Later he noticed a decrease in the quality of the erection. More recently he is, by his own criteria, completely impotent. He has occasional, brief nocturnal erections, but "he can never get an erection when he needs one." He is very frustrated. Which of the following is the most appropriate initial step in management?

Pharmacologic therapy. This patient has impotence, but it is not related to trauma for which surgical reconstruction would be indicated. Medical therapy can be tried as first step as an empiric trial. He is still able to obtain erections, which rules out total impotence. His remaining function can be augmented with one of the phosphodiesterase inhibitors such as sildenafil (Viagra), tadalafil (Cialis) or vardenafil (Levitra).

A 45-year-old man presents to the health care provider with muscle cramps, perioral numbness, and irritability over the past 3-4 months. Laboratory results reveal hypocalcemia, normal albumin level, mild hypomagnesemia, and hyperphosphatemia. Parathyroid hormone (PTH) level is decreased. Alkaline phosphatase level is normal. Which of the following is most likely the underlying cause of this clinical scenario?

Previous subtotal thyroidectomy. This patient is experiencing symptoms of hypocalcemia secondary to diminished parathyroid hormone (PTH) secretion. This must always be considered in a patient who undergoes total or subtotal thyroidectomy, because the parathyroids are nestled in the tissue surrounding the thyroid gland. Surgical attempts to leave portions of the parathyroids intact are sometimes unsuccessful. Other causes of decreased PTH include neck irradiation, autoimmune phenomena (polyglandular autoimmune syndromes), dysembryogenesis (as in DiGeorge syndrome), and heavy metal damage (Wilson disease, hemosiderosis, hemochromatosis).

A 52-year-old nurse seeks medical retirement because of a "heart condition." She complains of disabling attacks of tachycardia and palpitations. Physical examination and electrocardiogram studies confirm that her pulse is between 100 and 105/min at all times, and she has documented repeated episodes of paroxysmal atrial fibrillation. It is also noted that she is fidgety and constantly moving, and various examiners remark that she arrives for tests lightly dressed when it is rather cold outside. Thyroid function studies show elevated free thyroxine (T4) and undetectable levels of thyroid-stimulating hormone (TSH). Her thyroid gland is not clinically enlarged or tender. Which of the following is the most appropriate next step in diagnosis?

RAIU. It has been established clinically and by the laboratory that this patient is hyperthyroid, and the thyroid gland is not abnormal on physical examination. Circumstances suggest that the potential for self-administration of thyroid hormone for secondary gain (e.g., weight loss) is a possibility. Radioactive iodine uptake should be high if her gland is indeed hyperfunctioning, but it will be near zero if it is suppressed by the exogenous hormone. The initial diagnostic study performed for someone suspected of having hyperthyroidism is a radioactive iodine uptake study. Patients who have Graves disease will have an increased uptake on the radioactive iodine uptake because the gland itself is hyperfunctioning. Patients who have thyroiditis and those who are factitiously taking thyroid hormone will have a normal percentage seen on radioactive iodine uptake scanning, because the gland itself is not overactive (it is secreting stored hormone in patients who have thyroiditis and does not need to function in patients who are exogenously taking thyroid hormone). Thyroid scans are preferentially used in the assessment of thyroid nodules to ascertain whether they are hypo- or hyperfunctioning. If a patient who has Graves disease has a thyroid scan performed, diffuse uptake will be seen, whereas patients who have multinodular goiter will have patchy uptake seen on this study.

A 44-year-old woman has a palpable nodule in the right lobe of her thyroid gland. The nodule measures 2 cm and is firm. The rest of the thyroid gland cannot be felt and is not tender. She also describes losing weight in spite of a ravenous appetite, along with palpitations and heat intolerance. She is thin, fidgety, and constantly moving. Examination also shows the patient to have moist skin and a pulse of 105/min. She has no exophthalmos or pretibial edema. Her TSH is reported as much lower than normal, and she has elevated levels of free T4. Which of the following is the most appropriate next step in diagnosis?

Radionuclide thyroid scan. There is no question, both clinically and by laboratory testing, that the patient is hyperthyroid. She has no clinical signs of acute thyroiditis, because her thyroid gland is not tender and she has no neck pain. She has none of the other findings seen in Graves disease; however, she has a thyroid nodule, which raises the possibility of a hyperfunctioning adenoma (a "hot" adenoma). If indeed she does, the scan will show that the nodule traps all the iodine, with suppression of the rest of the gland. The typical first step in the evaluation of thyroid nodules is thyroid scan. If a nodule is found to be cold, diagnostic FNA is done. Cold nodules may be consistent with cancer or are more likely previously hot, hyperfunctioning thyroid nodules that have subsequently burned out to become cold (hypofunctioning) thyroid nodules.

A 31-year-old woman comes into the office because she has not had a menstrual period for 7 months. She previously had normal cycles. She states that over the past year she has felt increasingly weak and tired. She always feels cold and her hair has been thinning over the course of the year. She also complains of constipation, weight gain, and depression. Her temperature is 36.7º C (98º F), blood pressure 100/60 mm Hg, pulse 56/min, and respirations 10/min. Physical examination is significant for brittle hair and delayed deep tendon reflexes. Urine human chorionic gonadotropin (hCG) is negative. Thyroid-stimulating hormone (TSH) is 20 µU/mL (normal 0.4-4.0). Prolactin is normal. Which of the following is the most likely cause of this patient's amenorrhea?

Secondary amenorrhea is defined as the absence of menses for 6 cycle intervals or 12 months in a woman who previously had regular cycles. This patient, therefore, has secondary amenorrhea. She also has a constellation of signs and symptoms that are highly suggestive of hypothyroidism. Patients who have hypothyroidism often complain of some combination of weakness, fatigue, cold intolerance, constipation, weight gain, depression, or thinning of the hair. Physical examination can reveal bradycardia and low blood pressure. Laboratory evaluation often shows an elevated TSH as the pituitary attempts to stimulate the underfunctioning thyroid. Many patients who have hypothyroidism will be asymptomatic, however, and the thyroid abnormality is found by thyroid function tests. Hypothyroidism likely leads to amenorrhea through changes in GnRH production. Treatment with thyroid replacement will often return these patients to regular menses.

Eight hours after completion of a trans-sphenoidal hypophysectomy for a prolactinoma, a 24-year-old woman becomes lethargic, confused, and eventually comatose. The anesthetic and the operation had proceeded without any apparent problems. Except for her mental status and dry mucosal membranes, the physical examination is unremarkable and her pupils are of equal size and reactive to light. Review of the record shows that she has not been given narcotics, and the only medications received are her intravenous fluids, 5% dextrose in half normal saline at 100 mL per hour. Her blood pressure is 98/72 mm Hg, with a pulse of 94/min, and her urinary output has averaged 600 mL per hour since the operation. Which of the following studies is most likely to establish the diagnosis?

Serum sodium concentration. The large urinary output at a time when she has a modest intravenous fluid intake suggests that she has developed diabetes insipidus—something that could happen with surgery in the area of the pituitary gland. The large water loss leads to rapidly developing hypernatremia, resulting in CNS symptomatology. Patients who have diabetes insipidus will have an elevated serum osmolality when compared to the urine osmolality.

A 60-year-old woman complains of a 1-year history of bone pain, anorexia, weight loss, and constipation. Serum calcium is elevated and she is diagnosed with primary hyperparathyroidism. She is treated surgically, with removal of her parathyroid adenoma. She then has a long period of hypocalcemia and continued treatment with vitamin D and calcium. After 6 months, she becomes normocalcemic and no longer needs therapy. Which of the following situations would explain these events?

She previously had severe bone disease. The patient initially had osteitis fibrosa cystica as a result of the primary hyperparathyroidism. When the adenoma was removed and PTH level decreased, the skeleton underwent rapid remineralization. This created an increase in calcium requirement. Once the repair was complete, the calcium demand decreased. Primary hyperparathyroidism, a central problem resting in the parathyroid gland, is associated with several problems because of the long-standing hypercalcemia that results from this condition: osteitis fibrosa cystica, renal calculi, and osteoporosis. When the hyperparathyroidism is corrected surgically, the patient needs additional calcium supplementation, as the bone that was demineralized starts to absorb too much calcium, magnesium, and phosphate. This condition, known as "hungry bone syndrome," frequently occurs after parathyroidectomy and corrects itself when the bone is fully mineralized. After bone mineralization, the patient should be put on appropriate calcium and vitamin D supplementation as part of their overall health maintenance against osteoporosis.

A series of photographs taken of a middle-aged man over a period of 2 decades demonstrates gradual coarsening of facial features and progressive protrusion of the brows. Upon questioning, the patient reports having to wear larger shoes than he did as a young man. Which of the following pair of hormones normally regulates the hormone responsible for these changes?

Somatostatin and GHRH. The disease is acromegaly, which is typically produced by a growth hormone-secreting pituitary adenoma. Growth hormone synthesis is predominately regulated by hypothalamic GHRH (growth hormone releasing hormone), and its pulsatile secretion is predominately regulated by hypothalamic somatostatin. Acromegaly occurs as a result of an increase in the production and release of growth hormone. Acromegaly is caused by a pituitary adenoma the vast majority of the time. The insulin-like growth factor-1 levels are elevated in this condition; this is the test that is primarily ordered to identify acromegaly, because this substance has an extremely long half-life in the circulation (18-20 hours). Somatostatin analogs such as octreotide (Sandostatin) are used if surgery fails or cannot be done, as these substances act like somatostatin, which inhibits multiple hormones, including growth hormone, glucagon, insulin, LH, and VIP.

A 50-year-old man with moderate familial hypertriglyceridemia is treated with gemfibrozil (Lopid). Which of the following is the primary mechanism of action of this drug?

Stimulation of lipoprotein lipase. Gemfibrozil (Lopid), as well as fenofibrate (Tricor), works by increasing the activity of lipoprotein lipase, leading to increased clearance of VLDLs, along with inhibiting the synthesis of VLDL carrier apoprotein B. This medication also inhibits peripheral lipolysis and decreases hepatic free fatty acid extraction. Increased triglycerides are elevated in familial hypertriglyceridemia. Binding of bile acids is the mechanism of action of RESINS such as cholestyramine (Questran). They cause the liver to use cholesterol for the synthesis of new bile acids. Inhibition of hepatic VLDL secretion is the mechanism of action of niacin. Inhibition of HMG-CoA reductase is the mechanism of action of lovastatin (Mevacor), simvastatin (Zocor), atorvastatin (Lipitor), rosuvastatin (Crestor), and pravastatin (Pravacor). Stimulation of HDL production may occur with both gemfibrozil and niacin, but it is not the main mechanism of action.

During a routine pediatric examination, a 12-year-old boy is noted to be in growth arrest. The child is obese and has multiple small bruises on his arms and legs. He has difficulty rising from a crouching position. Measurements of blood pressure demonstrate hypertension when compared with age-based standards. This patient most likely has which of the following disorders?

The child has features of Cushing syndrome, which can be caused by excess corticotropin (ACTH), a pituitary adenoma, or ectopic ACTH production. This condition can also occur independently of ACTH production from such causes as adrenal adenoma, adrenal carcinoma, alcohol, or exogenous steroids. The presentation of Cushing syndrome includes truncal obesity, moon facies, and buffalo hump. These features cannot always be clearly distinguished from ordinary obesity, however, particularly in populations such as children. It is consequently of great help to be aware of other features that may suggest the diagnosis. Children with Cushing syndrome almost inevitably have growth arrest, which may, as in this patient, be the initial diagnostic clue that more than simple obesity is present. Some children also show precocious puberty, secondary to androgen excess. Features present in both children and adults that are particularly helpful in discriminating Cushing syndrome from obesity include easy bruising and a mild proximal myopathy that can be demonstrated by asking the patient to rise from a crouching position. Hypertension is another prominent feature, which tends to be more common in patients who have Cushing syndrome than in those who are obese. A predisposition to infections can be another helpful clue. Other features that can be present (usually in adult cases) include psychiatric abnormalities, most commonly depression and lethargy; osteoporotic vertebral collapse leading to lost height, seen in long-standing Cushing syndrome; red-purple striae of the abdomen or thighs; plethoric appearance secondary to skin thinning; and sometimes skin pigmentation in ectopic ACTH syndrome and with some pituitary tumors. Once Cushing syndrome is suspected, it can be investigated with a variety of endocrine techniques, including urinary free cortisol, dexamethasone suppression tests, and plasma ACTH, and it can be treated surgically if an adrenal or pituitary tumor is found. Cushing syndrome results from any cause of excess corticosteroids in the body, whereas Cushing disease is specifically a cause of excess corticosteroid production in a patient caused by pituitary adenoma and no other etiology.

Dark urine and pale or clay-colored stools occur in which of the following settings? A. Elevated conjugated bilirubin B. Elevated unconjugated bilirubin C. Esophageal variceal bleeding D. Hepatic adenoma E. Cavernous hemangioma

The correct answer is A. Elevated conjugated bilirubin results in dark urine and pale-colored stools, as the conjugated bilirubin cannot get into the gastrointestinal tract, most commonly because of obstruction.

A 33-year-old man complains that his chest hurts when he eats, especially when he eats meat. A radiograph shows a dilated esophagus, and achalasia is suspected. Esophageal manometry is used to confirm the diagnosis. Swallowing induced relaxation is reduced at which anatomic location in this man? A. Lower esophageal sphincter (LES) B. Proximal esophagus C. Middle esophagus D. Pharynx E. Upper esophageal sphincter (UES)

The correct answer is A. Achalasia is a disorder of esophageal motility that affects the LES and lower 2/3 of the esophageal body. The LES remains tonically contracted and does not relax as food moves down the esophagus. Food therefore cannot move easily from the esophagus into the stomach. The esophagus proximal to this contracted area becomes greatly dilated. Patients who have achalasia most commonly complain of dysphagia (difficulty swallowing), chest pain, and regurgitation. Since this is a motor disorder, dysphagia occurs for both solids and liquids. Clinical Pearls Achalasia is a condition in which there is failure of the lower 2/3 of the esophagus to relax along with poor peristaltic stripping waves along the body of the esophagus. When food is swallowed, the LES does not contract, trapping food in the lower esophagus and which then dilates proximal to the area of increased contraction. This results in dysphagia for both solids and liquids (consistent with a motor disorder), regurgitation, and retrosternal chest pain.

A patient is seen in the office with complaints related to severe diarrhea. The patient has a high fever in addition to the diarrhea. The patient has just returned from a trip to the Bahamas. Which of the following should be avoided as part of the treatment regimen? A. Loperamide (Imodium) B. Gatorade mixed with water C. Ciprofloxacin (Cipro) D. Potassium, as indicated E. Rifaximin

The correct answer is A. Antidiarrheal agents such as loperamide need to be avoided in the setting of traveler's diarrhea unless it is mild diarrhea without fever.

A patient is seen in the office complaining of pain in the epigastrium, radiating to the back. The pain is described as steady and dull but also severe. Lying down and eating meals make the pain worse. The patient is anorexic at present but did have a great deal of nausea and vomiting prior to this visit. Examination reveals abdominal distention along with epigastric tenderness. Physical examination reveals bilateral flank ecchymosis. What is this physical finding known as? A. Grey Turner sign B. Cullen sign C. Fox sign D. Murphy sign E. Obturator sign

The correct answer is A. Bilateral flank bruising is known as the Grey Turner sign, which occurs as a result of the patient laying supine in the setting of hemorrhagic pancreatitis.

A patient is admitted to the hospital with signs and symptoms consistent with biliary colic. Ultrasound fails to identify any stones in the gallbladder. Hepatobiliary iminodiacetic acid (HIDA) scan and IV injection of cholecystokinin fail to show any contractility of the gallbladder. Which of the following is the cause for this presentation? A. Biliary dyskinesia B. Bile duct stricture C. Choledochal cysts D. Primary biliary cholangitis E. Primary sclerosing cholangitis

The correct answer is A. Biliary dyskinesia has impaired ejection fraction of the gallbladder after injection of cholecystokinin, which is listed as gallbladder ejection fraction on the HIDA scan with cholecystokinin challenge.

Black pigmented gallstones are associated with which of the following conditions? A. Hemolysis B. Infection C. Obesity D. Ileal resection E. Crohn disease

The correct answer is A. Black pigmented stones seen with chronic hemolytic conditions such as sickle cell disease.

A patient is admitted to the hospital with severe left lower quadrant pain. The patient is suspected of having acute diverticulitis. Which of the following diagnostic evaluations should be performed for this patient? A. CT scan with oral and IV contrast B. CT scan without contrast C. MRI D. Barium enema E. Colonoscopy

The correct answer is A. CT with oral and IV contrast is the test of choice in the evaluation of a patient with suspected acute diverticulitis, as it has a high sensitivity for identifying this condition and can also identify other causes to explain the patient's symptoms.

Which of the following has been shown to decrease the risk for colorectal cancer formation? A. Calcium supplementation B. Treatment of hypertension with beta-blockers C. Treatment of clinical depression D. Supplementation with vitamin A E. Low-salt diet

The correct answer is A. Calcium supplementation and, possibly, daily aspirin therapy can decrease the risk for colon cancer.

A patient with long-standing inflammatory bowel disease is diagnosed with primary sclerosing cholangitis. Ongoing symptoms occur with this condition. Which of the following treatments is indicated for managing the pruritus that occurs with this condition? A. Cholestyramine (Questran) B. Supplemental vitamin D and calcium C. Lactulose D. Antihistamines such as diphenhydramine (Benadryl) E. Glucocorticoids such as prednisone

The correct answer is A. Cholestyramine helps the itching that is seen with primary sclerosing cholangitis.

A 52-year-old man with peptic ulcer disease has been on drug therapy for 3 months and has noticed changes in his bowel habits, increasing headaches, dizziness, skin rashes, loss of libido, and gynecomastia. Which of the following drugs is most likely responsible for these side effects? A. Cimetidine (Tagamet) B. Famotidine (Pepcid) C. Metronidazole (Flagyl) D. Omeprazole (Prilosec) E. Sucralfate (Carafate)

The correct answer is A. Cimetidine (Tagamet), an H2-receptor antagonist, can produce all the side effects exhibited when taken in high doses over a long period of time. In addition, cimetidine can alter the hepatic metabolism of several drugs and can elevate liver transaminases. It is the oldest member of this class of medications and due to its potential for side effects, this medication is not used as much as the other drugs in this class. Clinical Pearls Cimetidine (Tagamet), an H2 blocker, selectively antagonizes histamine H2 receptors and is used in the management of peptic ulcer disease. It has more side effects than any of the agents in this class. It activates the CYP-450 system and elevates liver transaminase levels. It is associated with headache, gynecomastia, drowsiness, and rash.

A patient has long-standing Crohn disease. As the disease progresses, several complications can occur. Which of the following complications is most likely to occur with this condition? A. Cholesterol gallstones B. Toxic megacolon C. Eosinophilic colitis D. Sarcoidosis E. Polymyalgia rheumatica

The correct answer is A. Crohn disease can be complicated by cholesterol gallstones because of bile salt malabsorption. Other complications that can occur with Crohn disease include calcium oxalate kidney stones and amyloidosis.

Which of the following treatments is recommended for patients with peptic stricture? A. Esophageal dilation B. Fluticasone inhaler used without a spacer C. Botulism toxin injection D. Long-term proton-pump inhibitor (PPI) therapy E. Long-term H2 blockers

The correct answer is A. Esophageal dilation is helpful in the management of esophageal strictures and achalasia, but it is not helpful in the management of erosive esophagitis, which is more likely to occur in the setting of a lax lower esophageal sphincter rather than a tight one.

Which of the following electrolyte abnormalities is most commonly seen with squamous cell carcinoma of the esophagus? A. Hypercalcemia B. Hypocalcemia C. Hyperkalemia D. Hypokalemia

The correct answer is A. Hypercalcemia occurs in the setting of adenocarcinoma of the esophagus because of production of parathyroid hormone-related peptide hypersecretion as part of this paraneoplastic syndrome.

Certain markers of increased mortality are seen in the setting of acute pancreatitis. Which of the following laboratory results is most closely associated with an increase in the mortality rate when they findings occur with acute pancreatitis? A. Hypoalbuminemia B. Elevated triglyceride level C. Hypocalcemia D. Hyperglycemia E. Elevated serum lipase level

The correct answer is A. Hypoalbuminemia and increased serum lactic dehydrogenase (LDH) levels seen in the setting of acute pancreatitis are associated with increased mortality. Low albumin levels reflect poor overall health status, and increased LDH is associated with increased tissue destruction.

A patient is seen in the office with anorexia, nausea, vomiting, fever, jaundice, and tender hepatomegaly. The patient has laboratory testing revealing aspartate aminotransferase (AST) elevated more than twice the alanine aminotransferase (ALT) level. Which of the following is the most likely diagnosis? A. Alcoholic hepatitis B. Non-alcoholic fatty liver disease (NAFLD) C. Autoimmune hepatitis D. Primary sclerosing cholangitis E. Alcohol-related fatty liver disease

The correct answer is A. In alcoholic hepatitis, the AST level exceeds the ALT level by at least a factor of 2. This is a classic laboratory result.

A patient is admitted to the hospital with abdominal complaints. The patient undergoes a barium enema as part of the evaluation, and the findings on this study reveal "thumbprinting" of the colonic wall. Which of the following is the most likely diagnosis? A. Acute mesenteric ischemia B. Acute diverticulosis C. Acute colonic perforation D. Acute bowel obstruction E. Intussusception

The correct answer is A. Mesenteric ischemia causes thumbprinting on barium enema from the edematous mucosal folds.

A patient is admitted to the hospital with bleeding caused by arteriovenous malformation. Which of the following cardiac abnormalities is associated with gastrointestinal (GI) bleeding due to an arteriovenous (AV) malformation? A. Aortic stenosis B. Aortic regurgitation C. Mitral stenosis D. Mitral regurgitation E. Mitral valve prolapse

The correct answer is A. Of people with bleeding AV malformations, 25% will have aortic stenosis. The association between chronic GI bleeding due to angiodysplasia and calcific aortic stenosis has been termed Heyde syndrome.

Which of the following patients with acute hepatitis A is at highest risk for the development of fulminant hepatic failure? A. Patients who already have chronic hepatitis C B. Patients who are noncompliant with therapy C. Patients who recently received the hepatitis A vaccine and then became exposed to hepatitis A D. Patients who have type 2 diabetes E. Patients who have chronic kidney disease before becoming infected

The correct answer is A. Patients with chronic liver disease, especially those with hepatitis B or C, have increased risk for fulminant hepatic failure.

In addition to gastric acid, which of the following substances is most commonly linked with the development of peptic ulcer disease? A. Pepsin B. Acetylcholine C. Bicarbonate D. Prostaglandin E2 E. Serotonin

The correct answer is A. Pepsin and gastric acid are aggressive causes of peptic ulcer disease.

A patient was recently hospitalized for sepsis and had multiple antibiotic regimens during his hospitalization. The patient read an article regarding pseudomembranous colitis and its high incidence for patients in the hospital and for patients who have taken antibiotics. Which of the following signs or symptoms would be consistent with pseudomembranous colitis? A. Profuse watery diarrhea B. Bloody diarrhea C. Diarrhea with a large amount of mucus D. Black diarrhea E. Green diarrhea

The correct answer is A. Profuse watery diarrhea is consistent with Clostridium difficile colitis. This diarrhea is often foul-smelling, but it is typically not associated with bleeding.

A 9-month-old girl is brought to the clinic by her parents because she has an abdominal mass. Physical examination shows an umbilical defect about 1 cm in diameter, with a small bulge when the girl cries. The hernial contents can be easily reduced. The hernia is not painful and the girl is otherwise asymptomatic. She is eating well and reaching all developmental milestones. Which of the following is the most appropriate next step in management? A. No therapy unless the hernia persists beyond the age of 2 years B. Repeated injections of sclerosing agents C. Elective laparoscopic surgical repair D. Elective open surgical repair E. Urgent surgical repair

The correct answer is A. Small umbilical hernias can close spontaneously up to the age of 2. If they are asymptomatic and pose no immediate risk for strangulation, they should be left alone Clinical Pearls Umbilical hernias occur as a result of the umbilical ring failing to close. This defect allows protrusion of the peritoneal sac, which could contain intra-abdominal contents such as the omentum or bowel. Umbilical hernias are the only type of hernia by which spontaneous healing occurs. Most umbilical hernias will close on their own by age 4-5, often correcting themselves as infants develop abdominal musculature by crawling and walking. If the umbilical hernia fails to resolve, elective surgical correction may be considered after age 5 (due to higher risk for incarceration if the hernia persists later in life).

A patient is seen with complaints of abdominal pain radiating straight to the back. She is taking a lot of medications, which may be the precipitating cause for these symptoms. Which of the following medications is known to cause this acute condition? A. Sulfonamides B. Isoniazid (INH) C. Meloxicam (Mobic) D. Omeprazole (Prilosec) E. Ranitidine (Zantac)

The correct answer is A. Sulfonamides, estrogen, tetracycline, valproic acid, and 5-ASA can cause acute pancreatitis with long-term use.

A 40-year-old obese white woman, mother of 5 children, gives a history of repeated episodes of right upper quadrant abdominal pain. The pain is triggered with the ingestion of fatty foods and relieved with anticholinergic medications. The pain is spasmodic, radiates to the right shoulder and around to the back, and is accompanied by nausea and occasional vomiting. The patient is in no pain at the moment but is anxious to avoid further episodes. She is afebrile, and physical examination is unremarkable. Which of the following is the most appropriate next step in this patient's evaluation? A. Sonogram of the biliary tract and gallbladder B. Upper gastrointestinal series with barium C. Antibiotics, intravenous fluids, and nothing by mouth D. Endoscopic retrograde cholangiopancreatogram (ERCP) E. Laparoscopic cholecystectomy

The correct answer is A. The clinical description is classic for biliary colic caused by gallstones that are intermittently impacted at the cystic duct. The diagnostic study of choice to confirm the presence of gallstones is a sonogram. Confirmation that stones are present is needed prior to surgical cholecystectomy unless the patient has a gangrenous gallbladder. The clinical picture is typical for gallbladder stones. Follow the 4 F's in cases of a colicky abdominal pain exacerbated by fatty meals: Fat Forty Fertile Female The diagnostic study of choice is an upper-abdominal ultrasound. Diagnosis has to be made prior to the patient being taken to the operating room for laparoscopic cholecystectomy.

A patient with complaints related to the gastrointestinal tract is seen in the office. The patient has questions about a proposed test that another health care provider discussed during the last office visit. Upon checking the electronic medical record, you note that a referral to a gastroenterologist is being considered in order to perform an endoscopic secretin stimulation test. What is primarily being assessed by this diagnostic study? A. Exocrine function of the pancreas B. Endocrine function of the pancreas C. Possibility for pancreatic cancer D. Function of the gallbladder to contract when stimulated E. Whether cystic duct obstruction is present

The correct answer is A. The endoscopic secretin test is performed in order to assess the exocrine function of the pancreas. This test assesses whether there are sufficient functional pancreatic cells able to produce bicarbonate, which demonstrates exocrine functioning of this gland.

A patient in the ICU is evaluated for abdominal pain and is diagnosed with acalculous cholecystitis. Which of the following treatments should be pursued? A. Cholecystectomy B. Replacement of thyroid hormone C. Broad spectrum antibiotics D. Endoscopic retrograde cholangiopancreatography (ERCP) E. Magnetic resonance cholangiopancreatography (MRCP)

The correct answer is A. The risk for development of acalculous cholecystitis is significant illness, and emergency cholecystectomy or percutaneous drainage of the gallbladder is recommended for treatment of these patients. These patients have a high mortality so prompt intervention is needed.

A patient is seen in the office complaining of pain in the epigastrium, radiating to the back. The pain is described as steady and dull but also severe. Lying down and eating meals make the pain worse. The patient is anorexic at present but did have a great deal of nausea and vomiting prior to this. This is his third episode this year. Which of the following abdominal film findings would be consistent with this diagnosis? A. Presence of sentinel loop B. Gastric air bubble displaced laterally C. Free air under the diaphragm D. Air-fluid levels of the large bowel E. Calcifications at the stomach pylorus

The correct answer is A. The sentinel loop sign (air filled bowel in left lower quadrant) is seen with acute pancreatitis.

Which of the following is known to precipitate hepatic encephalopathy in the setting of liver failure? A. Gastrointestinal bleeding B. Diarrhea C. Lactulose D. High-carbohydrate meal E. Asthma flare

The correct answer is A. The typical precipitants of hepatic encephalopathy in the setting of a patient with underlying liver disease include gastrointestinal bleeding, azotemia, constipation, high-protein meals, hypokalemic alkalosis, CNS depressant medications, hypoxia, hypercarbia, and sepsis.

A 43-year-old man reports a 9-pound weight loss over the past 9 months, accompanied by difficulty swallowing solids and liquids. He has woken on several occasions at 4 AM and regurgitated partially digested dinner contents. An upper gastrointestinal series reveals a widely dilated esophagus with a smoothly tapering distal esophagus. There appears to be partially digested food present in the esophagus. Which of the following is the most likely diagnosis? A. Achalasia B. Diffuse esophageal spasm C. Esophageal squamous cancer D. Peptic stricture E. Scleroderma

The correct answer is A. This patient has the symptoms of a motility-type dysphagia in that he has difficulty with both solids and liquids from the onset of his symptoms. The nocturnal aspiration of food occurs because the esophagus remains filled for hours or even days after completing a meal. The radiograph reveals the typical dilated esophagus of achalasia, which is termed a bird's-beak esophagus, with distal esophageal tapering. Clinical Pearls Achalasia is a motor disorder affecting the esophagus. There are two simultaneous problems that occur with this disorder: failure of the lower esophageal sphincter to relax and poor peristalsis in the distal third of the esophagus. The end result of this condition is that food is retained in the lower esophagus with dilation of the distal esophagus and poor transfer of food into the stomach. Patients present with simultaneous dysphagia for solids and liquids along with regurgitation and retrosternal chest pain. Definitive therapy is surgery with pneumatic dilation versus laparoscopic cardiomyotomy. Conservative therapy may also include nitrates or calcium-channel blockers that help to relax the lower esophageal sphincter, which helps to empty the food from the esophagus.

A 36-year-old schoolteacher comes to the clinic for a new-patient evaluation. She teaches second grade at a local school and just recently moved to the area. She has no current health complaints but reports a history of hepatitis. She apparently contracted "chronic hepatitis" from a blood transfusion received shortly after the birth of her first child 20 years ago, although she is unsure of the details of the disease. She was told she was infected, but she has always felt well and she doubts the diagnosis. Since then she has not suffered any symptomatic liver disease, and her liver function tests have always been normal. Review of symptoms fails to reveal any evidence of liver disease, and physical examination is unremarkable. A review of medical records that she has brought with her reveals the following: Hepatitis A Ab - absent, Hepatitis B surface Ag - absent, Hepatitis B surface Ab - positive, Hepatitis C Ab - positive In addition to disease education and counseling, which of the following is an appropriate intervention for this patient at this time? A. Hepatitis A vaccination B. Hepatitis B vaccination C. Liver biopsy for risk stratification D. Referral for interferon/ribavirin treatment E. Right upper abdominal ultrasound

The correct answer is A. This patient is extremely likely to have chronic hepatitis C but is currently asymptomatic. Though asymptomatic patients without any elevation of liver enzymes do not require treatment, they do need protection against other forms of hepatitis; that is because, if infected, they are at a much higher risk for fulminant liver disease. Once the HCV antibody is positive on the test, clinicians should order HCV RNA, which will tell the clinician whether the patient has chronic hepatitis C. If the patient has chronic hepatitis C, genotype testing and assessment for liver damage is done along with discussion as to whether curative therapy for hepatitis C should be pursued. If the patient has chronic hepatitis C, immunizations against hepatitis B and hepatitis A should be done if the patient does not have antibodies against these hepatitis viruses. The current recommendations for hepatitis A vaccination are as follows: Clinical Pearls Hepatitis C infection usually occurs from parenteral drug use but could also be acquired as a result of a blood transfusion given many years ago, before the mandatory testing of blood products against hepatitis C. Almost 85% of patients who are infected with hepatitis C will develop a chronic carrier state without spontaneously clearing the virus. The majority of hepatitis C patients who live in the United States have genotype 1. Modern therapy has revolutionized the treatment for this type of chronic hepatitis C infection. Patients who have hepatitis B or C should have vaccination against all of the hepatitis viruses (if an immunization is possible) to prevent fulminate liver disease in case the patient gets infected with one of the hepatitis viruses.

A 77-year-old woman presents to the emergency department with 12 hours of passing bright red blood from her rectum in increasingly large amounts. She becomes dizzy on standing and appears pale. Her blood pressure is 112/60 mm Hg on laying supine and 90/56 mm Hg on sitting upright with legs dangling over the side. Abdominal examination is normal. Which of the following is the most likely diagnosis? A. Bleeding diverticulum in the appendix B. Communication between an arteriole and venule in the cecum C. Laceration at the gastroesophageal junction D. Rectal polyp E. Perforation of a sigmoid diverticulum

The correct answer is B. A communication between an arteriole and venule in the cecum is a description of a vascular ectasia, also known as an arteriovenous (AV) malformation. This is a common cause of painless colonic bleeding in the elderly and may present with acute gastrointestinal bleeding (as in this case), chronic gastrointestinal bleeding, or iron-deficiency anemia. These lesions may be difficult to demonstrate, as the bleeding may be intermittent or the colon may be so full of blood that the site of origin is obscured. Techniques used to demonstrate bleeding AV malformations include colonoscopy, intraoperative endoscopy, and visceral angiography. Treatment of these lesions is problematic, because many patients will subsequently develop new or recurrent bleeding vessels. Clinical Pearls AV malformation or angiodysplasia usually occurs in patients older than age 65 years. It produces intermittent, mild, or severe episodes of painless hematochezia. Significant bleeding may cause orthostatic hypotension, tachycardia, and low blood pressure. Diagnosis may be made via colonoscopy, which identifies these malformations. Treatment during colonoscopy can be with bicap therapy or injection of the vessel with a sclerosing agent.

A patient is seen in the office with gastrointestinal complaints. Which of the following is the main differentiating symptom between achalasia and esophageal cancer? A. Esophageal cancer alone causes weight loss. B. Achalasia causes dysphagia for solids and liquids from the onset. C. Only achalasia is associated with regurgitation. D. Only achalasia is associated with Schatzki ring. E. Only achalasia occurs with weight loss.

The correct answer is B. Achalasia has dysphagia for solids and liquids because it is a motor problem, whereas esophageal cancer patients have progressive worsening of solid food dysphagia. For these patients, dysphagia for liquids is a very late finding. Clinical Pearls Achalasia occurs from an inability of the lower esophageal sphincter to relax. Achalasia causes a motor dysfunction in the esophagus and neither solids nor liquids can pass through this small opening. Esophageal cancer causes an obstruction in the lumen of the esophagus leading to dysphagia for solids long before dysphagia for liquids. The first consideration for unintentional weight loss in an older individual is esophageal cancer.

Which of the following is the most likely cell type for stomach carcinoma? A. Transitional cell B. Adenocarcinoma C. Squamous cell D. Epithelial cell

The correct answer is B. Adenocarcinoma is the cell type for the development of gastric carcinoma, which usually is focal and commonly occurs in the antrum or lesser curvature of the stomach.

For patients who are diagnosed with gastric carcinoma, which of the following cell types is most likely? A. Transitional cell B. Adenocarcinoma C. Squamous cell D. Large cell E. Lymphoma

The correct answer is B. Adenocarcinoma is the most common cell type of gastric cancer. Adenocarcinoma is also the most common type of colon cancer.

A patient with long-standing Crohn disease has recurrent flares that do not respond to medical management. The patient ultimately has an ileal resection, which results in bile salt malabsorption. Which of the following clinical manifestations is consistent with this bile salt deficiency state? A. Flushing due to serotonin loss B. Diarrhea C. Heartburn D. Melena E. Fecal incontinence

The correct answer is B. Bile salt deficiency from ileal resection results in diarrhea due to the loss of surface area.

Which of the following tumor markers is typically followed in the setting of pancreatic cancer? A. Carcinoembryonic antigen (CEA) B. CA 19-9 C. Thyroglobulin D. Alpha fetoprotein (AFP) E. Beta human chorionic gonadotropin (HCG)

The correct answer is B. CA 19-9 is tumor marker for assessing for the return of pancreatic cancer. The role of tumor markers in the setting of cancer is not for screening; rather, they are best utilized to assess for the return (recurrence) of that particular cancer.

A patient is seen in the office complaining of pain in the epigastrium, radiating to the back. The pain is described as steady and dull but also very severe in nature. Lying down and eating meals make the pain worse. The patient is anorexic at present but did have a great deal of nausea and vomiting prior to this visit. This is his third episode this year. Abdominal film reveals pancreatic calcifications. Which of the following conditions would the patient also be expected to have in this scenario? A. Diabetes insipidus B. Steatorrhea C. Pyloric obstruction D. Peptic ulceration E. Biliary dyskinesia

The correct answer is B. Chronic pancreatitis is associated with steatorrhea, diabetes mellitus, and pancreatic calcifications.

A 65-year-old man develops truncal obesity and a buffalo hump. He takes no medications. Early morning cortisol is 35 mcg/dL (normal .025-0.6 mcg/dL). Evening salivary cortisol is 4 mcg/dL (normal 0.01-0.09 mcg/dL). Cortisol production does not suppress with either low or high dexamethasone. CT scan of the head demonstrates a normal sella turcica and pituitary gland. An ectopic source of ACTH production is suspected. A malignancy of which of the following organs is most likely to be the source of the ACTH? A. Colon B. Lung C. Pancreas D. Stomach E. Testis

The correct answer is B. Cushing syndrome is a cluster of clinical abnormalities seen in the setting of chronic exposure to high cortisol levels. Truncal obesity, buffalo hump, increased susceptibility to infection, hypertension, and glucose intolerance are common manifestations. Cushing syndrome can be caused by administration of exogenous corticosteroids, adrenal hyperplasia or tumor, pituitary adenoma, or ectopic ACTH production by another malignancy. The latter is most commonly a small cell carcinoma of the lung.

A patient with a history of chronic intake of alcohol and smoking cigarettes is seen in the office. The patient would like to have an esophagogastroduodenoscopy done because his uncle was just diagnosed with esophageal cancer. Which of the following signs or symptoms is considered to be the earliest complaint/finding in a patient with esophageal cancer? A. Dysphagia for liquids B. Dysphagia for solids C. Jaundice D. Early satiety E. Odynophagia

The correct answer is B. Dysphagia for solids and weight loss are the earliest findings with esophageal cancer.

A patient is admitted to the hospital for elective cholecystectomy because she was found to have a porcelain gallbladder. For which of the following reasons is elective cholecystectomy performed? A. Increased risk of cholangitis B. Increased risk of gallbladder cancer C. Increased risk of sclerosing cholangitis D. Increased risk of pancreatitis E. Increased risk of hepatic abscess

The correct answer is B. Elective cholecystectomy is pursued in setting of porcelain gallbladder because 50% of patients go on to develop gallbladder cancer.

Ten days after having an exploratory laparotomy for blunt abdominal trauma, a 53-year-old man develops daily fever spikes to 38.9ºC (102.0ºF) and 39.5ºC (103.1ºF), as well as leukocytosis. At the time of surgery he was found to have a seat-belt injury to the small bowel and required resection and re-anastomosis of the affected area. He then had an uneventful postoperative course until the fever became apparent on day 10 of postop. He has been tolerating oral intake since day 5 of postop, and at this time his abdominal examination is unremarkable except for the fresh midline wound. Rectal examination is also unremarkable. The abnormality responsible for the fever will most likely be demonstrated by which of the following studies? A. Chest radiograph B. CT scan of the abdomen C. Plain films of the abdomen D. Repeated blood cultures E. Upper and lower gastrointestinal radiographs with water-soluble contrast

The correct answer is B. Fever that starts about 10 days after a contaminated abdominal surgical procedure is most likely caused by a deep abscess, either pelvic or subphrenic. The former has been ruled less likely by the patient having a normal rectal examination; the latter can best be demonstrated by CT scan. Clinical Pearls Several causes of postoperative fever are well known. These causes are somewhat time-dependent, with early causes being atelectasis, wound infections, urinary tract infections, pulmonary emboli, and pneumonia. If a fever is suspected 10 days postoperative following an abdominal surgery, peritoneal abscess should be the primary consideration.

Which of the following diagnostic studies is considered to be the gold standard for identifying gastroesophageal reflux disease (GERD)? A. Esophagogastroduodenoscopy (EGD) B. 24-hour pH monitoring C. Manometry D. Upper gastrointestinal (GI) series E. Capsule endoscopy

The correct answer is B. For GERD, 24-hour pH monitoring is the most sensitive and specific test, as it is able to capture the various associated acidic changes that occur in the esophagus.

A patient undergoes a colonoscopy, revealing multiple polyps. This patient also has multiple sebaceous cysts, multiple benign soft tissue tumors, and desmoid tumors. Which of the following is the most likely diagnosis? A. Familial adenomatous polyposis B. Gardner syndrome C. Turcot syndrome D. Peutz-Jeghers syndrome E. Hereditary nonpolyposis colorectal cancer

The correct answer is B. Gardner syndrome has multiple colonic polyps, sebaceous cysts, multiple benign soft tissue tumors, dental abnormalities, and desmoid tumors. This condition is one of the familial adenomatous polyposis conditions that encompasses these additional extracolonic manifestations.

Which of the following hereditable gastrointestinal (GI) polyposis syndromes has associated osteomas, fibromas, lipomas, and epidermal cysts? A. Familial adenomatous polyposis B. Gardner syndrome C. Lynch syndrome (nonpolyposis syndrome) D. Peutz-Jeghers syndrome E. Juvenile polyposis

The correct answer is B. Gardner syndrome is a variant of familial polyposis coli that is associated with soft tissue tumors including epidermoid cysts, osteomas, lipomas, fibromas, and desmoid tumors.

A symptomatic patient has just been diagnosed with autoimmune hepatitis. Which of the following is the initial treatment of choice? A. Azathioprine (Imuran) B. Glucocorticoids C. Pegylated interferon D. Ribavirin E. Cyclosporine

The correct answer is B. Glucocorticoids are the mainstay treatment for autoimmune hepatitis.

Infection with Helicobacter pylori is associated with all of the following clinical entities EXCEPT: A. Chronic gastritis B. Esophageal cancer C. Atrophic gastritis D. Gastric cancer E. Peptic ulcer disease

The correct answer is B. H. pylori colonizes gastric antral mucosa, produces urease, and causes active gastritis and atrophic gastritis. This environment lessens gastric acid output, which lowers the risk for esophageal cancer since refluxed material will be less acidic and less likely to lead to Barrett esophagus with changes from squamous to columnar cells.

Which of the following types of hepatitis is most likely to be spread via perinatal methods if no clinical intervention occurs? A. HAV B. HBV C. HCV D. HDV E. HEV

The correct answer is B. Hepatitis B is most likely to cause disease perinatally if the mother and/or baby is not treated with prophylaxis. In fact, women who are hepatitis B positive who deliver a baby are almost certain to spread hepatitis B to their infant if the infant is not treated within 12 hours of birth. Infants who acquire hepatitis B in this manner are most likely to become chronic carriers.

Hepatitis exists in various forms, each with certain associated pathophysiologic characteristics. Which of the following types of hepatitis is associated with immune-complex phenomena leading to arthritis, glomerulonephritis, and vasculitis? A. HAV B. HBV C. HCV D. HDV E. HEV

The correct answer is B. Hepatitis B may be associated with immune complex conditions leading to arthritis and glomerulonephritis.

Which of the following treatments is first-line therapy for a patient with gastric complaints due to Zollinger-Ellison (Z-E) syndrome? A. Histamine-2 blockers B. Proton-pump inhibitors (PPIs) C. Sucralfate D. Misoprostol E. Sodium bicarbonate

The correct answer is B. High-dose proton-pump inhibitors (PPIs) are the treatment of choice for Z-E syndrome in order to lower maximum gastric ulcer output.

A patient is admitted to the hospital with suspected appendicitis. You are performing the medical clearance for surgery because the patient has long-standing hypertension and diabetes. As part of your patient education, you tell the patient that the most common cause of acute appendicitis in the United States is which of the following? A. Fecalith B. Hyperplasia of lymphoid tissue C. Parasite D. Carcinoid tumor E. Klatskin tumor

The correct answer is B. Hyperplasia of lymphoid tissue accounts for 60% of acute appendicitis cases, as it leads to obstruction of the lumen of the appendix.

Patients with acute pancreatitis need appropriate management for improved outcomes. Which of the following clinical interventions is most important in the management of a patient with acute pancreatitis? A. IV antibiotics B. IV fluid resuscitation C. Bowel rest via nasogastric (NG) tube suctioning D. Narcotic pain medications E. IV somatostatin therapy

The correct answer is B. IV fluid resuscitation is the most important element of treatment for a patient with acute pancreatitis. These patients have third space fluid losses, necessitating fluid resuscitation to prevent hypovolemia and hypotension.

A patient with cirrhosis has acute esophageal variceal bleeding. In addition to endoscopic therapy for these varices, which of the following medications is first-line therapy in this setting? A. IV vasopressin B. IV octreotide C. IV metoprolol D. IV diltiazem E. IV furosemide (Lasix)

The correct answer is B. IV octreotide is best at lowering portal pressures with bleeding esophageal varices.

A patient has bleeding due to acute diverticulosis. Medical management may be tried to stop this bleeding prior to the patient needing to have surgical resection. Which of the following medications can be utilized in order to curtail this bleeding prior to surgery? A. Dicyclomine (Bentyl) B. Vasopressin C. Tetracycline D. Growth hormone E. Papaverine

The correct answer is B. IV vasopressin is given as a temporizing measure for acute diverticular bleeding and also helps to localize the lesion. This treatment improves mortality prior to the patient having surgical resection of the bleeding site.

Patients with ulcerative colitis and Crohn disease may receive azathioprine (Imuran) or 6-mercaptopurine for which of the following reasons? A. To heal rectosigmoid disease B. To allow the dosage of steroids to be lowered C. To allow better penetration of metronidazole D. To prevent extension of the disease E. To lower the risk for subsequent colorectal cancer

The correct answer is B. Immunosuppressants used for ulcerative colitis and Crohn disease are steroid-sparing agents. These agents help to potentiate the other agents, making them more effective.

Which of the following viruses produces disease or sequelae that is/are more severe if the infection occurs at a very young age? A. Epstein-Barr virus B. Hepatitis B virus C. Measles virus D. Poliovirus E. Varicella zoster virus

The correct answer is B. Infection with hepatitis B virus (HBV) at birth or a very young age is associated with chronic HBV infection and the development of hepatocellular carcinoma later in life. In fact, infants born to hepatitis B surface antigen (HBsAg)-positive mothers are commonly infected, and approximately 90% become chronic carriers of the virus. In chronic carriers, hepatocellular carcinoma develops at an incidence more than 200 times higher than in noncarriers. The current recommendation for infants born to HBsAg-positive mothers is administration of hepatitis B immunoglobulin (HBIg) in the delivery room, with the first dose of the hepatitis B vaccine given at the same time or within 12 hours of birth. The second and third doses of the vaccine are then given at 1 and 6 months. With this protocol, 94% protection is achieved. Clinical Pearls The risk for becoming a chronic carrier (failing to completely clear the hepatitis B virus) is increased in patients who acquire the disease early in life. This inability to clear the hepatitis B virus from the body is one reason that newborn vaccination is performed within 12 hours of birth in infants born to mothers who are known to be positive for hepatitis B infection.

A male patient has a sudden onset of fretfulness and pain. He curls up with his legs located in a fetal position. Over the next few hours he alternates between episodes of pain/crying with tears and acting normally. The patient's mother fears something is terribly wrong and brings him to the emergency department. His past medical history is unremarkable. The previous week he had had a cold with a runny nose. Stools had been normal that day but at the hospital he has a semi-soft stool with some blood and mucus. On examination the patient is quiet. The abdomen is surprisingly soft and normal. This classic presentation of intussusception is most likely to occur in which of the following age groups? A. Birth to 4 weeks of age B. 3 to 12 months of age C. 3 to 5 years of age D. Early adolescence E. Late adolescence

The correct answer is B. Intussusception usually occurs between ages 3 and 12 months, with the peak incidence at 6 months of age. It characteristically presents with episodes of distress and crying interspersed with quiet periods of normal behavior and playing. More than 50% of children will pass stool mixed with mucus and blood, with a red brick color classically known as the "currant jelly" stool. Usually the abdomen is soft and nontender, but with advanced intussusception, there may be signs of peritonitis. Sometimes a sausage-like mass may be palpable in the upper abdomen. A barium enema may be performed for the diagnosis and treatment of the intussusception. Clinical Pearls Intussusception is a condition in the abdomen in which there is a prolapse of one part of the intestine into the lumen of an immediately distal adjoining part of the intestine. The ileocecal region is the most commonly affected portion of the intestine, but it could affect any portion of the colon. This condition may lead to bowel wall edema from venous obstruction and it can progress to bowel necrosis if left untreated. The most commonly affected population is at 3-12 months of age, at which time it is typically an idiopathic phenomenon. When intussusception occurs in an older individual, it is typically secondary to a pathologic lead point, such as polyp, lymphoma, cancer, or Meckel diverticulum. The geriatric population may have intussusception at a point where there is a polyp or in patients with a malignancy or immunocompromised state.

A football player is tackled and swallows a great deal of dirt that was kicked up into his face. The player gets up and starts retching and vomiting. He is found to have some flecks of blood in the vomitus. There is no chest pain and no subcutaneous emphysema is noted. Which of the following is the most likely diagnosis? A. Esophageal spasm B. Mallory-Weiss syndrome C. Boerhaave syndrome D. Esophageal varices E. Peptic ulcer bleeding

The correct answer is B. Mallory-Weiss syndrome is a submucosal tear that occurs as a result of forceful vomiting. This tear is located at the gastroesophageal junction and is typically linear. This bleeding is typically small in amount and self-limited, so there is no need for treatment.

Which of the following medications is typically used to treat hepatic encephalopathy? A. Cholestyramine (Questran) B. Neomycin C. Furosemide (Lasix) D. Activated protein C E. Pentoxifylline (Trental)

The correct answer is B. Neomycin and metronidazole are poorly absorbed antibiotics that decrease the number of gut bacteria and therefore lower protein formation, which helps with hepatic encephalopathy. Rifaximin may also be used in this setting as another way of lowering the amount of gut bacteria so that less protein is produced.

Which of the subtypes of hepatitis is a DNA rather than an RNA virus? A. HAV B. HBV C. HCV D. HDV E. HEV

The correct answer is B. Only hepatitis B is a DNA type of virus.

Colic symptoms are not the same for gallbladder colic and ureteral colic. Signs and symptoms of biliary colic include all of the following EXCEPT: A. Right upper quadrant pain that occurs suddenly B. Intermittent right upper quadrant pain C. Pain lasting several hours D. Abdominal pain that may radiate into the right scapula or back E. Pain occurring 30 to 90 minutes following meal

The correct answer is B. Unlike ureteral colic, biliary colic pain is a constant pain lasting several hours rather than intermittent abdominal pain, which is what happens with ureteral colic caused by a renal stone.

Patients with chronic pancreatitis are at increased risk for the development of deficiency for which of the following? A. Glucose B. Vitamin B12 C. Vitamin B1 (thiamine) D. Vitamin B6 E. Vitamin C

The correct answer is B. Vitamin B12 is low in the setting of chronic malabsorption with chronic pancreatitis. Patients with chronic pancreatitis and an exocrine pancreatic deficiency state will have a reduction in intestinal pH, which interferes with vitamin B12 binding with intrinsic factor, leading to vitamin B12 deficiency.

A 24-year-old student complains of midepigastric pain that she describes as a "dull ache" that is relieved by eating. She has awakened from sleep on several occasions at 2 AM because of severe exacerbation of these symptoms, which are relieved with magnesium hydroxide. She takes frequent acetaminophen for menstrual cramping. Which of the following is the most likely cause of her symptoms? A. Autonomous gastrin secretion B. Gram-negative organism C. Gram-positive organism D. Prostaglandin inhibition E. Vagal inhibition

The correct answer is B. Peptic ulcer disease, particularly duodenal ulceration, is strongly suggested by chronic midepigastric pain that is severe enough to awaken a patient at night and is relieved by antacids such as magnesium hydroxide. Although the differential diagnosis listed in textbooks for peptic ulcer disease is long, most patients who have ulcer symptoms are either taking nonsteroidal anti-inflammatory agents (NSAIDs) or are colonized by the gram-negative organism Helicobacter pylori. This patient is not taking NSAIDs, so H. pylori colonization is the most likely answer. This organism colonizes the mucous layer that lines the stomach and disrupts the integrity of the mucus, predisposing for both chronic gastritis and peptic ulcer disease. Clinical Pearls The majority of duodenal ulcers are caused by H. pylori, which is a gram-negative organism. When ulceration occurs in this setting, the patient should have eradication of this organism performed. Various treatment modalities are used for eradicating this organism, with the common eradication regimens including three or four drug regimens.

A patient is admitted due to lower gastrointestinal bleeding caused by angiodysplasia of the colon (arteriovenous [AV] malformations). If the patient does not respond to colonoscopic coagulation of the lesion, which of the following clinical interventions is indicated? A. Coiling of the involved vessel B. Right hemicolectomy C. Left hemicolectomy D. Diverting colostomy E. Abdominal perineal resection

The correct answer is B. Right hemicolectomy is done for patients with AV malformation bleeding not responding to colonoscopic coagulation of the lesion. Surgery is the definitive therapy for angiodysplasia that does not respond to endoscopic treatment.

A 45-year-old chronic alcoholic presented to the emergency department 5 years ago with 24 hours of epigastric pain radiating to his back, nausea, and vomiting. He gradually recovered from that acute episode. Over the next 5 years, he is repeatedly admitted for similar symptoms. He then presents with gradual onset of weight loss, midabdominal pain radiating to his back, and steatorrhea. Which of the following conditions has most likely occurred? A. Cholangiocarcinoma B. Exocrine insufficiency of the pancreas C. Gastric outlet obstruction D. Pancreatic adenocarcinoma E. Scarring of the entire length of the common bile duct

The correct answer is B. Severe epigastric pain radiating to the back and accompanied by nausea and vomiting suggests pancreatitis. Pancreatitis is most likely to be encountered in alcoholics (such as this man) and patients who have biliary tract disease. This patient has a history of recurrent alcoholic pancreatitis. The development of gradual weight loss, chronic pain radiating to the back, and steatorrhea suggests that he has now developed chronic pancreatitis. This condition is often complicated by both endocrine and exocrine insufficiency of the pancreas, secondary to loss of much of the tissue of the pancreas to the disease process. The pancreas can fail as both an endocrine and an exocrine gland. Clinical Pearls Patients who have chronic pancreatitis can develop both endocrine and exocrine dysfunction as a result of inflammation and fibrosis of the gland. As a result of this tissue destruction, patients develop malabsorption with greasy stools, weight loss, and nutritional deficiencies. These patients need to be treated with pancreatic enzyme supplementation to replace the missing enzymes that the pancreas can no longer manufacture.

For patients with gastrointestinal disease, which of the following is the primary use for the Child-Pugh classification system? A. To determine prognosis in a patient with pancreatitis B. To assess the severity of liver disease C. To assess the activity of the liver to make clotting factors D. To assess the severity of sepsis E. To determine the risk for esophageal variceal bleeding

The correct answer is B. The Child-Pugh classification for the severity of liver disease assesses the degree of ascites, the serum concentrations of bilirubin and albumin, the prothrombin time, and the degree of encephalopathy. The other prognostic liver disease classification is the MELD (model for end-stage liver disease) score.

A 56-year-old alcoholic man gives a history of several years of constant epigastric pain, radiating straight through to the back. The pain is severe, present at all times, and exacerbated by eating. He also has steatorrhea and diabetes. He relates that he began to have episodes of acute alcoholic pancreatitis in his mid-thirties. At first these occurred every few years, but eventually they grew in number to several attacks per year. Eventually the pain became constant. Although he made numerous attempts to quit drinking, he was not successful until about 1 year ago; still, his current abstinence has not alleviated the pain. Which of the following is the most appropriate initial step in evaluation? A. Arteriogram B. CT scan of the upper abdomen C. Endoscopic retrograde cholangiopancreatogram (ERCP) D. Sonogram of the upper abdomen E. Upper gastrointestinal series with barium

The correct answer is B. The clinical diagnosis is chronic pancreatitis, and CT scan will give useful information about potentially correctable features of the disease, such as pseudocysts, dilated ducts, calculi, or areas amenable to resection. CT scan of the abdomen is considered to be the most sensitive test for picking up chronic pancreatitis. If surgery is contemplated, ERCP will then be needed. Clinical Pearls CT scan of the abdomen in a patient who is suspected of having chronic pancreatitis will show pancreatic calcifications, focal or diffuse enlargement of the pancreas, ductal dilation, and/or vascular complications. Although it is more expensive and has radiation exposure associated with it, the CT scan of the abdomen is 10-20% more sensitive for picking up chronic pancreatitis than abdominal ultrasound. Abdominal ultrasound is used in the assessment for possible chronic pancreatitis only if CT scan of the abdomen is not available. The plain abdominal film showing pancreatic calcifications is not very sensitive for mild to moderate chronic pancreatitis and this study cannot provide the level of anatomical detail that is needed when assessing for chronic pancreatitis.

Advanced liver disease will result in prolongation of which of the following? A. Activated partial thromboplastin time (aPTT) B. Prothrombin time C. Thrombin time D. Bleeding time E. Platelet count

The correct answer is B. The liver makes the clotting factors, which primarily affect the extrinsic clotting system. When the clotting factors of the extrinsic system are not made in the setting of end-stage liver disease, the prothrombin time is prolonged. Clinical Pearls The liver has many functions including making protein and ridding the body of toxins. One of the last functions that the liver loses is the ability of the liver to make clotting factors. Patients with end stage liver disease may have spontaneous bleeding which denotes that the liver has fully failed and mortality is very high at this point.

A 70-year-old man comes to the emergency department complaining of abdominal pain. He describes the pain as crampy and primarily located in his left lower quadrant. He has had minimal nausea, but complains of constipation. His past medical history is significant for hypertension, hyperlipidemia, and gout. His medications include atenolol and simvastatin. He is allergic to penicillin. His temperature is 38.0º C (100.4º F), blood pressure is 140/60 mm Hg, pulse is 100/min, and respirations are 20/min. His physical examination is significant for tenderness to palpation at the left lower quadrant without rebound or guarding. His stool is negative for occult blood. His heart and lung examinations are unremarkable. Which of the following is the most likely diagnosis? A. Appendicitis B. Diverticulitis C. Diverticulosis D. Ischemic colitis E. Sigmoid volvulus

The correct answer is B. The presence of cramping, left lower quadrant pain associated with fever and constipation is a classic presentation of diverticulitis. In the United States, the most common site of diverticulitis is the sigmoid colon, which is the area of the bowel with the highest intraluminal pressures. This is why inflamed diverticulum in the sigmoid colon results in left lower quadrant pain. Clinical Pearls Acute diverticulitis consists of left lower quadrant pain, tenderness, fever, and constipation. Rebound tenderness in the left lower quadrant may occur, and CT scan can show thickening of the bowel wall, mass, abscess, and some streakiness in the mesenteric fat. Treatment is with antibiotics and analgesia. If the diverticulitis leads to bowel perforation, surgery is indicated.

After passing his physical examination, a 19-year-old army recruit gives urine and blood samples for further testing. Serum analysis yields elevated ALT, HBsAg, anti-HBc, HBeAg, and bilirubin. All other values are normal. What is the hepatitis B status of this recruit? A. Asymptomatic carrier B. Active carrier C. Fulminant hepatitis B D. Recovered from acute self-limited HBV E. Vaccinated against HBV

The correct answer is B. The presence of elevated ALT, HBsAg, anti-HBc, HBeAg, and bilirubin all point to active hepatitis B. The HBeAg is known as the envelope, and this test parallels the level of active virus replication and predicts the level of hepatitis B infectivity. Clinical Pearls Patients who have hepatitis B will have measurable markers in their blood that will determine the status of the patient with regard to infectivity. Patients who are successfully immunized against hepatitis B will have +HBsAb. Patients who have recent infection will develop HBsAg. If the patient becomes infected with HBV and goes on to clear the infection (spontaneously or with treatment), the patient will undergo a change from +HBsAg to negative status with development of +HBsAb. True infection will cause +HBcAb formation, first with IgM (recent) followed by IgG becoming positive. The IgM core antibody may be the only marker while the patient is recovering from the hepatitis B infection, and this is known as the core window. Vaccination will not cause any reaction to the core. HBeAg is a marker of active viral replication and a marker for how potentially infectious the person is if someone had a blood exposure to that patient. Patients who have active carrier states will have +HBsAg, +HBcAb, and +HBeAg, with this pattern showing active viral replication.

A 53-year-old man is admitted to the hospital for fever and abdominal pain. He has a history of cirrhosis and is known to be hepatitis C positive. He was diagnosed with cirrhosis 4 years ago. He denies any alcohol or tobacco use. His only medications are spironolactone (Alactone) and propranolol (Inderal). He reports that 5 days ago he had a temperature of 38.9º C (102º F) along with a gradual onset of diffuse abdominal pain. On examination his temperature is 38.3º C (101º F), blood pressure 100/50 mm Hg, and pulse 110/min and regular. Lungs are clear. There are numerous spider angiomata on the thorax and back, and the abdomen appears distended. Laboratory studies show: Leukocytes 13,200/mm3 (normal 5,000-10,000) Hematocrit 33% (normal 41-50%) Prothrombin time 15.2 seconds (normal 11-13 seconds) Albumin 0.1 g/dL (normal 3.4-5.4 g/dL) Sodium 135 mEq/L (normal 135-145 mEq/L) Potassium 4.7 mEq/L (normal 3.5-5.3 mEq/L) Which of the following is the most appropriate next step in diagnosis? A. Abdominal CT scan B. Abdominal paracentesis C. Abdominal ultrasound D. Electrocardiogram E. Lumbar puncture

The correct answer is B. The prevalence of infection of ascites fluid, so called spontaneous bacterial peritonitis (SBP), is estimated at 50% in hospitalized cirrhotic patients. This patient has had long-standing cirrhosis, has an extremely low serum albumin level, and is on spironolactone (a direct aldosterone antagonist), which is prescribed to patients with ascites. All febrile patients admitted with ascites must have abdominal paracentesis performed to both determine the cause for the ascites and rule out infection of the ascites. A diagnosis of SBP is made when there is an elevated ascitic fluid absolute neutrophil count (>250 cells/mm3) without an evident intra-abdominal or surgically treatable cause for the infection. SBP most often occurs with portal hypertensive ascites. The most sensitive marker available for such ascites is a serum/ascites albumin gradient >1.1 g/dL. Clinical Pearls All febrile patients admitted with presumed ascites must undergo abdominal paracentesis to determine the cause of the ascites and rule out infection. SBP is diagnosed with an absolute neutrophil count >250 cells/mm3. The most sensitive marker available for spontaneous bacterial peritonitis is a serum/ascites albumin gradient >1.1 g/dL.

A 2-week-old infant is brought to the office for a newborn visit. His mother states that he has been a very slow eater and that he is constipated, not having had a bowel movement in 3 days. On physical examination, the infant has poor muscle tone, an enlarged tongue, an umbilical hernia, an enlarged anterior fontanelle, and hypothermia. He also looks slightly jaundiced, with slightly dry skin and brittle hair. Which of the following is the most likely diagnosis? A. Beckwith-Wiedemann syndrome B. Congenital hypothyroidism C. Hurler syndrome D. Trisomy 21 E. Turner syndrome

The correct answer is B. The signs and symptoms of congenital hypothyroidism may not be obvious for several days or weeks. Symptoms include physical sluggishness, constipation, large tongue, umbilical hernia, hypothermia, bradycardia, enlarged fontanelles, and persistent jaundice. Skin may be dry and scaly, and the hair may be dry, coarse, and brittle.

A 72-year-old woman comes to the emergency department complaining of severe left lower quadrant pain for 24 hours. She has a long history of constipation and frequently has left lower quadrant cramping after meals. Over the past 24 hours, she has had increasing discomfort in the left lower quadrant and has had a temperature to 39.4ºC (102.8ºF). On examination there is tenderness to palpation and guarding in the left lower quadrant. CT scan reveals an abscess involving and contiguous to the sigmoid colon. Which of the following is most likely causing this patient's condition? A. Bleeding at the site of the abscess B. Micro- or macroperforation of a diverticulum C. Mucosal inflammation D. Mucosal ischemia E. Pancreatitis

The correct answer is B. This question illustrates the classic presentation of acute diverticulitis. It is important to distinguish diverticulosis, the presence of diverticula in the colon, and acute diverticulitis, which implies acute inflammation of a diverticulum. Diverticulosis is primarily a disease of the elderly and of individuals who consume a low fiber diet and it results in the development of multiple diverticula. Most of the diverticula tend to develop in the very distal colon (where the stool tends to be the most dehydrated and hardest to move by peristalsis), although in severe cases even the ascending colon may be involved. This predilection for distal colonic involvement tends to clinically produce chronic constipation and left lower cramping after meals, as seen in this patient. Acute diverticulitis should be suspected in cases like this one. Most cases of acute diverticulitis occur as the result of a micro- or macroperforation of a diverticulum, which allows gut bacteria to escape from the bowel lumen. In this case, a transmural perforation has created an abscess in the region of the sigmoid colon. Clinical Pearls Diverticulosis results from overactivity of colonic smooth muscle, which results in herniation of mucosa and submucosa through the muscle layers of the colon. Diverticulosis typically is associated with multiple diverticuli. Low dietary fiber and high meat intake along with aging are risk factors for the development of these diverticuli, because these factors cause an increase in the intraluminal pressure within the colon. When diverticuli become inflamed secondary to infection, diverticulitis occurs. When symptoms occur with diverticular disease, constipation and left lower quadrant abdominal pain may occur. Treatment of acute diverticulitis includes bowel rest, hydration, antibiotics, and potentially surgery if conservative management is not effective.

A patient is seen in the office with complaints related to severe diarrhea. The patient has a high fever in addition to the diarrhea. The patient has just returned from a trip to the Bahamas. Which of the following is indicated at this time? A. Keeping the patient NPO until the diarrhea stops B. Ciprofloxacin (Cipro) C. Pepto-Bismol D. Amoxicillin E. Clarithromycin (Biaxin)

The correct answer is B. When diarrhea is acquired in a resource-poor environment, this traveler's diarrhea should be treated with ciprofloxacin. Clinical Pearls The most essential treatment for traveler's diarrhea is maintenance of hydration. Preferential antibiotics utilized in the setting of traveler's diarrhea are azithromycin, fluoroquinolone antibiotics, and rifaximin.

A patient with gastric cancer has metastatic disease signified by a palpable lymph node in the periumbilical area. What is this palpable lymph node known as? A. Krukenberg tumor B. Blumer shelf tumor C. Sister Mary Joseph nodule D. Virchow node E. Irish node

The correct answer is C. A Sister Mary Joseph nodule is a palpable periumbilical lymph node seen in the setting of metastatic gastric cancer.

Patients with cirrhosis may go on to develop ascites. In addition to increased portal pressures, which of the following conditions provides the other explanation as to why ascites develops in this situation? A. Lack of clotting factors produced by the liver B. Overhydrating the patient C. Hypoalbuminemia D. High-salt diet E. Continued intake of alcohol

The correct answer is C. Ascites arises because of hypoalbuminemia and increased portal pressures. Without enough protein and albumin in the circulatory system, there is a leakage of fluid out of the cells, further contributing to ascites.

In the setting of acute pancreatitis, which of the following evaluation strategies has the benefit of both confirming the diagnosis and identifying potential complications of this condition? A. Plain film of the abdomen B. Ultrasound of the abdomen C. Computed tomography (CT) of the abdomen D. Magnetic resonance angiography (MRA) E. Positron emission (PET) scan of the abdomen

The correct answer is C. CT of the abdomen confirms acute pancreatitis and demonstrates the potential complications of this condition. When done several days following the onset of symptoms, this diagnostic test can identify pancreatic necrosis and can also identify local complications from acute pancreatitis.

Which of the following causes of diarrhea is considered to be the most common cause of bacterial diarrhea in the United States? A. Enterotoxigenic E. coli B. E. coli 0157:H7 C. Campylobacter jejuni D. Staphylococcal food poisoning E. Salmonella

The correct answer is C. Campylobacter is the most common cause of bacterial diarrhea in the United States. This condition can be complicated by myasthenia gravis. It is typically acquired by the ingestion of contaminated, undercooked poultry and can cause both bloody and watery diarrhea.

A patient is admitted to the general hospital floor. The patient complained of skin flushing, sweating, wheezing, and abdominal pain. If carcinoid syndrome is suspected, which of the following neurotransmitters is secreted with this condition? A. Dopamine B. Epinephrine C. Serotonin D. Norepinephrine E. Prolactin

The correct answer is C. Carcinoid tumors secrete serotonin, and this is followed with 5HIAA levels.

A patient presents to the emergency department with acute abdominal pain. Which of the following descriptions of the patient's pain would be consistent with a diagnosis of diverticulitis? A. Burning substernal pain after meals B. Severe, diffuse ache in the periumbilical region C. Steady ache in the left lower quadrant with referral to the back D. Steady, boring epigastric pain with referral to the back E. Sudden, severe pain in the lower quadrant with referral to the flank

The correct answer is C. Diverticulitis commonly produces a steady, aching pain, localized to the left lower quadrant of the abdomen, with referral to the back in some cases. In addition, a tubular mass (caused by inflammation) may be appreciated on abdominal examination, and the patient may be febrile with an increased white count. Remember, if a patient presents with symptoms and signs similar to appendicitis, but the complaints concern the left rather than the right side of the abdomen, think of diverticulitis as a likely diagnosis. Also, take note that diverticulosis, not diverticulitis, is the most common cause of massive lower gastrointestinal bleeding in adults. Clinical Pearls Symptomatic diverticulitis presents with fever, leukocytosis, and left lower quadrant pain. It is caused by overactivity of colonic smooth muscle that results in herniation of mucosa and submucosa through the muscle layers of the colon, which form a diverticulum. Diverticulitis is inflammation of the diverticulum, which may be caused by infection. These diverticula can become inflamed resulting in pain in the left lower quadrant. Antibiotic coverage for this condition required coverage for anaerobes and gram-negative organisms. A complication of acute diverticulitis is perforation.

Which of the following treatments is the initial intervention performed in a patient who develops upper gastrointestinal bleeding as a result of esophageal variceal bleeding? A. Infusion of somatostatin B. Infusion of octreotide C. Endoscopic variceal ligation D. Insertion of Sengstaken-Blakemore tube E. Transjugular intrahepatic portosystemic shunt (TIPS)

The correct answer is C. Esophageal variceal ligation or sclerotherapy is the initial and primary treatment for esophageal varices. These procedures are performed endoscopically and are considered to be the first line in order to control bleeding acutely.

A 28-year-old man with end-stage renal disease (ESRD) on continuous ambulatory peritoneal dialysis (CAPD) for 2 months is brought to the emergency department with fever, abdominal pain, and cloudy dialysis fluid. The pain has been present for 12 hours. There is no diarrhea or vomiting. The patient has ESRD secondary to chronic glomerulonephritis; there is no history of diabetes, urinary infections, or antibiotic use. Examination reveals a temperature of 38.9ºC (102ºF) and blood pressure 110/70 mm Hg. The throat is clear, as are the lungs. Cardiac examination reveals a grade 2/6 systolic murmur. Abdominal examination reveals decreased bowel sounds with diffuse tenderness. There is mild rebound tenderness. There is no edema or skin rash. A complete blood count shows a leukocyte count of 14,200/mm3 (normal 5,000-10,000/mm3) and hemoglobin 12.5 g/dL (normal 13.8-17.0 g/dL). Peritoneal fluid is cloudy with 1,000 white blood cells, 85% of which are polymorphonuclear leukocytes. Gram stain of the peritoneal fluid is negative. Cultures of blood and peritoneal dialysis fluid are taken. Which of the following is the most appropriate initial step in management? A. Fluconazole B. Immediate removal of dialysis catheter C. Intravenous ceftriaxone and vancomycin D. Intravenous gentamicin E. Oral ciprofloxacin

The correct answer is C. Gram-negative organisms classically cause peritonitis in a patient on CAPD, but there is a recent tendency for this condition to be caused by gram-positive pathogens such as Staphylococcus aureus or Epidermidis. It is usually characterized by abdominal pain and over 100 white blood cells (typically polymorphonuclear leukocytes) in a sample of peritoneal dialysis fluid. Intravenous ceftriaxone and vancomycin would be a reasonable treatment to cover both gram-negative and gram-positive pathogens. Clinical Pearls Patients on chronic ambulatory peritoneal dialysis are at increased risk for peritonitis. When peritonitis occurs, it is likely to be caused by gram-negative enteric organisms or Staphylococcus aureus or Staphylococcus epidermidis, which will respond to ceftriaxone plus vancomycin. Other causative organisms include Enterobacteriaceae and enterococcus, which are part of the gut flora.

Patients who are diagnosed with hepatitis have many questions about whether they will recover or become chronically infected. Which of the following subtypes of hepatitis is most likely to progress to chronic disease if not treated? A. HAV B. HBV C. HCV D. HDV E. HEV

The correct answer is C. Hepatitis C can lead to a chronic carrier state if no intervention is performed. The majority of patients who acquire hepatitis C in the United States have genotype 1, and there is now effective therapy to eliminate this virus if patients does not clear this infection on their own.

Liver damage and liver disease are common entities seen in primary care practices. All of the following are known complications for liver failure EXCEPT: A. Portal hypertension B. Low albumin C. Low estrogen D. Low blood glucose E. Elevated bilirubin levels

The correct answer is C. Liver failure is complicated by high estrogen levels demonstrated by the patient having red palms, spider angiomas, gynecomastia, and testicular atrophy.

Which of the following hereditable gastrointestinal (GI) polyposis syndromes has associated endometrial and ovarian tumors accompanying the polyps? A. Familial adenomatous polyposis B. Gardner syndrome C. Lynch syndrome (familial nonpolyposis syndrome) D. Peutz-Jeghers syndrome E. Juvenile polyposis

The correct answer is C. Lynch syndrome is a familial syndrome with a 50% risk of colon carcinoma, which is associated with other primary cancers including endometrial. This nonpolyposis syndrome is due to defective DNA mismatch repair.

Which of the following diagnostic studies is recommended to perform in the setting of a patient suspected of having chronic mesenteric ischemia in order to confirm the diagnosis? A. CT of the abdomen with oral and IV contrast B. CT of the abdomen without contrast C. Mesenteric arteriography D. Plain upright abdominal film E. MRA of the abdomen

The correct answer is C. Mesenteric arteriography used to confirm chronic mesenteric ischemia. This study is able to identify the vascular flow pattern to the mesenteric artery and can be used to ascertain the underlying cause for this patient's ischemic pattern such as thrombosis, stenosis, or embolism. Mesenteric arteriography is still considered the definitive test for identifying mesenteric ischemia.

Which of the following laboratory study results is consistent with nonalcoholic steatohepatitis? A. Elevated prothrombin time B. Elevated activated partial thromboplastin time (aPTT) C. Elevated aspartate transaminase (AST) and alanine transaminase (ALT) D. Low level of bilirubin E. Low level of ceruloplasmin

The correct answer is C. Nonalcoholic steatohepatitis will cause elevated AST and ALT levels.

Outpatient treatment of acute diverticulitis may consist of which of the following single antibiotic regimens? A. Ciprofloxacin B. Trimethoprim-sulfamethoxazole (Bactrim) C. Ampicillin/clavulanate (Augmentin) D. Metronidazole (Flagyl) E. Tetracycline

The correct answer is C. Outpatient management of acute diverticulitis can consist of ampicillin/clavulanate, which provides both aerobic and some anaerobic coverage for the pathogens most commonly cause acute diverticulitis.

A patient is seen in the office with progressive jaundice, anorexia, abdominal pain, anorexia, fever, and fatigue. Anti-liver/kidney microsomal (anti-LKM) antibodies are found in the serum. Which of the following is the most likely diagnosis? A. Wilson disease B. Alpha-1 anti-trypsin deficiency C. Autoimmune hepatitis D. Primary biliary cholangitis E. Primary sclerosing cholangitis

The correct answer is C. Patients with autoimmune hepatitis will have overt symptoms of liver disease plus anti-smooth muscle antibodies. These smooth muscle antibodies include anti-LKM antibodies, which can also be found in patients with chronic hepatitis C infection.

Irritable bowel syndrome is the most common clinical entity evaluated by gastroenterologists in the United States. The diagnosis of irritable bowel syndrome is made on clinical grounds because there is no specific diagnostic laboratory test. Which of the following signs or symptoms would be consistent with a patient having irritable bowel syndrome? A. Bloody diarrhea B. Diarrhea occurring at night C. Relief of abdominal pain with passing bowel movement D. Weight loss E. Increased symptoms following a stressful event

The correct answer is C. Patients withirritable bowel syndrome have improvement of symptoms with passing stool. Patients with this condition can have mucus in stools, but there is no weight loss or nocturnal symptoms. This condition has its onset prior to the third decade so another diagnosis should be considered if these symptoms begin later in life.

The Child-Pugh classification system used for classifying the severity of cirrhosis includes calculation for all of the following factors EXCEPT: A. Serum bilirubin B. Serum albumin level C. Partial thromboplastin time D. Presence or absence of ascites E. Presence of absence of hepatic encephalopathy

The correct answer is C. Prothrombin time, not partial thromboplastin time, is part of the Child-Pugh classification system for classifying severity of cirrhosis.

A 15-year-old girl comes to the emergency department because of the sudden onset of profuse watery diarrhea. The girl was previously healthy. Her only medications are topical benzoyl peroxide and oral clindamycin for acne vulgaris. Physical examination reveals a slightly distended abdomen that is diffusely tender. Her temperature is 38.1°C (100.5°F). She has not been exposed to any uncooked meat or eaten unusual foods. Which of the following is the most likely diagnosis? A. Gastroenteritis B. Irritable bowel syndrome C. Pseudomembranous enterocolitis D. Salmonella infection E. Ulcerative colitis

The correct answer is C. Pseudomembranous enterocolitis is caused by the toxins produced by Clostridium difficile. It occurs in some patients after treatment with antibiotics (especially clindamycin, cephalosporins, and amoxicillin). Patients develop fever and abdominal pain with diarrhea containing leukocytes and blood. Clinical Pearls C. difficile colitis is most likely to occur as a complication of antibiotic use. Although any antibiotic use puts a patient at risk for acquiring this condition, clindamycin is infamous for causing this condition. The other main risk factor for C. difficile colitis is recent hospitalization. Patients who have this condition present with recurrent episodes of profuse watery diarrhea and abdominal pain. There may also be associated fever and abdominal tenderness.

Which of the following tests for Helicobacter pylori infection is useful to perform for patients who have had exposure to H. pylori but is not useful as a test for cure demonstrating eradication of this organism? A. Rapid urease test B. Histology with staining C. Serology testing D. Urea breath testing E. Stool antigen testing

The correct answer is C. Serology for H. pylori assesses whether infection is recent (IgM antibodies) or remote (IgG antibodies), but this test will not provide information about eradication following treatment, since IgG measurements will continue to be positive even in the setting of successful eradication.

Which of the following studies is indicated to perform for a hospitalized patient suspected of having acute mesenteric ischemia? A. Blood urea nitrogen (BUN) to creatinine ratio B. Absolute lymphocyte count C. Serum lactate level D. Procalcitonin level E. SED rate

The correct answer is C. Serum lactate level should be ordered with suspected acute mesenteric ischemia, as these levels will reflect the degree of acidosis and aid in determining the severity of illness.

A patient with long-standing alcohol-related cirrhosis and ascites is admitted to the hospital with a high fever and mental status change. Examination reveals abdominal pain, vomiting, and fever. Which of the following conditions is most likely? A. Hepatic encephalopathy B. Hepatorenal syndrome C. Spontaneous bacterial peritonitis D. Decompensated cirrhosis E. Portal hypertension

The correct answer is C. Spontaneous bacterial peritonitis causes fever and mental status changes in patients with ascites. A high index of suspicion should be held for patients who have ascites and develop these changes, because sepsis can progress if this is not treated.

A 69-year-old woman comes to the health care provider for a periodic health maintenance examination. She has no complaints. She takes 1,500 mg of calcium daily, a multivitamin, and daily aspirin. She exercises regularly, eats a fairly healthy diet, and does not smoke cigarettes. She drinks a couple of glasses of wine with dinner each evening and smokes marijuana occasionally with a friend who has glaucoma. She is concerned about her risks for breast, ovarian, and colon cancer because she has friends with each of these diseases. Breast and pelvic examination are unremarkable. A mammogram is normal. Findings from a barium enema are shown. Which of the following is the most common presenting symptom of this condition? A. Anemia B. Blumer shelf C. Constipation D. Virchow node E. Weight loss

The correct answer is C. The barium enema demonstrates an irregular lesion that causes obstruction in the sigmoid colon. This finding on barium enema is highly suspicious for adenocarcinoma. Cancers arising in the left and sigmoid colon are characterized by rectal bleeding, obstructive symptoms, and constipation possibly alternating with diarrhea. Stools may develop a narrow caliber as the tumor encroaches onto the lumen. Clinical Pearls Remember the most common signs for colon masses; some of them will depend on the location of the mass: A growing mass in the left colon is most likely to produce obstruction to the passage of feces, and thus, likely to present with constipation alternating with diarrhea or bloody bowel movements. A mass in the right side of the colon is most likely to present with chronic blood loss and anemia.

A 56-year-old man has been having bloody bowel movements on and off for the past several weeks. He reports that the blood is bright red, coats the outside of the stools, and appears in the toilet bowl even before he wipes himself. There is also blood on the toilet paper after he wipes. After further questioning it is ascertained that he has been constipated for the past 2 months and that the caliber of the stools has changed. They are now very thin compared to the usual diameter of an inch or so that was customary for him. He has some minor discomfort. Which of the following is the most likely diagnosis? A. Anal fissure B. Cancer of the cecum C. Cancer of the rectum D. External hemorrhoids E. Internal hemorrhoids

The correct answer is C. The combination of red blood coating the stools and a change in bowel habit and stool caliber spells out cancer of the rectum in someone in this age group. The stools become smaller in diameter as a result of the rectal tumor encroaching on the bowel lumen causing the stools to become smaller. Clinical Pearls The following are the combination of symptoms in the presentation of rectal cancer: Hematochezia (bright red blood in the stools) and/or enterorrhagia (intestinal bleeding) Change in bowel habit (constipation) Change in stool caliber (pencil-like feces)

Which portion of the gastrointestinal tract is most commonly involved with Crohn disease? A. Duodenum B. Jejunum C. Ileum D. Ascending colon E. Sigmoid colon

The correct answer is C. The terminal ileum is most commonly affected with Crohn disease. Clinical Pearls The involvement of the terminal ileum in the setting of Crohn disease will cause a patient to present with abdominal pain and diarrhea. Pain is more of a problem with Crohn disease, whereas bloody diarrhea is the main complaint of a patient with ulcerative colitis. Involvement of the terminal ileum is more likely to result in gallstones because of altered enterohepatic circulation. These gallstones are pigmented stones composed of increased amounts of bilirubin and calcium.

A 32-year-old woman has had a 15-year history of heartburn. Over the past 4 months, she has had difficulty swallowing large bites of solid food. She has no difficulty with soft foods or liquids, and she has not lost weight. Which of the following is the most likely explanation for her symptoms? A. Adenocarcinoma in the lower third of the esophagus B. Barrett's esophagus in the distal esophagus C. Fibrosis and narrowing at the distal esophagus D. Schatzki ring in the distal esophagus E. Squamous carcinoma in the mid-third of the esophagus

The correct answer is C. This patient has classic symptoms of mechanical dysphagia, as she has difficulty with large solid food but not softer foods or liquids. Mechanical dysphagia frequently follows many years of heartburn and is often indicative of a peptic stricture that has developed as a result of fibrosis after a long period of chronic inflammation caused by gastroesophageal reflux disease (GERD). These benign strictures can usually be dilated endoscopically. An intensive regimen of proton pump inhibitors should then be instituted to reduce the frequency of recurrence. This patient should be evaluated endoscopically to rule out Barrett's esophagus and adenocarcinoma of the distal esophagus, which may occur with long-term untreated GERD.

A patient is admitted to the hospital with suspected choledocholithiasis. The patient has a normal ultrasound of the right upper quadrant. Which of the following diagnostic strategies is indicated? A. CT of the abdomen B. MRI of the abdomen C. Angiogram of the abdomen D. Hepatobiliary iminodiacetic acid (HIDA) scan E. Endoscopic retrograde cholangiopancreatography RCP

The correct answer is E. ERCP or laparoscopic cholecystectomy with exploration of the common duct for removal of the common duct stone is a clinical intervention done in patients who have choledocholithiasis.

Four weeks after a camping trip, a 16-year-old boy begins to pass foul-smelling stools. He also develops anorexia and flatulence. None of his friends from the trip are ill. He is generally healthy and takes no medications. His temperature is 37.3ºC (99.1ºF). Physical examination shows diffuse abdominal pain and distention and guaiac-negative stool. The remainder of the examination is unremarkable. A microscopic image of his stool is shown. Which of the following statements is most accurate regarding his condition?? A. Asymptomatic infection is rare B. Boiling of water during the camping trip would not have killed the infective organism C. A 5- to 7-day course of metronidazole (Flagyl) has a cure rate of 80 to 95% D. Infection is limited to the large intestine E. The trophozoite is the infective form of this organism

The correct answer is C. This patient has giardiasis, which is caused by Giardia lamblia. G. lamblia is a flagellated protozoan that exists in a cyst and trophozoite form. The cyst, not the trophozoite, is the infective form (choice E). Infection is limited to the biliary tract and small intestine, not the large intestine (choice D). Contaminated water supplies are the common source of infection, and boiling water can kill the infective cysts (choice B). Many individuals infected with Giardia remain asymptomatic (choice A). A 5- to 7-day course of metronidazole has a cure rate of 80 to 95%. Clinical Pearls Giardiasis is an enteric infection caused by a flagellated protozoan parasite, Giardia lamblia. It is one of the most commonly identified intestinal parasites in the United States, though it is more common in the developing countries. Campers and those who drink fresh water sources are most likely to acquire this disease. Infection requires the oral ingestion of cysts, with as few as 10 cysts resulting in disease. Incubation is from 3-25 days. After the cysts are ingested, trophozoites are released into the small bowel. These trophozoites are converted to the infectious cysts in the large intestine. First-line therapy is metronidazole (Flagyl) or tinidazole (Tindamax), which has the advantage of a single 2-g curative dose. If treatment fails, a different class of medication should be tried, such as albendazole (Albenza). Parasites are classically cleared from the stool in 3-5 days and symptoms clear in 5-7 days. Health care providers need to stress the necessity of good hand washing after toileting or before preparing food to prevent spread of this disease. Patients need to avoid drinking untreated water, and any raw food that is going to be consumed should be washed thoroughly with uncontaminated water. When traveling to endemic areas, raw food should be avoided and all water boiled for at least 1 minute before consuming.

Which of the following is a known risk factor for the development of primary sclerosing cholangitis? A. Infection with Escherichia coli B. Porcelain gallbladder C. Ulcerative colitis D. Crohn disease E. Diverticulitis

The correct answer is C. Ulcerative colitis is a known risk factor for the development of primary sclerosing cholangitis.

Which of the following types of polyps has the greatest chance of undergoing malignant transformation? A. Tubular B. Tubulovillous C. Villous D. Hamartomas E. Hyperplastic

The correct answer is C. Villous adenomas have the greatest chance for malignant transformation.

A patient with gastric cancer has metastatic disease to the supraclavicular nodes on the left side. What is this metastatic disease known as? A. Krukenberg tumor B. Blumer shelf tumor C. Sister Mary Joseph nodule D. Virchow node E. Irish node

The correct answer is D. A Virchow node is a palpable left supraclavicular lymph node typically caused by metastatic gastric cancer.

Which of the following conditions is the most common cause of ascending cholangitis? A. Cholelithiasis B. Pancreatic pseudocyst C. Klatskin tumor D. Gallstone in the ampulla of Vater E. Pancreatic abscess

The correct answer is D. A gallstone in the ampulla of Vater is the most common cause of ascending cholangitis.

A 20-year-old man is brought to the emergency department 3 hours after ingesting 50 tablets of 325-mg acetaminophen in a suicide attempt. He has nausea and vomiting, but no other abnormalities on physical examination. His temperature is 37ºC (98.6ºF), blood pressure 135/80 mm Hg, pulse 100/min, and respirations 20/min. Serum acetaminophen concentration is within the range of probable toxicity but serum transaminases and other hepatic markers are normal. Which of the following is the most appropriate next step in management? A. Activated charcoal treatment B. Acetylcysteine treatment C. Penicillamine treatment D. Activated charcoal and acetylcysteine treatment E. Gastric lavage

The correct answer is D. Acetaminophen overdose may result in massive hepatocellular necrosis and fulminant hepatic failure. Liver metabolism of acetaminophen by P-450 cytochrome produces a highly toxic intermediate, which depletes glutathione stores, resulting in accumulation of free radicals. In the first few hours after ingestion, the patient may have nausea and vomiting, but no signs of liver failure. Biochemical evidence of hepatic damage, namely elevation of transaminases, begins 24-48 hours after ingestion. Therapy, therefore, should be started before hepatic transaminases begin to increase. The likelihood of hepatic toxicity is based on a combination of serum acetaminophen levels and time elapsed from ingestion. Nomograms have been developed to determine the likelihood that a specific level of acetaminophen at a given time following ingestion will produce hepatotoxicity. Within 4 hours following ingestion, the initial treatment should include activated charcoal (to adsorb residual drug in the intestine) and N-acetylcysteine. Treatment with N-acetylcysteine is most effective if started within 10 hours after ingestion. The FDA-approved recommendation is to continue oral treatment for 72 hours. Clinical Pearls Treatment of acetaminophen overdose is determined by the time since ingestion and the serum levels of acetaminophen, which are drawn serially. If the patient presents within 4 hours of ingestion of acetaminophen, treatment consists of N-acetylcysteine with or without an antiemetic, activated charcoal, and evaluation for possible liver transplantation.

In addition to lack of relaxation of the lower esophageal sphincter, which of the following other criteria is needed in order to make the diagnosis of achalasia? A. Esophageal ulceration B. Esophageal web C. Esophageal ring D. Poor peristalsis of the esophagus E. Alkaline pH of the esophageal aspirate

The correct answer is D. Achalasia has lack of lower esophageal sphincter relaxation along with poor peristaltic stripping waves.

A 50-year-old female is seen in the office with complaints of pruritus that is not responsive to antihistamines and topical skin lotions. Laboratory findings include an increase in the serum alkaline phosphatase level, along with increased serum bilirubin and cholesterol. If primary biliary cholangitis is suspected, which of the following additional laboratory tests is indicated? A. Serum iron level B. Prothrombin gene analysis C. Anti-smooth muscle antibody D. Antimitochondrial antibody E. Anti-liver/kidney microsomal antibody

The correct answer is D. Antimitochondrial antibodies are seen in primary biliary cholangitis.

A patient is seen in the office with recurrent bouts of painless rectal bleeding. The patient's bleeding has spontaneously stopped without other intervention. She is currently bleeding when seen in the office. Which of the following diagnostic evaluations should be performed for this patient? A. Plain abdominal films B. CT scan without contrast C. MRI D. Barium enema E. Colonoscopy

The correct answer is D. Barium enema is the test of choice in the evaluation of diverticulosis, as this procedure can identify the diverticulum.

A 70-year-old patient is admitted to the hospital with anemia related to ongoing gastrointestinal (GI) bleeding. He complained of hematochezia, and a rectal examination confirmed blood in the stool. After ensuring that the patient is stable, which of the following is the next step in his evaluation? A. Plain film of the abdomen B. Upper endoscopy C. Push procedure D. Colonoscopy E. Anoscopy

The correct answer is D. Colonoscopy is the first procedure indicated for patients who have hematochezia, as colon cancer is the primary consideration in this group.

A patient is admitted to the hospital with abdominal complaints. Which of the following patients is at highest risk for developing acute mesenteric ischemia due to nonocclusive mesenteric ischemia? A. Patient with atrial fibrillation B. Patient with peripheral vascular disease C. Patient with hypercoagulable state D. Patient who is critically ill and elderly E. Patient taking oral contraceptives

The correct answer is D. Critically ill, elderly patients are at highest risk for development of nonocclusive mesenteric ischemia secondary to splanchnic vasoconstriction secondary to low cardiac output.

A previously healthy 7-year-old girl comes to the office with complaints of episodic abdominal pain over the past several months. The pain is periumbilical and sharp but does not wake her from sleep and rarely interferes with play. She has no fever, joint complaints, or constipation or diarrhea. Growth and development have been normal. Physical examination is within normal limits. Which of the following is the most likely diagnosis? A. Acute appendicitis B. Acute cholecystitis C. Crohn's disease D. Functional abdominal pain E. Irritable bowel syndrome

The correct answer is D. Functional abdominal pain is pain that lasts for more than 3 months and rarely interferes with normal activity. The pain is periumbilical and often hard to describe. The pain typically does not awaken patients from sleep or interfere with pleasant activities. The pain is real and is the result of the regulation of gastrointestinal motility in response to either psychologic or physical stress. Clinical Pearls Functional abdominal pain is seen most frequently in girls 8-12 years of age, and patients who have this disorder typically complain of vague, persistent, central abdominal pain that can be associated with nausea and/or vomiting. Examination will reveal periumbilical tenderness with a soft abdomen and no rebound or guarding. The remainder of the physical examination is normal. This is a diagnosis of exclusion and no red flag symptoms are present (weight loss, failure to thrive, nocturnal diarrhea, signs of bleeding).

In addition to preventing hepatitis B in a patient, hepatitis B vaccination will also prevent which of the following other types of hepatitis? A. HAV B. None C. HCV D. HDV E. HEV

The correct answer is D. Hepatitis D can be prevented by vaccination against hepatitis B because hepatitis D can occur only in the setting of a patient having hepatitis B infection.

Pancreatitis can occur as a result of certain metabolic disorders. This is the reason as to why people are treated for these conditions so that acute pancreatitis does not occur. Which of the following metabolic conditions is most commonly associated with acute pancreatitis? A. Increased low-density lipoprotein (LDL) cholesterol B. Decreased high-density lipoprotein (HDL) cholesterol C. Increased chylomicrons D. Increased triglycerides E. Increased lipoprotein (Lp)(a)

The correct answer is D. Increased triglyceride levels are a cause of acute pancreatitis. Clinicians should start to worry when the triglyceride levels exceed 1,000.

Which of the following treatments is recommended for patients with erosive esophagitis? A. Esophageal dilation B. Fluticasone used without a spacer C. Botulism toxin injection D. Long-term proton-pump inhibitor (PPI) therapy E. Long-term H2 blockers

The correct answer is D. Long-term PPI therapy is needed for patients with erosive esophagitis, as these erosions are very difficult to treat. Twice-daily PPI therapy may be needed in order to allow sufficient healing to take place.

Various treatments are given for the management of gastrointestinal (GI)-related complaints. What is the mechanism of action for metoclopramide (Reglan) in the management of GI complaints? A. Blocks histamine release B. Blocks histamine receptors C. Stimulates the proton pump to stop making acid D. Promotility agent E. Antacid

The correct answer is D. Metoclopramide, a dopamine agonist, is a promotility agent used in the management of gastroparesis, gastroesophageal reflux disease, and chemotherapy-related and post-operative nausea and vomiting. This agent sensitizes tissues to acetylcholine, which stimulates upper GI tract motility.

A patient is seen in the office 4 weeks following admission to the hospital. If a pancreatic pseudocyst is suspected as a complication of acute pancreatitis, which of the following is the most common presenting sign or symptom? A. Fever B. Jaundice C. Tender, enlarged liver D. Abdominal pain E. Persistent nausea and vomiting

The correct answer is D. Persistent abdominal pain and tender upper abdominal mass are common findings for a patient with a pancreatic pseudocyst.

Which of the following hereditable gastrointestinal (GI) polyposis syndromes has associated hamartomatous polyps of the entire GI tract and mucocutaneous pigmentation accompanying these polyps? A. Familial adenomatous polyposis B. Gardner syndrome C. Lynch syndrome (familial nonpolyposis syndrome) D. Peutz-Jeghers syndrome E. Juvenile polyposis

The correct answer is D. Peutz-Jeghers syndrome results in numerous hamartomatous polyps of the entire GI tract with the highest number in the small bowel. These patients are at increased risk for the development of cancer at both GI and non-GI sites.

Some systemic diseases have physical manifestations outside of the original site that the condition affects. For example, extraintestinal signs of disease can help to diagnose an underlying gastrointestinal (GI) problem. Pyoderma gangrenosum is a skin manifestation of which of the following GI conditions? A. Chronic pancreatitis B. Eosinophilic colitis C. Crohn disease D. Ulcerative colitis E. Arteriovenous malformations

The correct answer is D. Pyoderma gangrenosum is associated with ulcerative colitis.

Which of the following causes of infectious diarrhea is most commonly acquired from eating food containing mayonnaise? A. Enterotoxigenic E. coli B. E. coli 0157:H7 C. Vibrio parahaemolyticus D. Staphylococcal food poisoning E. Salmonella

The correct answer is D. Staphylococcal food poisoning is acquired after food is handled by a carrier such as salads, dairy products, produce, milk, and eggs, which would include the ingestion of mayonnaise.

The Charcot triad is seen in which of the following clinical conditions? A. Acute pancreatitis B. Acute common bile duct stone C. Acute cholecystitis D. Acute cholangitis E. Mesenteric thrombosis

The correct answer is D. The Charcot triad consists of right upper quadrant pain, fever, and jaundice, which occurs in the setting of acute cholangitis. Suppurative cholangitis can lead to the Reynold pentad, which adds the symptoms of confusion and hypotension to the original three symptoms.

The Charcot triad is seen in which of the following clinical conditions? A. Acute pancreatitis B. Acute common bile duct stone C. Acute cholecystitis D. Acute cholangitis E. Mesenteric thrombosis

The correct answer is D. The Charcot triad consists of right upper quadrant pain, fever, and jaundice. These symptoms are seen in the setting of acute cholangitis.

A patient is admitted to the hospital with severe right upper quadrant abdominal pain that radiates into the back and right scapula. An ultrasound of the right upper quadrant is normal. The patient is still suspected of having acute cholecystitis but lack of a positive ultrasound finding prohibits surgery. Which of the following diagnostic tests is indicated for further assessment for acute cholecystitis? A. CT of the abdomen B. MRI of the abdomen C. Angiogram of the abdomen D. HIDA scan E. ERCP

The correct answer is D. The health care provider should perform a hepatobiliary iminodiacetic acid (HIDA) scan if ultrasound is normal. In this setting, acute cholecystitis will prevent visualization of the gallbladder in 4 hours, confirming a diagnosis of acute cholecystitis.

Three months after a needle-stick exposure to blood from a patient with hepatitis B, a nurse is evaluated for infection with the virus. Laboratory results reveal: On the basis of these results, which of the following most accurately describes the nurse's hepatitis B status? HBsAg - absent, anti-HBs antibody - absent, IgM anti-HBc - present, IgG anti-HBc - absent, HBeAg - absent A. She had been effectively vaccinated against hepatitis B before the needle-stick exposure occurred. B. She has mounted an inappropriate antibody response to hepatitis B as a result of an immunocompromised state. C. She is a carrier of hepatitis B. D. She is actively infected with hepatitis B. E. She was not infected with hepatitis B.

The correct answer is D. The nurse's elevated IgM anti-HBc indicates she was infected with hepatitis B. Formerly HBsAg (surface antigen) and anti-HBsAg (antibody to surface antigen) were used exclusively to determine this. Typically HBsAg is positive for up to 6 months, and anti-HBsAg is positive for years after that. Unfortunately this simple scheme has the disadvantage that many patients have a 2-week to 4-month window period, when the surface antigen (HBsAg) and the antibody (anti-HBs) are not detectable. Presumably, for a relatively brief period, HBsAg production exactly matches antibody production, and the two co-precipitate such that neither free species is present in adequate concentration to be detectable. This problem can be circumvented either by serial measurements of HBsAg and anti-HBs, or by concurrent measurement of other antigens and antibodies, including HBeAg, anti-HBe, and anti-HBc (HBcAg is not reliable). During the core window period, IgM anti-HBc may be the only marker of recent HBV infection, as it is in this nurse. If the nurse had been effectively vaccinated for hepatitis B (choice A), she would have had an elevated anti-HBs antibody level and no HBsAg present in the serum. Anti-HBc antibody would have been absent as well if her only exposure to hepatitis B was through vaccination. The negative HBeAg suggests that the patient does not have active replication of the virus at this time, as this "envelope" level parallels the active replication of the virus in the body. Even though there is not a high viral load at this time, there is no evidence that this patient has cleared this infection. Clinical Pearls Patients who have hepatitis B will have measurable markers in their blood, which determine the status with regard to infectivity. Patients who are successfully immunized against hepatitis B will have +HBsAb. Patients who have recent infection will develop HBsAg. If the patient becomes infected with HBV and goes on to clear the infection (spontaneously or with treatment), the patient will undergo a change from +HBsAg to negative status with development of +HBsAb. True infection will cause +HBcAb formation, first with IgM (recent), followed by IgG becoming positive. The IgM core antibody may be the only marker while the patient is recovering from the hepatitis B infection, and this is known as the core window. Vaccination will not cause any reaction to the core. HBeAg is a marker of active viral replication and a marker for how potentially infectious the person is if someone had a blood exposure to that patient.

The Charcot triad becomes the Reynold pentad when which of the following signs or symptoms occur? A. Fever B. Right upper quadrant pain C. Jaundice D. Altered mental status E. Anuria

The correct answer is D. TheReynold pentad adds altered mental state and septic shock to the Charcot triad of right upper quadrant pain, fever, and jaundice.

A 5-year-old boy is brought to the emergency department with 2-day history of fever, anorexia, loose stools, and yellow skin color. He attends a large daycare center. On physical examination his temperature is 38.1ºC (100.7ºF), blood pressure 88/56 mm Hg, pulse 74/min, and respirations 15/min. Initial laboratory evaluation reveals a total bilirubin of 1.8 mg/dL (normal <1.9 mg/dL) and alanine aminotransferase 764 U/L (normal <25 U/L in this age group). Nobody else in his family is sick. Which of the following is the most appropriate diagnostic test? A. Hepatitis B surface antigen in serum B. IgG for hepatitis A in serum C. IgG for hepatitis B surface antigen in serum D. IgM for hepatitis A in serum E. Stool culture for hepatitis A

The correct answer is D. This boy most likely has hepatitis A. While it is more common in developing countries, it is still a common infection in the developed world. It is transmitted by the fecal-oral route when contaminated food or water is ingested. The incubation period is about 15-40 days. 90% of children acutely infected with hepatitis A are asymptomatic. The virus is shed in the feces 2-3 weeks before the onset of jaundice until approximately 1 week after onset. Most infected children are infectious for a long time before they are symptomatic. Hepatitis A is therefore very difficult to control. Large outbreaks frequently occur in daycare centers. Diagnosis is best made by determination of IgM levels against hepatitis A virus. The presence of IgM for hepatitis A suggests acute infection; this antibody peaks at 4-6 weeks and does not persist beyond 6 months. IgG is produced in the primary infection, but for most viral infections, including hepatitis A, it persists for a lifetime. Prophylaxis with immunoglobulin is recommended for the household and close contacts of the infected person within the first 2 weeks of exposure. Strict hand washing is also very important. There is a killed-virus vaccine available that provides immunity and this vaccination is part of the recommended vaccination series. Clinical Pearls Risk factors for hepatitis A infection include travel outside the United States, contact with other children in day care, and poor sanitation. Hep A is acquired via a fecal-oral route of transmission. Determination of anti-hepatitis A virus IgM levels is the best confirmatory test, which suggests the presence of an acute infection. Hepatitis A infection will result in increased transaminase levels, with ALT typically becoming more greatly elevated than AST. Transaminase levels will increase before elevation in bilirubin. Alkaline phosphatase levels will also increase but to a minimal degree. IgM anti-HAV has a high sensitivity and specificity and can be detected even before the onset of symptoms and can persist for 4-6 months. Acute infections will have IgM elevation as part of antibody response before IgG elevations against HAV. IgG elevation will show previous infection and sufficient antibody protection. IgG elevation will rise after IgM elevation and will stay elevated throughout a person's lifetime.

A 69-year-old woman comes to her health care provider's office complaining of 1 week of crampy lower abdominal pain and bloody diarrhea. She had previously been followed for symptoms of stable exertional angina and hypertension. She had an uncomplicated myocardial infarction 3 years earlier. Her symptoms began 1 week ago with mild postprandial abdominal cramping followed by diarrhea, which became bloody after 2 days. She has not traveled recently and is a retired librarian. Her temperature is 38.6ºC (101.4ºF), blood pressure 120/84 mm Hg, and pulse 96/min. She has moderate tenderness to palpation of the left lower quadrant. Rectal examination reveals bloody stool and no masses. Which of the following is the most likely diagnosis? A. Arteriovenous malformation B. Diverticulitis C. Diverticulosis D. Ischemic colitis E. Ulcerative colitis

The correct answer is D. This elderly woman who has a history of atherosclerotic vascular disease as demonstrated by a history of a myocardial infarction and exertional angina has developed colitis symptoms, as demonstrated by the left lower quadrant pain and bloody diarrhea. This is typically caused by ischemia of small branches of the inferior mesenteric artery. The bleeding is caused by poor blood flow to the bowel, which leads to mucosal sloughing that can cause blood loss into the bowel lumen. The diagnosis is suspected clinically and generally confirmed with a flexible sigmoidoscopy, because many cases involve the rectosigmoid region. Clinical Pearls The risk factors for ischemic colitis are aging and cardiovascular disease. Risk factors include atrial fibrillation, myocardial infarction, atherosclerosis, and heart failure. The presentation of patients who have ischemic colitis is typically abdominal pain that is variable, depending on the portion of the bowel that is involved. Patients will present with hematochezia or melena along with diarrhea and abdominal tenderness. Treatment depends on the underlying cause. Patients will need fluid resuscitation and possible embolectomy or bypass procedure to restore blood flow to the colon.

A 25-year-old man presents with weight loss, abdominal pain, and bloody diarrhea. Sigmoidoscopy/colonoscopy reveals mucosal erythema and ulceration extending in a continuous fashion proximally from the rectum. Which of the following pathologic findings would also be characteristic of this patient's illness? A. Bowel wall thickening B. Cobblestone appearance of mucosa C. Fistulas D. Pseudopolyps E. Transmural lesions

The correct answer is D. You are being tested here on your ability to distinguish between ulcerative colitis (UC) and Crohn's disease. First, you need to figure out which one this patient has. Key clues are the bloody diarrhea (much more common in UC), the rectal involvement, and especially the continuous nature of the mucosal damage. Once you have identified UC, you need to identify the answer choice that is characteristic of UC. The correct answer is pseudopolyps, which are inflammatory polyps found in ulcerative colitis and not Crohn's disease. These post-inflammatory polyps (also known as pseudopolyps) are considered to be high risk for the development of colon cancer, because they are markers of significant inflammatory bowel disease. Clinical Pearls Ulcerative colitis is more likely than Crohn's disease to have rectal involvement, mucosal involvement (rather than transmural involvement), continuous proximal spread of inflammation from the rectum, and bloody diarrhea. Because of significant inflammation, patients will have pseudopolyp formation, which is a feature that is much more likely to occur with ulcerative colitis than with Crohn's disease. Surgical intervention in ulcerative colitis is considered to be curative, whereas the disease will recur in a patient who has Crohn's disease, which is more likely to have GI involvement anywhere from the mouth to the anus, with perianal involvement frequently being present. Extraintestinal manifestations are part of both types of inflammatory bowel disease.

You are managing a postoperative patient who underwent an aortic aneurysm resection several days ago. The patient began having melena, and you are called to assess him. Which of the following causes of this gastrointestinal (GI) bleeding should be primarily investigated? A. Duodenal ulcer B. Gastric ulcer C. Arteriovenous (AV) malformation D. Dieulafoy vascular malformation E. Aortoenteric fistula

The correct answer is E. Aortoenteric fistula causes GI bleeding following aortic aneurysm resection, possibly from pulsation of the vessel against the graph wall, which leads to bleeding.

Although both Crohn disease and ulcerative colitis are both types of inflammatory disease, there treatment may be significantly different. Which of the following therapeutic agents is much more useful in patients with Crohn disease rather than ulcerative colitis? A. Sulfasalazine B. Systemic corticosteroids C. Azathioprine D. 6-Mercaptopurine (6-MP) E. Cholestyramine (Questran)

The correct answer is E. Cholestyramine and colestipol are bile-acid sequestrants used for people with terminal ileal disease who cannot reabsorb bile salts.

Which of the following diagnostic evaluations is the preferred clinical intervention to pursue in a patient having a lower gastrointestinal (GI) bleed caused by angiodysplasia of the colon (arteriovenous malformations)? A. Plain abdominal films B. CT scan without contrast C. MRI D. Barium enema E. Colonoscopy

The correct answer is E. Colonoscopy is the preferred intervention for patients who have lower GI bleeding caused by AV malformation. Colonoscopy can identify and treat angiodysplasia.

On review of a patient's laboratory results during a follow-up visit, the patient's initial laboratory profile revealed that he had an elevated alkaline phosphatase level. A gamma-glutamyl transferase (GGT) was then ordered, and it has returned as normal. Which of the following conditions is consistent with this finding? A. Ongoing intake of alcohol B. Blockage of the gallbladder C. Acute hepatitis D. Sclerosis of the gallbladder ducts E. Bone disease

The correct answer is E. Elevated alkaline phosphatase and normal GGT mean bone disease or pregnancy. The GGT test is the most sensitive test for liver, and, if this test is normal, the alkaline phosphatase would need to be caused by something other than liver disease.

Which of the following is the most likely cause of a patient with cirrhosis to develop macrocytic anemia? A. Blood loss anemia from poor development of clotting factors B. Concurrent hypothyroidism C. Vitamin B12 deficiency state D. Pernicious anemia state E. Folate deficiency

The correct answer is E. Folate deficiency from long-term alcohol use and a poor nutritional state in cirrhosis is likely to lead to macrocytic anemia.

A 64-year-old man with a long history of "heart burn" comes to the emergency department with 6 episodes of hematemesis. He denies alcohol use, smoking, or drug use. He also denies significant nonsteroidal anti-inflammatory drug use. He has no known liver disease. His laboratory results are normal with the exception of hemoglobin 7.1 g/dL (normal male 13.8-17.2 g/dL). He is taken for an emergent esophageal gastroduodenoscopy, which reveals a gastric ulcer with a bleeding visible vessel. Compared with duodenal ulcers, gastric ulcers have which of the following? A. Better response to medication B. Less commonly associated with significant gastrointestinal bleeding C. Greater chance for development in those who drink coffee D. More common association with H. pylori E. More common association with malignancy

The correct answer is E. Gastric ulcers may be associated with malignancy, and therefore biopsy should be performed when they are discovered. Repeat esophageal gastroduodenoscopy (EGD) is necessary to assure that these ulcers have healed. This association with malignancy is not found with duodenal ulcers. Clinical Pearls Patients who have gastric ulceration should have a repeat EGD 6-8 weeks after treatment to ensure that healing has taken place and to rule out gastric cancer, because a small number of gastric cancers can present as gastric ulcers. Patients who have uncomplicated duodenal ulcers do not need follow-up endoscopy. If the patient was treated for H. pylori eradication, follow-up should be done to assess for cure, because resistance to treatment has been documented.

A patient is seen in the office in order to review laboratory results from a recent yearly physical. The patient was found to have an elevated unconjugated bilirubin level consistent with Gilbert disease. Which of the following is the treatment of choice? A. Metformin (Glucophage) B. Interferon C. Cholestyramine (Questran) D. Low-fat diet E. None

The correct answer is E. Gilbert disease causes an increase in serum indirect bilirubin in a fasting state and during stress. There is no pathology associated with this condition, and the elevated indirect bilirubin level will normalize when the patient is fed. Because there is no pathology associated with this inherited condition, no treatment is needed.

A patient with long-standing cirrhosis is examined in the office. The patient is noted to have spider angiomas, palmar erythema, gynecomastia, and testicular atrophy. Which of the following underlying conditions is responsible for these findings? A. Low albumin levels B. Chronically low glucose levels C. Hepatic encephalopathy D. Elevated bilirubin levels E. High estrogen levels

The correct answer is E. High estrogen levels cause gynecomastia, testicular atrophy, spider angiomas, and palmar erythema.

A patient is seen in the office complaining of pain in the epigastrium, radiating to the back. The pain is described as steady and dull but also severe. Lying down and eating meals make the pain worse. The patient is anorexic at present but did have a great deal of nausea and vomiting prior to this visit. Examination reveals abdominal distention along with epigastric tenderness. Which of the following electrolyte abnormalities can lead to this presentation? A. Hyponatremia B. Hypokalemia C. Hyperkalemia D. Hypocalcemia E. Hypercalcemia

The correct answer is E. Hypercalcemia can lead to acute pancreatitis, but this is not a common underlying cause. The proposed cause of hypercalcemia leading to acute pancreatitis is from calcifications being laid down in the pancreatic ducts and also the activation of trypsinogen due to high calcium levels.

Patients with acute pancreatitis may go on to develop chronic pancreatitis. Clinicians should have a high index of suspicion for chronic pancreatitis when they are following patients who have been treated for acute pancreatitis. Signs and symptoms of chronic pancreatitis include all of the following EXCEPT: A. Abdominal pain B. Diarrhea after eating C. Weight loss D. Steatorrhea E. Jaundice

The correct answer is E. Jaundice is not a common sign of chronic pancreatitis, whereas pain and signs of malabsorption are seen. Jaundice typically occurs with blockage of the common duct, which is typically treated and therefore not a problem for patients with chronic pancreatitis.

A 4-month-old boy has gained only 10 ounces since birth. He has failed to gain weight with multiple different formula preparations. His stools have been loose and fatty. An older sister had similar symptoms and has been repeatedly hospitalized for failure to thrive and recurrent pulmonary infections. Which of the following is the most likely cause of this patient's gastrointestinal symptoms? A. Achlorhydria B. Bacterial overgrowth C. Colonic inertia D. Gastric hypersecretion E. Pancreatic exocrine insufficiency

The correct answer is E. The patient described here has cystic fibrosis. This inherited disease primarily affects the gastrointestinal and respiratory systems. Cystic fibrosis can present in early infancy with meconium ileus, in the first year of life with steatorrhea and failure to thrive, or in older childhood with recurrent pulmonary infections. The pancreatic secretions are characteristically very thick and tend to occlude the pancreatic duct system. Steatorrhea has developed in this infant because of pancreatic duct inspissation of secretions and the resultant decrease in delivery of pancreatic enzymes to the small bowel. Clinical Pearls Cystic fibrosis is a genetically inherited condition that affects multiple organs, especially the GI and respiratory tracts. There is a defect in the salt and water transport across the epithelial surfaces that results in pancreatic dysfunction resulting in calorie malabsorption. There is also lung disease from mucus retention, infection, and inflammation. Pancreatic enzymes are given to support the patient's growth and nutrition. Due to the propensity for infections, long-term antibiotics are frequently used.

A 56-year-old alcoholic comes to the health care provider complaining of 6 months of worsening midepigastric pain radiating to his back. The pain is exacerbated by eating and continued alcohol use. One month ago he was told that he had developed diabetes, which requires insulin for control. He has also noted that, over the past 2 months, he has had greasy, foul-smelling, large-volume stools. Which of the following is the most likely cause of this man's insulin-dependent diarrhea? A. Acid inactivation of pancreatic enzymes B. Bacterial overgrowth C. Biliary insufficiency D. Excess secretion of glucagon E. Insufficient lipase available for normal fat digestion

The correct answer is E. Midepigastric pain radiating to the back and exacerbated by eating and continued alcohol use should suggest the diagnosis of pancreatitis. Significant risk factors for pancreatitis include alcoholism (as this patient has) and biliary tract disease (for which there is no evidence in this patient). It is a little unclear from the history whether the patient is experiencing recurrent episodes of acute pancreatitis or is developing the onset of chronic pancreatitis. Whichever is the case, both processes are capable of destroying enough endocrine and exocrine pancreatic tissue to predispose for endocrine and exocrine pancreatic insufficiency with resultant insufficient production of pancreatic enzymes, such as lipase, which normally facilitates fat usage, to allow for normal digestion and insulin which is needed for glucose metabolism. Steatorrhea, or excess fat in stool, is seen clinically as greasy, foul-smelling (because of bacterial action on the fat), large-volume (in part because of gas production by bacteria and in part because of failure to digest most of the food) stools. When endocrine function of the gland fails and sufficient beta cells in the pancreas are lost, insulin replacement will be needed to prevent ketoacidosis.

A patient is admitted with severe abdominal pain that is due to mesenteric ischemia. Acute mesenteric ischemia can be caused by a variety of processes. Which of the following causes of mesenteric ischemia occurs as a result of non-occlusive disease? A. Antithrombin III deficiency B. Atherosclerosis C. Vasculitis from systemic lupus erythematosus D. Factor V Leiden mutation E. Hypotension

The correct answer is E. Nonocclusive causes of mesenteric ischemia include hypotension; heart failure; arrhythmia; and the use of digitalis, which acts as a vasoconstrictor.

Which of the following esophageal disorders is most likely to be treated surgically rather than medically? A. Achalasia B. Corkscrew esophagus C. Plummer Vinson syndrome D. Type 1 esophageal hernia (sliding) E. Type 2 esophageal hernia (paraesophageal)

The correct answer is E. Paraesophageal hernias are treated surgically because of the risk for incarceration and strangulation.

Which type of hepatitis infection has a high fatality rate if acquired during pregnancy? A. HAV B. HBV C. HCV D. HDV E. HEV

The correct answer is E. Patients who acquire hepatitis E during pregnancy have a 10% to 20% mortality risk.

A patient is suspected of having cirrhosis. Which of the following laboratory findings would be an expected finding if this patient has long-standing cirrhosis? A. Hypernatremia B. Hyperalbuminemia C. Hyperglycemia D. Elevated testosterone level in male E. Prolonged prothrombin time (PT)

The correct answer is E. Patients with long-standing cirrhosis can have a prolonged PT due to the liver failing to make clotting factors.

A patient is evaluated for cholestasis symptoms and found to have bead-like structuring and bead-like dilations of the intrahepatic and extrahepatic ducts. Which of the following treatments would be most likely to help with symptomatic relief of pruritus? A. Ursodiol (Actigall) B. Hyoscyamine (Anaspaz) C. Penicillamine (Cuprimine) D. Lactulose E. Cholestyramine (Questran)

The correct answer is E. Patients with primary biliary cholangitis have symptomatic response to cholestyramine for pruritus.

A 61-year-old man presents with colicky abdominal pain and vomiting of 3 days duration. On physical examination there is moderate distention, high-pitched hyperactive bowel sounds, and a 5-cm tender groin mass. On direct questioning he explains that he has "had that bulge for many years." He has always been able to "push it back in" when he lies down but for the past 3 days he has been unable to do so. His temperature is 38.9ºC (102ºF) and white blood cell count 12,500/mm3 (normal 5,000-10,000/mm3). Which of the following is the most appropriate management at this time? A. Sonogram of the mass B. Trial of nasogastric suction and IV fluids for a few days C. Insertion of a long rectal tube via sigmoidoscopy D. Manual reduction of the hernia, followed by a period of observation E. Urgent surgical intervention

The correct answer is E. The clinical picture is that of a strangulated or irreducible inguinal hernia. If he only had the tender mass without signs of intestinal obstruction, he might have omentum trapped. If he had no fever with a normal white blood cell count but a tender mass, he could be obstructed but without strangulation. The combination that he has, however, is clearly that of obstruction with strangulation. He needs urgent surgery. Delay in surgery can lead to necrotic bowel, which increases both morbidity and mortality. Clinical Pearls A hernia that cannot be reduced might be strangulated. Typical signs are abdominal pain, fever, a mechanical ileus (entrapped bowel), and leukocytosis. Strangulated hernia is a surgical emergency. Open repair is preferred, because the patient may need surgical resection of the gangrenous section of the bowel. Mesh application is typically not part of the repair because of the risk for bacterial invasion caused by the compromised bowel.

Hemochromatosis may affect a variety of organs in the body with iron deposition. Which of the following is considered to be the primary organ involved with this iron deposition disease? A. Heart B. Joints C. Skin D. Thyroid E. Liver

The correct answer is E. The liver is the primary organ involved with hemochromatosis, as this organ preferentially absorbs iron once the red blood cells and bone marrow are saturated with it.

A 21-year-old college senior comes to the health care provider with a 2-month history of frequent episodes of loose stool, preceded by lower abdominal cramping. Over the past 4 weeks, the stools have become increasingly bloody. On a number of occasions he has had the sensation of rectal fullness but is unable to pass any fecal matter. A sigmoidoscopy reveals inflammation in a circumferential pattern from the anal verge to the mid-sigmoid colon, where a transition to normal mucosa is seen. Which of the following is the most appropriate treatment for this patient? A. IV hydrocortisone B. IV infliximab (Remicade) C. Oral azathioprine (Imuran) D. Oral prednisone E. Topical mesalamine (Asacol, Pentasa)

The correct answer is E. The patient described here has ulcerative colitis confined to the distal colon, also known as ulcerative proctosigmoiditis. Because the disease is limited to the distal colon, topical agents such as mesalamine (or alternatively hydrocortisone) would be effective in reducing inflammation. Mesalamine is an anti-inflammatory drug used principally to control ulcerative colitis. Its active ingredient is also known as 5-aminosalicylic acid, which is available in the forms of rectal suspension, suppositories, delayed release oral tablets, and controlled release oral capsules. The mode of action is unknown but is thought to involve topical (because mesalamine is poorly absorbed), rather than systemic, modulation of arachidonic acid metabolites, including prostaglandins, leukotrienes, and hydroxyeicosatetraenoic acids. It is usually well tolerated, but it can cause significant allergic reactions related to sulfite sensitivity. Clinical Pearls Ulcerative colitis treatment is dependent on the activity of the disease and the extent of colon involvement. The goal of therapy is to induce and to maintain remission. For mild to moderate disease affecting the distal colon, topical mesalamine is clearly a first-line choice. This is also the treatment of choice for distal colitis that is in remission. Oral mesalamine may be needed for patients who have extensive disease of the colon that has extensive colitis.

A 43-year-old man develops excruciating abdominal pain at 8:23 PM (he looked at his watch when the pain "hit him"). When seen in the emergency department about 30 minutes later, he has a rigid abdomen, lies motionless on the examination table, has no bowel sounds, and is obviously in great pain which he describes as constant and encompassing the entire abdomen. There is very severe pain when deep palpation of the abdomen is attempted in any of the 4 quadrants. The examining hand cannot make much of an indentation, however, because of the impressive muscle guarding. There is also severe rebound tenderness. Radiographs show free air under both diaphragms. Which of the following diagnosis is most likely? A. Acute abdomen, the nature of which cannot yet be defined B. Acute inflammatory process affecting an intra-abdominal viscera C. Acute obstruction of an intra-abdominal viscera D. Ischemic process affecting intra-abdominal organs E. Perforation of the gastrointestinal tract

The correct answer is E. There is no doubt that this patient has an acute abdomen, but we can tell more than that. The sudden onset, generalized extent, and silent abdomen in a man who does not want to move suggest a perforation. In addition, the presence of free air in the peritoneal cavity pinpoints the gastrointestinal tract as the source. We cannot tell whether he perforated a peptic ulcer, blew out a sigmoid diverticulum, or had his bowel perforated by a chicken bone, but there is a hole in his gastrointestinal tract. Clinical Pearls When there is a perforation in the GI tract hollow viscus, the radiographic manifestation will be free air under the diaphragm. Once a patient is shown to have this manifestation, surgical intervention is needed to identify the site of the perforation so that correction can be performed. The two most common GI sites for perforation are the stomach and duodenum from peptic ulcer disease and the rectosigmoid colon from acute diverticulitis.

Treatment of acute viral hepatitis may include all of the following interventions EXCEPT: A. High-calorie diet B. IV hydration if vomiting is a persistent symptom C. Cholestyramine (Questran) for pruritus D. Liver transplantation for fulminant hepatic failure E. Glucocorticoids for persistent inflammation

The correct answer is E. There is no role for glucocorticoids in the management of viral hepatitis.

A 44-year-old woman with a history of recurrent biliary colic presents with 18 hours of very severe right upper quadrant pain, fever, and jaundice. An abdominal ultrasound reveals a markedly dilated common bile duct. Multiple gallstones are seen in the gallbladder. Which of the following would best determine whether there is cystic duct obstruction? A. Abdominal CT scan B. Abdominal MRI C. Abdominal ultrasound with Doppler flow studies D. Endoscopic retrograde cholangiopancreatograph (ERCP) E. HIDA scan

The correct answer is E. This patient has evidence of a prolonged episode of biliary colic, which may progress to acute cholecystitis. An abdominal ultrasound has revealed a markedly dilated common bile duct, as well as gallstones within the gallbladder. Acute cholecystitis is the result of cystic duct obstruction, and this would be best demonstrated by a HIDA scan. A HIDA scan shows visualizes the iminodiacetic acid, which follows the flow of bile (i.e., from the bloodstream into the liver, into the gallbladder, and then through the cystic duct into the common bile duct). Clinical Pearls The HIDA scan or cholescintigraphy scan will directly show cystic duct obstruction by showing a failure of gallbladder filling with normal hepatic uptake of iminodiacetic acid. For a patient who has no blockage of the cystic duct, the test will show dye accumulation in the gallbladder. This test will also show the function of the gallbladder; some patients do not have cystic duct blockage but rather a failure of the gallbladder to contract. This would make the gallbladder dysfunctional, indicating a need for cholecystectomy in a symptomatic patient.

A 45-year-old woman has been experiencing generalized weakness and a sensation of "pins and needles" for the past 3 weeks. She exercises daily, rarely drinks alcohol, and is a strict vegetarian. Her temperature is 37ºC (98.6ºF), blood pressure 110/70 mm Hg, pulse 60/min, and respirations 18/min. Examination shows weakness of the proximal and distal muscles of the lower extremities. Deep tendon reflexes are increased. The gait is ataxic. Which of the following is the most likely cause of these symptoms? A. Guillain-Barré syndrome B. Lambert-Eaton syndrome C. Myasthenia gravis D. Polymyositis E. Vitamin B12 deficiency

The correct answer is E. This patient has subacute combined degeneration of the spinal cord, which is caused by a vitamin B12 deficiency. It is most often caused by pernicious anemia, but it may be acquired by patients who have strict vegetarian diets or small bowel disease. The clinical manifestations include weakness, paresthesias, loss of vibratory sensation, increased deep tendon reflexes, and extensor plantar responses. The gait is ataxic. Mental changes may also occur. The diagnosis is made by measuring serum vitamin B12 levels, homocysteine, and methylmalonic acid. Vitamin B12 deficiency occurs only after years of being a vegetarian, because vitamin B12 stores are present for years before depletion occurs. Neurologic changes caused by pernicious anemia are considered to be irreversible, so early treatment is warranted. The treatment is vitamin B12 replacement. Vitamin B12 deficiency is a condition that clinicians need to diagnose and treat. Typical symptoms which may occur with long-term vitamin B12 deficiency are: Pallor Weakness Paresthesia Ataxic gait Loss of vibratory sensation Increased deep tendon reflex Extensor plantar responses Dementia

A 4-month-old girl is brought to the office because of frequent crying, sleep disturbance, and decreased appetite. She has been irritable and crying a great deal since birth, especially after feeding when she might spit up and even vomit. She also seems to get hiccups frequently. She has not been gaining weight as expected and her parents are concerned that something may be wrong. Last night they thought she had stopped breathing for approximately 20 seconds. She has been on a diet of breast milk and some formula supplementation. On physical examination the infant seems in no acute distress. Her vital signs are within normal limits and her weight and length are in the tenth percentile. Otherwise the physical examination is within normal limits. Which of the following is the most appropriate next step in management? A. Inserting a nasogastric feeding tube B. Nissen fundoplication C. Prokinetics D. Proton pump inhibitors E. Thickening of feeds with cereal and elevating the head of the bed

The correct answer is E. This patient has the typical presentation of gastroesophageal reflux. Gastroesophageal reflux occurs with a reduction in lower esophageal sphincter pressure, inappropriate lower esophageal sphincter relaxation, hiatal hernia, or delayed gastric emptying. This condition is common in the pediatric population, particularly in infants with developmental delay or cerebral palsy. It is a minor condition and of no consequence (so-called "functional gastroesophageal reflux"); patients present with a wide array of symptoms. Some form of spitting up and even forceful vomiting are common. Apnea can be a presenting sign. Reflux into the hypopharynx triggers laryngospasm and subsequent obstructive apnea. Chronic cough and wheezing may signal aspiration. Some patients exhibit poor weight gain and failure to thrive. Sandifer syndrome presents with gastroesophageal reflux and opistotonus, presumably to avoid aspiration or decrease pain. A pH probe is the standard method that is used for diagnosing gastroesophageal reflux. Other imaging studies include technetium scanning and barium swallow. Initial therapy consists of antireflux measures, such as elevating the head of the bed and thickening of feeds. If this fails to provide relief of symptoms, medical management with antacids, prokinetics, H2-receptor blockers, and proton pump inhibitors should be attempted. Failure of medical management may require surgical correction with a Nissen fundoplication. Most patients, however, have resolution of symptoms without any treatment. Clinical Pearl Gastroesophageal reflux is a common entity encountered in clinical practice, with 10 to 20% of the population having GERD. All age groups are affected and this condition is a common occurrence in infants. Infants who have developmental delays and cerebral palsy are especially affected by GERD. First-line management consists of thickening the feeding and keeping the infant in a more upright position during and following feeding. If these modifications are ineffective, proton pump inhibitors may be added as first-line management.

A 54-year-old man, who 5 years ago underwent laparotomy for a gunshot wound to the abdomen, is admitted to the hospital because of protracted vomiting and progressive abdominal distention. The symptoms began 5 days ago, and since then he has not had a bowel movement or passed any gas. At the time of hospitalization, he has hyperactive bowel sounds and some abdominal discomfort but he does not have an acute abdomen. Abdominal radiographs show dilated loops of small bowel, multiple air-fluid levels, and no free air under the diaphragms. He is placed on nasogastric suction and intravenous fluids. After 6 hours he develops fever, leukocytosis, and abdominal tenderness. There is severe tenderness when external pressure is applied to his abdomen and then suddenly released. There are no audible bowel sounds. Which of the following is the most appropriate next step in management? A. Add antibiotics B. Barium tag and serial abdominal radiographs C. CT scan of the abdomen D. Upper gastrointestinal endoscopy and introduction of a long intestinal tube E. Emergency exploratory laparotomy

The correct answer is E. This patient presents with mechanical intestinal obstruction caused by adhesions which developed as a result of previous exploratory laparotomy, and he now has signs of bowel strangulation. If the strangulated loop is still viable, it has to be freed immediately. If it is necrotic, it has to be resected with equal urgency to prevent continued peritoneal soiling. This patient has now developed an acute abdomen, as manifested by rebound tenderness, fever, and leukocytosis. Clinical Pearls The clinical picture is typical of bowel obstruction. Abdominal distention, vomiting, and hyperactive bowel sounds are common manifestations. The most common cause of bowel obstruction is adhesions from a previous surgery. Regardless of the cause, it is always a surgical emergency when bowel function is at risk.

A 24-year-old man comes to the clinic for evaluation of a 4-month history of postprandial diarrhea, weight loss of 9 pounds, and lower abdominal pain. He denies recent travel or antibiotic use. On physical examination his temperature is 38.0ºC (100.4ºF), and he has several oral aphthous ulcers. On abdominal examination there is tenderness and mild voluntary guarding in the right lower quadrant. Rectal examination reveals brown stool that is strongly positive for occult blood. Which of the following is most likely causing this patient's symptoms? A. Gram-negative organism B. Folate deficiency C. Mucosal ulceration with no transmural involvement in the ascending colon D. Toxin-producing organism E. Transmural inflammation in the region of the terminal ileum

The correct answer is E. This patient, who has postprandial diarrhea, weight loss, low-grade fever, and right lower quadrant findings on physical examination, has the typical presentation of Crohn's disease, which most commonly involves the terminal ileum. Inflammation in this disease is transmural, as opposed to the inflammation in ulcerative colitis that is limited to the mucosa of the large intestine. Clinical Pearls Crohn's disease patients can present with chronic diarrhea, weight loss, and right lower quadrant abdominal pain. The diarrhea is caused by decreased absorption of bile acids with resultant secretory diarrhea. This depletion of the bile salt pool leads to malabsorption of fat with resultant steatorrhea and increased risk for gallstone formation. Fat soluble vitamins then become depleted. Diagnosis is confirmed with colonoscopy with biopsy. Because transmural inflammation of the bowel is the rule with Crohn's disease, bowel complications include obstruction, abscess formation, sinuses, and fistulae formation.

A patient with chronic liver disease is seen in the office. The patient is noted to have Kayser-Fleischer rings. Which of the following laboratory findings is most likely to be present? A. Alpha-1 antitrypsin deficiency B. Elevated hemoglobin A1c C. Positive antibodies to the glomerular basement membrane D. Howell-Jolly bodies on peripheral smear E. Low ceruloplasmin levels

The correct answer is E. Wilson disease is associated with accumulation of copper manifested by Kayser-Fleischer rings in the eye. This condition has a low level of ceruloplasmin, which is a copper-binding protein.

A 36-year-old man with ulcerative colitis develops pruritus and fatigue. Alkaline phosphatase is elevated. The biliary tree appears beaded on barium radiograph. Which of the following is the most likely diagnosis? A. Acute cholecystitis B. Cholesterolosis C. Chronic cholelithiasis D. Gallstone ileus E. Sclerosing cholangitis

The correct answer is E. Young men who have ulcerative colitis are at increased risk for developing primary sclerosing cholangitis (PSC), a chronic cholestatic condition that leads to fibrosis of the bile ducts. This cholestasis leads to an elevation in the alkaline phosphatase level. A classic clue to the diagnosis is a beaded appearance of the biliary tree on barium radiograph. Clinical Pearls Sclerosing cholangitis is a condition in which there is scarring in the bile ducts, leading to cholestatic liver disease, cirrhosis, and end-stage liver disease. It is most common in young and middle-aged men, especially those with underlying inflammatory bowel disease. Pruritus and jaundice are presenting symptoms. Serum alkaline phosphatase levels are elevated along with serum gamma-GT levels. When hepatocellular damage occurs, there is elevation of aminotransferase levels and conjugated bilirubin. ERCP and MRCP studies will show tight narrowing in the extrahepatic biliary tree caused by progressive structuring. Typically there is both intrahepatic and extrahepatic duct involvement. Sclerosing cholangitis can progress to causing cholangiocarcinoma.

A 47-year-old woman undergoes a minor elective operation and develops a cascade of complications. She has to be intubated after the procedure, and she is admitted to the intensive care unit. An older sister told the health care provider that the patient was normal when she last saw her before moving to a distant city, but that when she encountered her again several years later, the patient seemed to be "in slow motion" all the time. She spoke slowly and with a croaky voice, had gained weight, seemed to be mentally limited, and only talked about how cold she was and how ineffective the laxatives that she took were. Pursuing that information, stat T4 determination is made, and the report shows an extremely low value. At this time the patient most urgently needs which of the following therapies?

The diagnosis is clear: the patient is profoundly hypothyroid, a condition that had not been suspected when she had the surgery. Hypothyroidism can develop insidiously. Patients who are severely hypothyroid indeed do very poorly when they have surgery. Replacement of T4 with IV levothyroxine will replace the missing thyroid hormone that this patient has as a result of her previously unrecognized and untreated hypothyroidism. This patient will also need corticosteroid coverage during this replacement in order to support the adrenal gland functioning when thyroid hormone replacement is given.

A 56-year-old obese woman with hyperlipidemia and hypertension comes in for her biannual physical examination. She has been doing fairly well over the last 6 months, though she admits to having gained 10 pounds since her last visit. She denies any blurry vision, polyuria, or polydipsia, but she has a strong family history of type 2 diabetes. You are concerned that she is at risk for development of diabetes. Which of the following is the most appropriate screening test for diabetes?

The diagnosis of diabetes can be made in many ways but the screening test most commonly used is the fasting glucose level. Patients who have fasting glucose levels >126 mg/dL on 2 occasions meet the criteria for diabetes. The first intervention should be a trial of diet and exercise. Patients can be given a glucometer to take and record their blood sugar levels at home. If glucose values don't return to normal in approximately 3 months, medications should be administered. In symptomatic patients, a random blood glucose >200 mg/dL is diagnostic of diabetes. Patients with hemoglobin A1c of 6.5% or higher are also diagnosed with diabetes.

A 7-year-old girl is brought to the emergency department by her parents with a complaint of severe polyuria and polydipsia. Laboratory examination reveals ketones in her urine. Which of the following is the most likely source of the ketones?

The patient is presenting with signs and symptoms highly suggestive of type 1 diabetes. The primary source of ketones in the urine is free fatty acid breakdown. Ketone body formation occurs as follows: insulin deficiency → activated lipolysis → increased plasma free fatty acids → increased hepatic fatty acids → accelerated ketogenesis. In summary, as fatty acids break down, acetyl-CoA is generated. As acetyl-CoA levels increase, ketone bodies begin to form. When excessive amounts of ketone bodies are formed, the pathologic state known as ketosis can occur.

Following thyroidectomy, hoarseness of the voice may occur. This condition is caused by damage to which of the following?

The recurrent laryngeal nerves are branches of the vagus (CN X), and supply all intrinsic muscles of the larynx except the cricothyroid. The right recurrent laryngeal nerve recurs around the right subclavian artery. The left recurrent laryngeal nerve recurs in the thorax around the arch of the aorta and ligamentum arteriosum. Both nerves ascend to the larynx by passing between the trachea and esophagus, close to the thyroid gland.

A 2-week-old infant is brought to the office for a newborn visit. His mother states that he has been a very slow eater and that he is constipated, not having had a bowel movement in 3 days. On physical examination, the infant has poor muscle tone, an enlarged tongue, an umbilical hernia, an enlarged anterior fontanelle, and hypothermia. He also looks slightly jaundiced, with slightly dry skin and brittle hair. Which of the following is the most likely diagnosis?

The signs and symptoms of congenital hypothyroidism may not be obvious for several days or weeks. Symptoms include physical sluggishness, constipation, large tongue, umbilical hernia, hypothermia, bradycardia, enlarged fontanelles, and persistent jaundice. Skin may be dry and scaly, and the hair may be dry, coarse, and brittle.

A 20-year-old woman comes to the health care provider because she has never had a menstrual period. She has no medical problems, has never had surgery, and takes no medications. Examination reveals that she is a tall female with long extremities. She has normal-sized breasts, though the areolae are pale. She has scant axillary hair. Pelvic examination is significant for scant pubic hair and a short, blind-ended vaginal pouch. Which of the following is the most appropriate next step in management?

These findings are consistent with androgen insensitivity syndrome, formerly called testicular feminization syndrome. The syndrome results from genetic defects leading to abnormal androgen receptor function. Patients who have androgen insensitivity syndrome are genotypically males (46,XY) but phenotypically females with breasts and no external male genitalia. The reason that breasts develop is that estrogens, which are expressed at puberty and which also result from peripheral conversion of androgens, act upon the breast tissues unopposed by androgens because of the androgen receptor defect. This unopposed estrogen leads to breast growth and the resultant breasts are normal sized, although they have undeveloped nipples and pale areolae. There are no internal female organs, because mullerian-inhibiting substance is present during development. There are no external male organs because of the androgen receptor defect. Testicles do exist, but they are intra-abdominal. The gonads have a high rate of malignant degeneration in patients who have androgen insensitivity syndrome and therefore, after puberty, they should be removed via bilateral gonadectomy. It is important to wait until after puberty so that full physical development can take place.

A bone mass resulting from accumulation of osteoclasts, reactive multinucleated giant cells, and hemosiderin deposits in areas of microfractures is characteristic of which of the following endocrine abnormalities?

This brief histologic description refers to brown tumors, which develop in association with hyperparathyroidism. These lesions result from repeated microfractures, with subsequent accumulation of hemosiderin-laden macrophages and reactive osteoclastic and fibroblastic proliferation. The brown color is caused by hemosiderin deposition.

A 5-month-old girl is brought to the clinic by her mother for her first office visit. The mother is concerned that her daughter has poor eye contact and seems to be slow. The infant was born at home, without complications. On physical examination the patient is fair-skinned and has severely dry skin. She has obvious signs of developmental delay. No newborn screening tests were performed. What is the most likely diagnosis?

This is a classic description of phenylketonuria (PKU), an autosomal-recessive disease that affects the brain. Affected infants are normal at birth but develop severe mental retardation if a phenylalanine- and protein-restricted diet is not followed. A phenylalanine-restricted diet that is started within the first 10 days of life can prevent the onset of developmental delay. Screening for PKU is required in all newborns in the United States.

232: Patients who fail medical therapy for Gi complaints may need to be referred for surgery. If a patient with known normal esophageal motility is referred for a laparoscopic Nissen fundoplication which of the following surgeries will be performed? a. Correction of hiatal hernia b. Reinforcement of the lower esophageal sphincter to relax c. Cutting of muscle fibers in order to allow the lower esophageal spinster to relax D. removal of a Schatzki ring E. Clipping of a retropharyngeal pouch

b. Reinforcement of the lower esophageal sphincter to relax

A 19-year-old girl comes to the health care provider because she has not had a menstrual period. She experienced normal breast development through puberty but has yet to have a period. She has no other complaints. She has no medical problems. Examination shows the patient to be tall with long arms and big hands. The breasts are normal-appearing except that the nipples are immature and the areolae are pale. Pelvic examination shows scant pubic hair with a blind-ended vaginal pouch. Which of the following is the most likely diagnosis?

This patient has a presentation and findings that are most consistent with androgen insensitivity syndrome (also called testicular feminization syndrome). These patients are genotypically male (46,XY) but phenotypically female, because they have a defect that prevents normal androgen receptor function. The androgen receptor gene is located on the X chromosome, and various defects in the gene (e.g., absence of the gene or abnormalities in the androgen binding domain of the receptor) can lead to this syndrome. Patients who have androgen insensitivity are amenorrheic and have no internal female structures. Testes rather than ovaries are present. These patients also have minimal axillary and pubic hair. They do experience abundant breast development at puberty, as testosterone is unable to suppress the formation of breast tissues. These patients also tend to be very tall with big hands and feet and long arms. Testes should be removed after pubertal development is completed, as many of these patients will develop gonadal malignancies after puberty.

A 50-year-old chronic alcoholic man presents with dementia, paralysis of lateral gaze, and difficulty walking. Deficiency of which of the following vitamins is most likely to be present in this patient?

This patient has all the signs of Wernicke encephalopathy, characterized by dementia, ataxia, and ophthalmoplegia with nystagmus and strabismus. This disease typically occurs in alcoholic patients and is the result of thiamine, or vitamin B1, deficiency. Patients who repeatedly vomit, are alcohol-dependent, and who have magnesium deficiency states are prone to develop this condition. Thiamine (vitamin B1) deficiency presents according to how chronic the deficiency state is. Several clinical syndromes are associated with thiamine deficiency, including Wernicke encephalopathy, and wet and dry beriberi. Risk factors for thiamine deficiency include alcohol dependence, malabsorption, and poor dietary intake. Acute deficiency states can present with Wernicke encephalopathy with ocular abnormalities (nystagmus, strabismus), mental state changes, and ataxia ("wacky, wet, and wobbly"). Wet beriberi is associated with high output cardiac failure, edema, and orthopnea. Dry beriberi can present with distal peripheral neuropathy with paresthesias, decreased reflexes, and muscle wasting.

A 45-year-old woman is taken to an emergency department after an all-day car trip during which she had inadequate access to fluids. She had required many bathroom stops, including 3 times when she had urinated behind a tree because no bathroom was immediately available. In the emergency department, she is found to have severe orthostatic hypotension. Urine dipstick is negative for glucose. She continues to produce large volumes of urine, and urine osmolarity is low. On questioning, she does not note any unusual drinking pattern, but her husband states that she "usually goes through 4 six-packs of diet soft drink" every day. The patient's condition improves with intravenous fluid replacement. Vasopressin injection increases her urine osmolarity by more than 50%. Which of the following is the most likely diagnosis?

This patient has central diabetes insipidus. People can slowly develop dietary or drinking habits that seem perfectly normal to them, but appear strikingly unusual to other people. In this case, the patient essentially underwent a water deprivation test, but without the close medical supervision always offered when such a test is performed in a hospital setting. The facts that she continued to produce dilute urine and that the urine osmolarity could be increased with vasopressin indicate that the diabetes insipidus probably has a central origin. A head CT scan would likely confirm the diagnosis by showing a large pituitary adenoma, which could compromise antidiuretic hormone flow down the pituitary stalk.

A 13-year-old boy is brought to the health care provider because of recent-onset enuresis. He has had these accidents happen 4 or 5 times over the previous 3 weeks. The parents had also noticed that he was drinking unusually large amounts of water and his appetite also increased. He has lost 5 pounds in the past 2 months, despite this increased appetite. He had been in good health previously. He is in 8th grade and is a good student, physically active and attending karate class 3 times/week. On physical examination the patient is in no acute distress. Vital signs are within normal limits. He is in the 50th percentile for height and the 40th percentile for weight. Inspection of the mouth reveals dry but otherwise normal-appearing mucosa. Auscultation of the chest and palpation of the abdomen are unremarkable. Lab studies show elevated glucose at 270 mg/dL (normal 74-106 mg/dL) and bicarbonate 21 mEq/L (normal 20-29 mEq/L). Complete blood count is normal. Urine glucose is 4+ mmol/L (normal no glucose spilling) and trace ketones in the urine (normal no ketones in the urine). Which of the following is the most appropriate treatment approach?

This patient has developed type 1 diabetes mellitus. Diabetes mellitus results from a deficiency of insulin or a defect in its action, or both. This results in abnormal metabolism of carbohydrate, protein, and fat. The 2 major classifications of diabetes are type 1 diabetes (formerly known as juvenile diabetes mellitus), in which there is a severe lack of insulin and type 2, which is characterized by insulin resistance. Type 1 is very common in children and has a very strong correlation with autoimmune etiology. It is more common in people with other autoimmune diseases, and there is an association with certain human leukocyte antigens. Islet cell antibodies are present in most newly diagnosed patients. Preceding viral infections may serve as a trigger. Many children present with the classic symptoms of polyuria, polydypsia, polyphagia, and weight loss. Up to 25% present in diabetic ketoacidosis, with vomiting, abdominal pain, dehydration, Kussmaul respirations, and cerebral obtundation. Diabetic ketoacidosis occurs when the glucose is >300 mg/dL and there is ketonemia, acidosis (pH <7.30, bicarbonate, 15 mEq/L), glucosuria, and ketonuria. Diabetes mellitus also can be diagnosed in patients who have milder symptoms. The new criteria are symptoms of diabetes with a random plasma glucose ≥200 mg/dL, fasting glucose ≥126 mg/dL (on more than one occasion), 2-hour glucose tolerance ≥200 mg/dL, or elevated hemoglobin A1c. In all cases, hyperglycemia and glucosuria are necessary for diagnosis. Treatment of diabetic ketoacidosis involves correction of dehydration and electrolytes, and insulin. Treatment of type 1 diabetes mellitus involves insulin, diet, and exercise. Complications of diabetes mellitus include retinopathy, cataracts, nephropathy, neuropathy, and atherosclerosis. Most diabetic complications occur in adults. Dietary management is an important aspect to managing this disease, because patients are dependent on insulin, and learning to count carbohydrates will help to determine appropriate insulin-dosing.

A 64-year-old woman comes to the emergency department because of severe wrist pain after a fall. She says that she fell on an outstretched hand 2 days ago and has been experiencing severe pain ever since. She has had 3 episodes of nephrolithiasis, chronic muscle weakness, and vague abdominal complaints over the past few months. Physical examination is normal. Radiograph of the wrist shows a fracture of the distal radius and osteopenia. A splint is placed on her wrist and a follow-up visit is scheduled. Which of the following most likely explains these findings?

This patient has hyperparathyroidism, which is characterized by an elevation of circulating parathyroid hormone (PTH), most commonly caused by a benign adenoma. An elevation of PTH leads to high calcium and low phosphate levels. These patients often have unbalanced bone resorption and formation leading to osteopenia and fractures, recurrent nephrolithiasis, vague abdominal symptoms, weakness, and easy fatigability. "Bones, stones, abdominal groans, and moans" is a mnemonic used to remember the symptoms. The treatment is surgery to remove the parathyroid adenoma.

A 53-year-old man comes to the health care provider because of progressive weakness and weight loss over the past 2 months. He says that he also began noticing areas of his skin getting darker even though it is winter and he is never in the sun. He takes no medications and has no other medical conditions. Physical examination shows no abnormalities except for orthostatic hypotension and hyperpigmentation of the skin.

This patient most likely has Addison disease, which is primary adrenocortical deficiency. It is a rare disease caused by a progressive destruction of the adrenal glands, usually because of idiopathic atrophy, autoimmune disease, surgery, infection, or hemorrhage. Clinical symptoms include weakness, weight loss, hyperpigmentation, nausea and vomiting, and hypotension. Laboratory findings include hyponatremia (caused by aldosterone deficiency and inability to retain salt), hyperkalemia, and normocytic anemia with eosinophilia and lymphocytosis. Diagnosis is made with the ACTH stimulation test. Cortisol and aldosterone levels do not increase when the ACTH is given in primary adrenal insufficiency because the adrenal gland is unable to make these hormones. Treatment is lifetime glucocorticoid and mineralocorticoid replacement.

Foot examination is performed on a 45-year-old man who was diagnosed with type 1 diabetes mellitus at age 15. Although the man has no complaints of foot pain, both feet appear clearly deformed, with loss of the normal arches and abnormal orientation of the toes. No inflammation is noted. Radiographs show deformed bones with new bone formation adjacent to the bone cortex. Several large, bizarrely shaped osteophytes are seen at the joint margins. Which of the following is the most likely diagnosis?

This patient most likely has neurogenic arthropathy. The disorder develops in a setting of impaired pain perception and position sense, and can cause a rapidly destructive osteoarthritis-like arthropathy. The problems appear to be caused by unfelt minor injury, without the normal response of pain, causing resting of, and natural splinting (by muscle contraction or shifting position) of the affected joints. The most commonly encountered setting is diabetic foot disease. This peripheral neuropathy can also occur in a wide variety of other conditions, including tabes dorsalis (syphilis), syringomyelia, Arnold-Chiari malformation, meningomyelocele, leprosy, tumors of peripheral nerve or spinal cord, vertebral disease with damage to peripheral nerves, amyloidosis, and familial hereditary neuropathies. The joints affected are those that have lost pain innervation, possibly in association with palsies of nearby muscles.

What other diseases can neurogenic arthropathy of the joints occur in?

This peripheral neuropathy can also occur in a wide variety of other conditions, including tabes dorsalis (syphilis), syringomyelia, Arnold-Chiari malformation, meningomyelocele, leprosy, tumors of peripheral nerve or spinal cord, vertebral disease with damage to peripheral nerves, amyloidosis, and familial hereditary neuropathies.

A 34-year-old woman goes into labor in week 42 of gestation and delivers a girl who weighs 2,600 g and is 43 cm long. At 1 minute, the neonate is assessed to have a pale pink body with mottled and cool extremities. Her pulse is 95/min; there is a weak cough, generalized hypotonia with some flexion of the extremities, and a slow respiratory effort. Her skin is dry, with scarce lanugo. There is no significant change at her 5-minute assessment. Detailed physical examination reveals macroglossia with coarse facial features, enlarged fontanelles, and an umbilical hernia. Which of the following is the most appropriate screening test for this neonate?

Total T4 and Thyrotropin. This neonate has the typical clinical presentation of congenital hypothyroidism. The most appropriate screening tests to diagnose this disorder would be to measure total T4 and thyrotropin levels. Congenital hypothyroidism results from a deficiency of thyroid hormone. Congenital hypothyroidism occurs in approximately 1/4,000 births. Most cases occur because of thyroid dysgenesis. This is caused by either complete absence of or ectopic thyroid tissue. Congenital hypothyroidism is twice as common in females. Symptoms appear gradually. Prolonged jaundice may be the earliest sign. Poor feeding, somnolence, a large tongue, constipation, and an umbilical hernia may be present. The skin may be cold and mottled. The full clinical picture develops by 3-6 months. Growth and development are retarded. The fontanels stay widely open. Hair is coarse and brittle. There is generalized muscular hypotonia. Serum levels of total T4 are low, whereas thyrotropin (thyroid-stimulating hormone) levels are elevated. Newborn screening tests detect most cases of congenital hypothyroidism. Early diagnosis and treatment has greatly reduced the incidence of complications.

A 45-year-old man with no known medical history has had periodic "attacks" of headaches, sweating, and palpitations over the past 3 months. These episodes are associated with nausea and chest pain. They do not seem to be related to exercise or exertion. He reports that the episodes last from 1 to 5 minutes and then resolve on their own without intervention. His vital signs, physical examination, and electrocardiogram are normal. Which of the following is the most appropriate next test to establish the correct diagnosis?

Twenty-four-hour urine catecholamine. This patient has the classic findings of pheochromocytoma. These findings include episodes of headache, palpitations, and diaphoresis. This triad occurs in greater than 50% of patients. A normal physical examination in the office does not rule out this condition, because the symptoms occur in paroxysms. A patient's examination and blood pressure may be normal between attacks. A 24-hour urinary collection of catecholamines is the proper first step in diagnosis of this condition. The ideal time to perform this test is during or immediately after a hypertensive crisis. Beta-blockers and tricyclic antidepressants should be discontinued before collecting the urine, because these medications can interfere with these measurements. If pheochromocytoma is identified, alpha-blockers such as phenoxybenzamine (Dibenzyline) or doxazosin (Cardura) should be given initially followed by a beta-blocker such as metoprolol (Lopressor) to control acute hypertensive crisis caused by pheochromocytoma. Surgical resection of the tumor can also be performed.

A 58-year-old man undergoes a liver transplantation. The procedure goes well, without any noted complications, but 10 days later the patient's levels of gamma-glutamyl transferase (GGT), alkaline phosphatase, and bilirubin begin to rise. Which of the following is the most appropriate next step in diagnosis?

Ultrasound of biliary tract and Doppler studies of the anastomosed vessels. In all other solid organ transplants, deterioration of function 10 days out would suggest an acute rejection episode, and appropriate biopsies would be done to confirm the diagnosis. In the case of the liver, however, antigenic reactions are less common, whereas technical problems with the biliary and vascular anastomosis are the most common cause of early functional deterioration. They are, therefore, the first anomalies to be sought.

When is congenital hypothyroidism screened for?

day 1/2 after birth with heel stick blood sampling


Related study sets

Electrical Motor Controls, Chapters 21-22

View Set

Chapter 21 Gastrointestinal Disorders and Therapeutic Management

View Set

Case 9-4 Jasdip Properties SC, v. Estate of Richardson

View Set

اسئلة تقويم درس ابو بكر وعمر بن الخطاب + الاحاديث المطلوبة حفظ

View Set

practice exam Chapter 1 - Completing the application, underwriting, and Deli

View Set

Intro to Political Science Final (Reading Quizzes)

View Set